SlideShare a Scribd company logo
www.infimabih.com
567
INEQUALITY PROBLEMS WITH SOLUTIONS
IF
—A if a, b, c —re positive re—l num˜ersD then
a
b
+
b
c
+
c
a
≥
a2 + 1
b2 + 1
+
b2 + 1
c2 + 1
+
c2 + 1
a2 + 1
.
˜Avet a, b, c, d ˜e positive re—l num˜ersF€rove th—t
a2
− bd
b + 2c + d
+
b2
− ca
c + 2d + a
+
c2
− db
d + 2a + b
+
d2
− ac
a + 2b + c
≥ 0.
SolutionX
—Afy g—u™hyEƒ™hw—rz9s inequ—lityD ‡e h—veX
a2
+ b2
(a2 + 1) (b2 + 1) ≥ a2
+ b2
(ab + 1)
= ab a2
+ b2
+ a2
+ b2
≥ ab a2
+ b2
+ 2
⇒
a
b
+
b
a
=
a2
+ b2
ab
≥
a2
+ b2
+ 2
(a2 + 1) (b2 + 1)
=
a2 + 1
b2 + 1
+
b2 + 1
a2 + 1
fy ghe˜yshev9s inequ—lityD ‡e h—ve
a2
b2
=
a2
b2 + 1
+
a2
b2 (b2 + 1)
≥
a2
b2 + 1
+
b2
b2 (b2 + 1)
=
a2
+ 1
b2 + 1
.
„herefore
1 +
a
b
2
= 1 + 2
a
b
+
b
a
+
a2
b2
≥ 1 + 2
a2 + 1
b2 + 1
+
b2 + 1
a2 + 1
+
a2
+ 1
b2 + 1
= 1 +
a2 + 1
b2 + 1
2
.
„herefore
a
b
+
b
c
+
c
a
≥
a2 + 1
b2 + 1
+
b2 + 1
c2 + 1
+
c2 + 1
a2 + 1
—s requireF
˜Axoti™e th—t
2(a2
− bd)
b + 2c + d
+ b + d =
2a2
+ b2
+ d2
+ 2c(b + d)
b + 2c + d
=
(a − b)2
+ (a − d)2
+ 2(a + c)(b + d)
b + 2c + d
(1)
end simil—rlyD
2(c2
− db)
d + 2a + b
+ b + d =
(c − d)2
+ (c − b)2
+ 2(a + c)(b + d)
d + 2a + b
(2)
…sing g—u™hyEƒ™hw—rz9s inequ—lityDwe get
(a − d)2
b + 2c + d
+
(c − d)2
d + 2a + b
≥
[(a − b)2
+ (c − d)2
]
(b + 2c + d) + (d + 2a + b)
(3)
Q
(a − d)2
b + 2c + d
+
(c − b)2
d + 2a + b
≥
[(a − d)2
+ (c − b)2
]2
(b + 2c + d) + (d + 2a + b)
(4)
2(a + c)(b + d)
b + 2c + d
+
2(a + c)(b + d)
d + 2a + b
≥
8(a + c)(b + d)
(b + 2c + d) + (d + 2a + b)
(5)
prom @IAD@PAD@QAD@RA —nd @SAD we get
2(
a2
− bd
b + 2c + d
+
c2
− db
d + 2a + b
) + b + d ≥
(a + c − b − d)2
+ 4(a + c)(b + d)
a + b + c + d
= a + b + c + d.
or
a2
− bd
b + 2c + d
+
c2
− db
d + 2a + b
≥
a + c − b − d
2
sn the s—me m—nnerDwe ™—n —lso show th—t
b2
− ca
c + 2d + a
+
d2
− ac
a + 2b + c
≥
b + d − a − c
2
—nd ˜y —dding these two inequ—litiesDwe get the desired resultF
inqu—lity holds if —nd only if a = c —nd b = dF
PD
vet a, b, c ˜e positive re—l num˜ers su™h th—t
a + b + c = 1
€rove th—t the following inequ—lity holds
ab
1 − c2
+
bc
1 − a2
+
ca
1 − b2
≤
3
8
SolutionX prom the given ™ondition „he inequ—lity is equiv—lent to
4ab
a2 + b2 + 2(ab + bc + ca)
≤
3
2
˜ut from g—uhy ƒhw—rz inequ—lity
4ab
a2 + b2 + 2(ab + bc + ca)
≤
ab
a2 + ab + bc + ca
+
ab
b2 + ab + bc + ca
=
ab
(a + b)(a + c)
+
ab
(b + c)(a + b)
=
a(b + c)2
(a + b)(b + c)(c + a)
„hus ‡e need prove th—t
3(a + b)(b + c)(c + a) ≥ 2 a(b + c)2
whi™h redu™es to the o˜vious inequ—lity
ab(a + b) ≥ 6abc
„he Solution is ™ompletedFwith equ—lity if —nd only if
a = b = c =
1
3
R
yr ‡e ™—n use the f—™t th—t
4ab
a2 + b2 + 2(ab + bc + ca)
≤
4ab
(2ab + 2ac) + (2ab + 2bc)
≤
ab
2a(b + c)
+
ab
2b(a + c)
=
1
2
b
b + c
+
a
a + c
=
1
2
b
b + c
+
c
b + c
=
3
2
QD vet a, b, c ˜e the positive re—l num˜ersF €rove th—t
1 +
ab2
+ bc2
+ ca2
(ab + bc + ca)(a + b + c)
≥
4. 3
(a2 + ab + bc)(b2 + bc + ca)(c2 + ca + ab)
(a + b + c)2
SolutionX wultiplying ˜oth sides of the —˜ove inequ—lity with (a + b + c)2
it9s equiv—lent to
prove th—t
(a + b + c)2
+
(a + b + c)(ab2
+ bc2
+ ca2
)
ab + bc + ca
≥ 4. 3
(a2 + ab + bc)(b2 + bc + ca)(c2 + ca + ab)
‡e h—ve
(a + b + c)2
+
(a + b + c)(ab2
+ bc2
+ ca2
)
ab + bc + ca
=
(a2
+ ab + bc)(c + a)(c + b)
ab + bc + ca
fy using ewEqw inequ—lity ‡e get
(a2
+ ab + bc)(c + a)(c + b)
ab + bc + ca
≥ 3.
3
(a2 + ab + bc)(b2 + bc + ca)(c2 + ca + ab)[(a + b)(b + c)(c + a)]2
ab + bc + ca
ƒin™e it9s suffi™es to show th—t
√
3. 3
(a + b)(b + c)(c + a) ≥ 2.
√
ab + bc + ca
whi™h is ™le—rly true ˜y ewEqw inequ—lity —g—inF „he Solution is ™ompletedF iqu—lity
holds for a = b = c
RD
vet a0, a1, . . . , an ˜e positive re—l num˜ers su™h th—t ak+1 −ak ≥ 1 for —ll k = 0, 1, . . . , n−1.
€rove th—t
1 +
1
a0
1 +
1
a1 − a0
· · · 1 +
1
an − a0
≤ 1 +
1
a0
1 +
1
a1
· · · 1 +
1
an
SolutionX ‡e will prove it ˜y indu™tionF
por n = 1 ‡e need to ™he™k th—t
1 +
1
a0
1 +
1
a1 − a0
≤ 1 +
1
a0
1 +
1
a1
whi™h is equiv—lent to a0(a1 − a0 − 1) ≥ 0, whi™h is true ˜y given ™onditionF
vet
1 +
1
a0
1 +
1
a1 − a0
· · · 1 +
1
ak − a0
≤ 1 +
1
a0
1 +
1
a1
· · · 1 +
1
ak
S
it rem—ins to prove th—tX
1 +
1
a0
1 +
1
a1 − a0
· · · 1 +
1
ak+1 − a0
≤
≤ 1 +
1
a0
1 +
1
a1
· · · 1 +
1
ak+1
fy our hypothesis
1 +
1
a0
1 +
1
a1
· · · 1 +
1
ak+1
≥
≥ 1 +
1
ak+1
1 +
1
a0
1 +
1
a1 − a0
· · · 1 +
1
ak − a0
id estD it rem—ins to prove th—tX
1 +
1
ak+1
1 +
1
a0
1 +
1
a1 − a0
· · · 1 +
1
ak − a0
≥
≥ 1 +
1
a0
1 +
1
a1 − a0
· · · 1 +
1
ak+1 − a0
fut
1 +
1
ak+1
1 +
1
a0
1 +
1
a1 − a0
· · · 1 +
1
ak − a0
≥
≥ 1 +
1
a0
1 +
1
a1 − a0
· · · 1 +
1
ak+1 − a0
⇔
⇔
1
ak+1
+
1
ak+1a0
1 +
1
a1 − a0
· · · 1 +
1
ak − a0
≥
≥
1
(ak+1 − a0)a0
1 +
1
a1 − a0
· · · 1 +
1
ak − a0
⇔
⇔ 1 ≥
1
ak+1 − a0
1 +
1
a1 − a0
· · · 1 +
1
ak − a0
fut ˜y our ™onditions ‡e o˜t—inX
1
ak+1 − a0
1 +
1
a1 − a0
· · · 1 +
1
ak − a0
≤
≤
1
k
1 +
1
1
· · · 1 +
1
k − 1
= 1.
„husD the inequ—lity is provenF
SD
qiven a, b, c > 0F €rove th—t
3 a2 + bc
b2 + c2
≥ 9.
3
√
abc
(a + b + c)
Solution X „his ineq is equiv—lent toX
a2
+ bc
3
abc(a2 + bc)
2
(b2 + c2)
≥
9
(a + b + c)
3
fy ewEqw ineq D ‡e h—ve
a2
+ bc
3
abc(a2 + bc)
2
(b2 + c2)
=
T
=
a2
+ bc
3
(a2 + bc)c(a2 + bc)b(b2 + c2)a
≥
3(a2
+ bc)
sym
a2b
ƒimil—rlyD this ineq is true if ‡e prove th—tX
3(a2
+ b2
+ c2
+ ab + bc + ca)
sym
a2b
≥
9
(a + b + c)
3
a3
+ b3
+ c3
+ 3abc ≥
sym
a2
b
‡hi™h is true ˜y ƒ™hur ineqF iqu—lity holds when a = b = c
TD
vet a, b, c ˜e nonneg—tive re—l num˜ers su™h th—t ab + bc + ca > 0F €rove th—t
1
2a2 + bc
+
1
2b2 + ca
+
1
2c2 + ab
≥
2
ab + bc + ca
.
„he inequ—lity is equiv—lent to
ab + bc + ca
2a2 + bc
≥ 2, (1)
or
a(b + c)
2a2 + bc
+
bc
bc + 2a2
≥ 2.(2)
…sing the g—u™hyEƒ™hw—rz inequ—lityD ‡e h—ve
bc
bc + 2a2
≥
( bc)
2
bc(bc + 2a2)
= 1.(3)
„hereforeD it suffi™es to prove th—t
a(b + c)
2a2 + bc
≥ 1.(4)
ƒin™e
a(b + c)
2a2 + bc
≥
a(b + c)
2(a2 + bc)
it is enough to ™he™k th—t
a(b + c)
a2 + bc
≥ 2, (5)
whi™h is — known resultF
‚em—rkX
2ca + bc
2a2 + bc
+
2bc + ca
2b2 + ca
≥
4c
a + b + c
.
UD
vet a, b, c ˜e non neg—tive re—l num˜ers su™h th—t ab + bc + ca > 0F €rove th—t
1
2a2 + bc
+
1
2b2 + ca
+
1
2c2 + ab
+
1
ab + bc + ca
≥
12
(a + b + c)2
.
SolutionX IA ‡e ™—n prove this inequ—lity using the following —uxili—ry result
if 0 ≤ a ≤ min{a, b}D then
1
2a2 + bc
+
1
2b2 + ca
≥
4
(a + b)(a + b + c)
.
U
in f—™tD this is used to repl—™ed for 4no two of whi™h —re zero4D so th—t the fr—™tions
1
2a2 + bc
,
1
2b2 + ca
,
1
2c2 + ab
,
1
ab + bc + ca
h—ve me—ningsF
fesidesD the i—ker —lso works for itX
1
2a2 + bc
+
1
2b2 + ca
+
1
2c2 + ab
≥
2(ab + bc + ca)
a2b2 + abc(a + b + c)
fut our Solution for ˜oth of them is exp—nd
vet a, b, c ˜e non neg—tive re—l num˜ers su™h th—t ab + bc + ca > 0F €rove th—t
1
2a2 + bc
+
1
2b2 + ca
+
1
2c2 + ab
+
1
ab + bc + ca
≥
12
(a + b + c)2
.
PA gonsider ˜y ewEqw inequ—lityD ‡e h—ve
2 a2
+ ab + b2
(a + b + c)
= (2b + a) 2a2
+ bc + (2a + b) 2b2
+ ca
≥ 2 (2a + b)(2b + a) (2a2 + bc) (2b2 + ca).
end ˜y ewEqw inequ—lityD ‡e h—ve
c2
(2a + b)
2a2 + bc
+
c2
(2b + a)
2b2 + ca
≥ 2
c4(2a + b)(2b + a)
(2a2 + bc) (2b2 + ca)
≥
2c2
(2a + b)(2b + a)
(a2 + ab + b2) (a + b + c)
=
4c2
a + b + c
+
6abc
a + b + c
c
a2 + ab + b2
2c2
a + bc2
+ 2ab2
+ b2
c
2a2 + bc
=
c2
(2a + b)
2a2 + bc
+
c2
(2b + a)
2b2 + ca
≥
4c2
a + b + c
+
6abc
a + b + c
c
a2 + ab + b2
=
4 a2
+ b2
+ c2
a + b + c
+
6abc
a + b + c
c
a2 + ab + b2
≥
4 a2
+ b2
+ c2
a + b + c
+
6abc
a + b + c
(a + b + c)2
c (a2 + ab + b2)
=
4 a2
+ b2
+ c2
ab + c
+
6abc
ab + bc + ca
⇒
2a2
b + 2ab2
+ 2b2
c + 2bc2
+ 2c2
a + 2ca2
2a2 + bc
V
= (b + c) +
2c2
a + bc2
+ 2ab2
+ b2
c
2a2 + bc
≥ (b + c) +
4 a2
+ b2
+ c2
a + b + c
+
6abc
ab + bc + ca
=
8 a2
+ b2
+ c2
+ ab + bc + ca
a + b + c
−
2 a2
b + ab2
ab + bc + ca
⇒
1
2a2 + bc
+
1
ab + bc + ca
≥
4 a2
+ b2
+ c2
+ ab + bc + ca
(a + b + c) ( (a2b + ab2))
≥
12
(a + b + c)2
.
<=>
(a + b)(a + c)
2a2 + bc
+
a2
+ bc
2a2 + bc
− 2 ≥
12(ab + bc + ca)
(a + b + c)2
prom
2a2
+ 2bc
2a2 + bc
− 3 =
bc
2a2 + bc
≥ 1
‡e get
a2
+ bc
2a2 + bc
− 2 ≥ 0
xowD ‡e will prove the stronger
(a + b)(a + c)
2a2 + bc
≥
12(ab + bc + ca)
(a + b + c)2
prom ™—u™hyEs™h—rztD ‡e h—ve
(a + b)(a + c)
2a2 + bc
= (a+b)(b+c)(c+a)(
1
(2a2 + bc)(b + c)
≥
3(a + b)(b + c)(c + a)
ab(a + b) + bc(b + c) + ca(c + a)
pin—llyD ‡e only need to prove th—t
(a + b)(b + c)(c + a)
ab(a + b) + bc(b + c) + ca(c + a)
≥
4(ab + bc + ca)
(a + b + c)2
(a + b + c)2
ab + bc + ca
≥
4[ab(a + b) + bc(b + c) + ca(c + a)
(a + b)(b + c)(c + a)
= 4 −
8abc
(a + b)(b + c)(c + a)
a2
+ b2
+ c2
ab + bc + ca
+
8abc
(a + b)(b + c)(c + a)
≥ 2
whi™h is old pro˜lemF yur Solution —re ™ompleted equ—lity o™™ur if —nd if only
a = b = c, a = b, c = 0
or —ny ™y™li™ permutionF
VD vet a, b, c ˜e positive re—l num˜ers su™h th—t 16(a + b + c) ≥ 1
a + 1
b + 1
c F €rove th—t
1
a + b + 2(a + c)
3 ≤
8
9
.
SolutionX „his pro˜lem is r—ther e—syF …sing the ewEqw inequ—lityD ‡e h—veX
a + b + 2(c + a) = a + b +
c + a
2
+
c + a
2
≥ 3
3 (a + b)(c + a)
2
.
W
ƒo th—tX
1
a + b + 2(c + a)
3 ≤
2
27(a + b)(c + a)
.
„husD it9s enough to ™he™k th—tX
1
3(a + b)(c + a)
≤ 4 ⇐⇒ 6(a + b)(b + c)(c + a) ≥ a + b + c,
whi™h is true sin™e
9(a + b)(b + c)(c + a) ≥ 8(a + b + c)(ab + bc + ca)
—nd
16abc(a + b + c) ≥ ab + bc + ca ⇒
16(ab + bc + ca)2
3
≥ ab + bc + ca ⇐⇒ ab + bc + ca ≥
3
16
.
„he Solution is ™ompletedF iqu—lity holds if —nd only if a = b = c = 1
4 F
WD vet x, y, z ˜e positive re—l num˜ers su™h th—t xyz = 1F €rove th—t
x3
+ 1
x4 + y + z
+
y3
+ 1
y4 + z + x
+
z3
+ 1
z4 + x + y
≥ 2
√
xy + yz + zx.
SolutionX …sing the ewEqw inequ—lityD ‡e h—ve
2 (x4 + y + z)(xy + yz + zx) = 2 [x4 + xyz(y + z)](xy + yz + zx)
= 2 (x3 + y2z + yz2)(x2y + x2z + xyz)
≤ (x3
+ y2
z + yz2
) + (x2
y + x2
z + xyz)
= (x + y + z)(x2
+ yz) =
(x + y + z)(x3
+ 1)
x
.
it follows th—t
x3
+ 1
x4 + y + z
≥
2x
√
xy + yz + zx
x + y + z
.
edding this —nd it —n—logous inequ—litiesD the result followsF
IHD vet a, b, c ˜e nonneg—tive re—l num˜ers s—tisfying a + b + c =
√
5F €rove th—t
(a2
− b2
)(b2
− c2
)(c2
− a2
) ≤
√
5
SolutionX por this oneD ‡e ™—n —ssume ‡vyq th—t c ≥ b ≥ a so th—t ‡e h—ve
P = (a2
− b2
)(b2
− c2
)(c2
− a2
) = (c2
− b2
)(c2
− a2
)(b2
− a2
) ≤ b2
c2
(c2
− b2
).
elso note th—t
√
5 = a + b + c ≥ b + c sin™e a ≥ 0F xowD using the ewEqw inequ—lity ‡e
h—ve
(c + b) ·
√
5
2
− 1 · c
2
·
√
5
2
+ 1 b
2
· (c − b)
≤ (c + b)
√
5(b + c)
5
5
≤
√
5;
ƒo th—t ‡e get P ≤
√
5F end hen™e ‡e —re doneF iqu—lity holds if —nd only if (a, b, c) =√
5
2 + 1;
√
5
2 − 1; 0 —nd —ll its ™y™li™ permut—tionsF 2
IH
IID vet a, b, c > 0 —nd a + b + c = 3F €rove th—t
1
3 + a2 + b2
+
1
3 + b2 + c2
+
1
3 + c2 + a2
≤
3
5
SolutionX ‡e h—veX
1
3 + a2 + b2
+
1
3 + b2 + c2
+
1
3 + c2 + a2
≤
3
5
<=>
3
3 + a2 + b2
+
3
3 + b2 + c2
+
3
3 + c2 + a2
≤
9
5
a2
+ b2
3 + a2 + b2
≥
6
5
…sing g—u™hyEƒ™hw—rz9s inequ—lityX
a2
+ b2
3 + a2 + b2
( 3 + a2
+ b2
) ≥ ( a2 + b2)2
„h—t me—ns ‡e h—ve to prove
( a2 + b2)2
≥
6
5
( (3 + a2
+ b2
))
(a2
+ b2
) + 2 (a2 + b2)(a2 + c2) ≥
54
5
+
12
5
a2
8 a2
+ 10 ab ≥ 54 <=> 5(a + b + c)2
+ 3 a2
≥ 54
it is true with a + b + c = 3F
IPD
qiven a, b, c > 0 su™h th—t ab + bc + ca = 1F €rove th—t
1
4a2 − bc + 1
+
1
4b2 − ca + 1
+
1
4c2 − ab + 1
≥ 1
SolutionX in f—™tD the sh—rper inequ—lity holds
1
4a2 − bc + 1
+
1
4b2 − ca + 1
+
1
4c2 − ab + 1
≥
3
2
.
„he inequ—lity is equiv—lent to
1
a(4a + b + c)
+
1
b(4b + c + a)
+
1
c(4c + a + b)
≥
3
2
.
…sing the g—u™hyEƒ™hw—rz inequ—lityD ‡e h—ve
1
a(4a + b + c)
4a + b + c
a
≥
1
a
2
=
1
a2b2c2
.
„hereforeD it suffi™es to prove th—t
2
3a2b2c2
≥
4a + b + c
a
+
4b + c + a
b
+
4c + a + b
c
.
ƒin™e
4a + b + c
a
= 3 +
a + b + c
a
= 9 +
(a + b + c)(ab + bc + ca)
abc
= 9 +
a + b + c
abc
,
II
this inequ—lity ™—n ˜e written —s
9a2
b2
c2
+ abc(a + b + c) ≤
2
3
,
whi™h is true ˜e™—use
a2
b2
c2
≤
ab + bc + ca
3
3
=
1
27
,
—nd
abc(a + b + c) ≤
(ab + bc + ca)2
3
=
1
3
.
IQD qiven a, b, c ≥ 0 su™h th—t ab + bc + ca = 1F €rove th—t
1
4a2 − bc + 2
+
1
4b2 − ca + 2
+
1
4c2 − ab + 2
≥ 1
SolutionX xoti™e th—t the ™—se abc = 0 is trivi—l so let us ™onsider now th—t abc > 0F …sing
the ewEqw inequ—lityD ‡e h—ve
4a2
− bc + 2(ab + bc + ca) = (2a + b)(2a + c) ≤
[c(2a + b) + b(2a + c)]2
4bc
=
(ab + bc + ca)2
bc
=
1
bc
.
it follows th—t
1
4a2 − bc + 2
≥ bc.
edding this —nd its —n—logous inequ—litiesD ‡e get the desired resultF
IRD qiven a, b, c —re positive re—l num˜ersF €rove th—t
(
1
a
+
1
b
+
1
c
)(
1
1 + a
+
1
1 + b
+
1
1 + c
) ≥
9
1 + abc
.
SolutionX „he origin—l inequ—lity is equiv—lent to
abc + 1
a
+
abc + 1
b
+
abc + 1
c
1
a + 1
+
1
b + 1
+
1
c + 1
≥ 9
or
cyc
1 + a2
c
a
1
a + 1
+
1
b + 1
+
1
c + 1
≥ 9
fy g—u™hy ƒ™hw—rz ineq —nd ewEqw ineqD
cyc
1 + a2
c
a
≥
cyc
c(1 + a)2
a(1 + c)
≥ 3 3
(1 + a)(1 + b)(1 + c)
—nd
1
a + 1
+
1
b + 1
+
1
c + 1
≥
3
3
(1 + a)(1 + b)(1 + c)
wultiplying these two inequ—litiesD the ™on™lusion followsF iqu—lity holds if —nd only if
a = b = c = 1F
ISF qiven a, b, c —re positive re—l num˜ersF €rove th—tX
a(b + 1) + b(c + 1) + c(a + 1) ≤
3
2
(a + 1)(b + 1)(c + 1)
IP
SolutionX g—seIFif a + b + c + ab + bc + ca ≤ 3abc + 3 <=> 4(ab + bc + ca + a + b + c) ≤
3(a + 1)(b + 1)(c + 1) …sing g—u™hyEƒ™h—wrz9s inequ—lity D‡e h—veX
( a(b + 1) + b(c + 1) + c(a + 1))2
≤ 3(ab + bc + ca + a + b + c) ≤
9(a + 1)(b + 1)(c + 1)
4
„he inequ—lity is trueF g—sePF ifa + b + c + ab + bc + ca ≤ 3abc + 3.
<=>
9(a + 1)(b + 1)(c + 1)
4
≥ 2(a + b + c + ab + bc + ca) + 3abc + 3
fy ewEqw9s inequ—lity X
2 ab(b + 1)(c + 1) ≤ [ab(c + 1) + (b + 1)] = a + b + c + ab + bc + ca + 3abc + 3
=> ab + bc + ca + a + b + c + 2 ab(b + 1)(c + 1) ≤
9
4(a + 1)(b + 1)(c + 1)
=> ( a(b + 1) + b(c + 1) + c(a + 1))2
≤ [
3
2
(a + 1)(b + 1)(c + 1)]2
=> Q.E.D
inqu—lity holds when a = b = c = 1.
ITD qiven a, b, c —re positive re—l num˜ersF €rove th—tX
1
a2 + b2
+
1
b2 + c2
+
1
c2 + a2
≥
10
(a + b + c)2
SolutionX essume c = min{a, b, c}F „hen
1
a2 + c2
+
1
b2 + c2
≥
2
ab + c2
⇐⇒ (ab − c2
)(a − b)2
≥ 0
end ˜y g—u™hyEs™hw—rz
((a2
+ b2
) + 8(ab + c2
))
1
a2 + b2
+
2
ab + c2
≥ 25
ren™e ‡e need only to proveX
5(a + b + c)2
≥ 2((a2
+ b2
) + 8(ab + c2
)) ⇐⇒
3(a − b)2
+ c(10b + 10a − 11c) ≥ 0
iqu—lity for a = b, c = 0 or permut—tionsF
IUD vet a, b —nd c —re nonEneg—tive num˜ers su™h th—t ab + ac + bc = 0F €rove th—t
a2
(b + c)2
a2 + 3bc
+
b2
(a + c)2
b2 + 3ac
+
c2
(a + b)2
c2 + 3ab
≤ a2
+ b2
+ c2
Solution:
fy g—u™hyEƒ™hw—rz ineq D ‡e h—ve
a2
(b + c)
2
a2 + bc
=
a2
(b + c)
3
(a2 + bc)(b + c)
=
a2
(b + c)
3
b(a2 + c2) + c(a2 + b2)
≤
a2
(b + c)
4
(
b2
b(a2 + c2)
+
c2
c(a2 + b2)
) =
a2
(b + c)
4
(
b
a2 + c2
+
c
a2 + b2
)
ƒimil—rlyD ‡e h—ve
LHS ≤ a2
(b + c)(
b
a2 + c2
+
c
a2 + b2
) =
c(a2
(b + c) + b2
(c + a))
a2 + b2
IQ
= a2
+ b2
+ c2
+
abc(a + b)
a2 + b2
≤ a2
+ b2
+ c2
+
abc(a + b)
a2 + b2
≤ a2
+ b2
+ c2
+ ab + bc + ca
whi™h is true ˜y ewEqw ineq
„he origin—l inequ—lity ™—n ˜e written —s
(a + b)2
(a + c)2
a2 + bc
≤
8
3
(a + b + c)2
.
ƒin™e (a + b)(a + c) = (a2
+ bc) + a(b + c) ‡e h—ve
(a + b)2
(a + c)2
a2 + bc
=
(a2
+ bc)2
+ 2a(b + c)(a2
+ bc) + a2
(b + c)2
a2 + bc
= a2
+ bc + 2a(b + c) +
a2
(b + c)2
a2 + bc
,
—nd thus the —˜ove inequ—lity is equiv—lent to
a2
(b + c)2
a2 + bc
≤
8
3
(a + b + c)2
− a2
− 5 ab,
or
a2
(b + c)2
a2 + bc
≤
5(a2
+ b2
+ c2
) + ab + bc + ca
3
.
ƒin™e
5(a2
+ b2
+ c2
) + ab + bc + ca
3
≥ a2
+ b2
+ c2
+ ab + bc + ca
it is enough show th—t
a2
(b + c)2
a2 + bc
≤ a2
+ b2
+ c2
+ ab + bc + ca.
FiFh
IVD qiven
a1 ≥ a2 ≥ . . . ≥ an ≥ 0, b1 ≥ b2 ≥ . . . ≥ bn ≥ 0
n
i=1
ai = 1 =
n
i=1
bi
pind the m—xmium of
n
i=1
(ai − bi)2
‡SolutionXithout loss of gener—lityD —ssume th—t
a1 ≥ b1
xoti™e th—t for
a ≥ x ≥ 0, b, y ≥ 0
‡e h—ve
(a − x)2
+ (b − y)2
− (a + b − x)2
− y2
= −2b(a − x + y) ≤ 0.
e™™ording to this inequ—lityD ‡e h—ve
(a1 − b1)2
+ (a2 − b2)2
≤ (a1 + a2 − b1)2
+ b2
2,
(a1 + a2 − b1)2
+ (a3 − b3)2
≤ (a1 + a2 + a3 − b1)2
+ b2
3, · · · · · ·
IR
(a1 + a2 + · · · + an−1 − b1)2
+ (an − bn)2
≤ (a1 + a2 + · · · + an − b1)2
+ b2
n.
edding these inequ—litiesD ‡e get
n
i=1
(ai − bi)2
≤ (1 − b1)2
+ b2
2 + b2
3 + · · · + b2
n
≤ (1 − b1)2
+ b1(b2 + b3 + · · · + bn)
= (1 − b1)2
+ b1(1 − b1) = 1 − b1 ≤ 1 −
1
n
.
iqu—lity holds for ex—mple when
a1 = 1, a2 = a3 = · · · = an = 0
—nd
b1 = b2 = · · · = bn =
1
n
IWD qiven
a, b, c ≥ 0
su™h th—t
a2
+ b2
+ c2
= 1
€rove th—t
1 − ab
7 − 3ac
+
1 − bc
7 − 3ba
+
1 − ca
7 − 3cb
≥
1
3
SolutionX pirstD ‡e will show th—t
1
7 − 3ab
+
1
7 − 3bc
+
1
7 − 3ca
≤
1
2
.
…sing the g—u™hyEƒ™hw—rz inequ—lityD ‡e h—ve
1
7 − 3ab
=
1
3(1 − ab) + 4
≤
1
9
1
3(1 − ab)
+ 1 .
it follows th—t
1
7 − 3ab
≤
1
27
1
1 − ab
+
1
3
,
—nd thusD it is enough to show th—t
1
1 − ab
+
1
1 − bc
+
1
1 − ca
≤
9
2
,
whi™h is †—s™9s inequ—lityF xowD ‡e write the origin—l inequ—lity —s
3 − 3ab
7 − 3ac
+
3 − 3bc
7 − 3ba
+
3 − 3ca
7 − 3cb
≥ 1,
or
7 − 3ab
7 − 3ac
+
7 − 3bc
7 − 3ba
+
7 − 3ca
7 − 3cb
≥ 1 + 4
1
7 − 3ab
+
1
7 − 3bc
+
1
7 − 3ca
.
ƒin™e
4
1
7 − 3ab
+
1
7 − 3bc
+
1
7 − 3ca
≤ 2
IS
it is enough to show th—t
7 − 3ab
7 − 3ac
+
7 − 3bc
7 − 3ba
+
7 − 3ca
7 − 3cb
≥ 3,
whi™h is true —™™ording to the ewEqw inequ—lityF
PID vet
a, b, c ≥ 0
su™h th—t
a + b + c > 0
—nd
b + c ≥ 2a
por
x, y, z > 0
su™h th—t
xyz = 1
€rove th—t the following inequ—lity holds
1
a + x2(by + cz)
+
1
a + y2(bz + cx)
+
1
a + z2(bx + cy)
≥
3
a + b + c
SolutionX ƒetting
u =
1
x
, v =
1
y
—nd
w =
1
z
—nd using the ™ondition
uvw = 1
the inequ—lity ™—n ˜e rewritten —s
u
au + cv + bw
=
u2
au2 + cuv + bwu
3
a + b + c
F
epplying g—u™hyD it suffi™es to prove
(u + v + w)
2
a u2 + (b + c) uv
3
a + b + c
1
2
· (b + c − 2a) (x − y)2
0D
whi™h is o˜vious due to the ™ondition for
a, b, c
PPD qiven
x, y, z > 0
su™h th—t
xyz = 1
IT
€rove th—t
1
(1 + x2)(1 + x7)
+
1
(1 + y2)(1 + y7)
+
1
(1 + z2)(1 + z7)
≥
3
4
SolutionX pirst ‡e prove this ineq e—sy
1
(1 + x2)(1 + x7)
≥
3
4(x9 + x
9
2 + 1)
end this ineq ˜e™—meX
1
x9 + x
9
2 + 1
+
1
y9 + y
9
2 + 1
+
1
z9 + z
9
2 + 1
≥ 1
with
xyz = 1
it9s —n old result
PQD vet
a, b, c
˜e positive re—l num˜ers su™h th—t
3(a2
+ b2
+ c2
) + ab + bc + ca = 12
€rove th—t
a
√
a + b
+
b
√
b + c
+
c
√
c + a
≤
3
√
2
.
SolutionX vet
A = a2
+ b2
+ c2
, B = ab + bc + ca
2A + B = 2 a2
+ ab ≤
3
4
3 a2
+ ab = 9.
fy g—u™hy ƒ™hw—rz inequ—lityD ‡e h—ve
a
√
a + b
=
√
a
a
a + b
≤
√
a + b + c
a
a + b
.
fy g—u™hy ƒ™hw—rz inequ—lity —g—inD ‡e h—ve
b
a + b
=
b2
b(a + b)
≥
(a + b + c)2
b(a + b)
=
A + 2B
A + B
a
a + b
= 3 −
b
a + b
≤ 3 −
A + 2B
A + B
=
2A + B
A + B
hen™eD it suffi™es to prove th—t
(a + b + c) ·
2A + B
A + B
≤
9
2
IU
gonsider
(a + b + c)
√
2A + B
= (A + 2B) (2A + B)
≤
(A + 2B) + (2A + B)
2
=
3
2
(A + B)
⇒ (a + b + c) ·
2A + B
A + B
≤
3
2
√
2A + B ≤
9
2
—s requireF
fy ewEqw ineq e—sy to see th—t
3 ≤ a2
+ b2
+ c2
≤ 4
fy g—u™hyEƒ™hw—rz ineqD ‡e h—ve
LHS2
= (
a
√
a + c
(a + b)(a + c)
) ≤ (a2
+ b2
+ c2
+ ab + bc + ca)(
a
(a + b)(a + c)
)
…sing the f—mili—r ineq
9(a + b)(b + c)(c + a) ≥ 8(a + b + c)(ab + bc + ca)
‡e h—ve
a
(a + b)(a + c)
=
2(ab + bc + ca)
(a + b)(b + c)(c + a)
≤
9
4(a + b + c)
end ‡e need to prove th—t
9(a2
+ b2
+ c2
+ ab + bc + ca)
4(a + b + c)
≤
9
2
⇔
6 − (a2
+ b2
+ c2
)
24 − 5(a2 + b2 + c2)
≤ 1
⇔ (6 − (a2
+ b2
+ c2
))2
≤ 24 − 5(a2
+ b2
+ c2
)
⇔ (3 − (a2
+ b2
+ c2
))(4 − (a2
+ b2
+ c2
)) ≤ 0
‡hi™h is true ‡e —re done equ—lity holds when
a = b = c = 1
PRF
qiven
a, b, c ≥ 0
€rove th—t
1
(a2 + bc)(b + c)2
≤
8(a + b + c)2
3(a + b)2(b + c)2(c + a)2
SolutionX in f—™tD the sh—rper —nd ni™er inequ—lity holdsX
a2
(b + c)2
a2 + bc
+
b2
(c + a)2
b2 + ca
+
c2
(a + b)2
c2 + ab
≤ a2
+ b2
+ c2
+ ab + bc + ca.
a2
(b + c)2
a2 + bc
+
b2
(c + a)2
b2 + ca
+
c2
(a + b)2
c2 + ab
≤ a2
+ b2
+ c2
+ ab + bc + ca
IV
PSF
qiven
a, b, c ≥ 0
su™h th—t
ab + bc + ca = 1
€rove th—t
1
8
5 a2 + bc
+
1
8
5 b2 + ca
+
1
8
5 c2 + ab
≥
9
4
essume ‡vyq
a ≥ b ≥ c
this ineq
1
8
5 a2 + bc
−
5
8
+
1
8
5 b2 + ca
−
5
8
+
1
8
5 c2 + ab
− 1 ≥ 0
8 − 8a2
− 5bc
8a2 + 5bc
+
8 − 8b2
− 5ca
8b2 + 5ca
+
1 − 8
5 c2
− ab
c2 + 8
5 ab
≥ 0
8a(b + c − a) + 3bc
8a2 + 5bc
+
8b(a + c − b) + 5ac
8b2 + 5ca
+
c(a + b − 8
5 c)
c2 + 8
5 ab
≥ 0
xoti™e th—t ‡e only need to prove this ineq when
a ≥ b + c
˜y the w—y ‡e need to prove th—t
8b
8b2 + 5ca
≥
8a
8a2 + 5bc
(a − b)(8ab − 5ac − 5bc) ≥ 0
i—sy to see th—tX if
a ≥ b + c
then
8ab = 5ab + 3ab ≥ 5ac + 6bc ≥ 5ac + 5ac
ƒo this ineq is trueD ‡e h—ve qFdFe D equ—lity hold when
(a, b, c) = (1, 1, 0)
PTD qive
a, b, c ≥ 0
€rove th—tX
a
b2 + c2
+
b
a2 + c2
+
c
a2 + b2
≥
a + b + c
ab + bc + ca
+
abc(a + b + c)
(a3 + b3 + c3)(ab + bc + ca)
a
b2 + c2
=
a2
ab2 + c2a
≥
(a + b + c)2
(ab2 + c2a)
,
it suffi™es to prove th—t
a + b + c
(ab2 + c2a)
≥
1
ab + bc + ca
+
abc
(ab + bc + ca) (a3 + b3 + c3)
,
IW
˜e™—use
a + b + c
(ab2 + c2a)
−
1
ab + bc + ca
=
3abc
(ab + bc + ca) (ab2 + ca2)
,
it suffi™es to prove th—t
3 a3
+ b3
+ c3
≥ ab2
+ c2
a ,
whi™h is true ˜e™—use
2 a3
+ b3
+ c3
≥ ab2
+ c2
a .
‚em—rkX
a
b2 + c2
+
b
c2 + a2
+
c
a2 + b2
≥
a + b + c
ab + bc + ca
+
3abc(a + b + c)
2(a3 + b3 + c3)(ab + bc + ca)
.
qive
a, b, c ≥ 0
€rove th—t
1
a2 + bc
+
1
b2 + ca
+
1
c2 + ab
≥
3
ab + bc + ca
+
81a2
b2
c2
2(a2 + b2 + c2)4
iqu—lity o™™ur if —nd if only
a = b = c, a = b, c = 0
or —ny ™y™li™ permutionF
it is true ˜e™—use
(1)
1
a2 + bc
+
1
b2 + ca
+
1
c2 + ab
≥
3 a2
+ b2
+ c2
a3b + ab3 + b3c + bc3 + c3a + ca3
—nd
(2)
3 a2
+ b2
+ c2
a3b + ab3 + b3c + bc3 + c3a + ca3
≥
3
ab + bc + ca
+
81a2
b2
c2
2(a2 + b2 + c2)4
.
fe™—use
a2
(a3b + ab3)
−
1
ab + bc + ca
=
abc(a + b + c)
(ab + bc + ca) ( (a3b + ab3))
,
it suffi™es to prove th—t
2(a + b + c) a2
+ b2
+ c2 4
≥ 27abc(ab + bc + ca) a3
b + ab3
,
whi™h is true ˜e™—use
(a) (a + b + c) a2
+ b2
+ c2
≥ 9abc,
(b) a2
+ b2
+ c2
≥ ab + bc + ca,
(c) 2 a2
+ b2
+ c2 2
≥ 3 a3
b + ab3
,
whi™h
(c)
PH
is equiv—lent to
a2
− ab + b2
(a − b)2
≥ 0,
whi™h is trueF
PUD vet
a, b, c
˜e nonneg—tive num˜ersD no two of whi™h —re zeroF €rove th—t
a2
(b + c)
b2 + bc + c2
+
b2
(c + a)
c2 + ca + a2
+
c2
(a + b)
a2 + ab + b2
2(a2
+ b2
+ c2
)
a + b + c
.
SolutionX
a2
(b + c)
b2 + bc + c2
=
4a2
(b + c)(ab + bc + ca)
(b2 + bc + c2) (ab + bc + ca)
≥
4a2
(b + c)(ab + bc + ca)
(b2 + bc + c2 + ab + bc + ca)
2
=
4a2
(ab + bc + ca)
(b + c)(a + b + c)2
,
it suffi™es to prove
a2
b + c
≥
(a + b + c) a2
+ b2
+ c2
2(ab + bc + ca)
,
or
a2
b + c
+ a ≥
(a + b + c)3
2(ab + bc + ca)
,
or
a
b + c
≥
(a + b + c)2
2(ab + bc + ca)
,
whi™h is true ˜y g—u™hyEƒ™hw—rz inequ—lity
a
b + c
=
a2
a(b + c)
≥
(a + b + c)2
2(ab + bc + ca)
.
‡e just w—nt to give — little note hereF xoti™e th—t
a2
(b + c)
b2 + bc + c2
+
a(b + c)
a + b + c
=
a(b + c)(a2
+ b2
+ c2
+ ab + bc + ca)
(b2 + bc + c2)(a + b + c)
,
—nd
2(a2
+ b2
+ c2
)
a + b + c
+
a(b + c)
a + b + c
=
2(a2
+ b2
+ c2
+ ab + bc + ca)
a + b + c
.
„hereforeD the inequ—lity ™—n ˜e written in the form
a(b + c)
b2 + bc + c2
+
b(c + a)
c2 + ca + a2
+
c(a + b)
a2 + ab + b2
≥ 2,
xote th—t
cyc
a(b + c)
b2 + bc + c2
=
cyc
4a(b + c)(ab + bc + ca)
4(b2 + bc + c2)(ab + bc + ca) cyc
4a(ab + bc + ca)
(b + c)(a + b + c)2
.
PI
ƒo th—t ‡e h—ve to proveX
cyc
4a(ab + bc + ca)
(b + c)(a + b + c)2
2,
or
cyc
a
b + c
(a + b + c)2
2(ab + bc + ca)
,
whi™h is o˜viously true due to the g—u™hyEƒ™hw—rz inequ—lityF
„his is —nother new SolutionF pirstD ‡e will prove th—t
(a2 + ac + c2)(b2 + bc + c2) ≤
ab(a + b) + bc(b + c) + ca(c + a)
a + b
.(1)
indeedD using the g—u™hyEƒ™hw—rz inequ—lityD ‡e h—ve
√
ac ·
√
bc + a2 + ac + c2 · b2 + bc + c2 ≤ (ac + a2 + ac + c2)(bc + b2 + bc + c2)
= (a + c)(b + c).
it follows th—t
(a2 + ac + c2)(b2 + bc + c2) ≤ ab + c2
+ c a + b −
√
ab ≤ ab + c2
+ c a + b −
2ab
a + b
=
ab(a + b) + bc(b + c) + ca(c + a)
a + b
.
xowD from @IAD using the ewEqw inequ—lityD ‡e get
1
a2 + ac + c2
+
1
b2 + bc + c2
≥
2
(a2 + ac + c2)(b2 + bc + c2)
≥
2(a + b)
ab(a + b) + bc(b + c) + ca(c + a)
.
(2)
prom
(2)
‡e h—ve
a(b + c)
b2 + bc + c2
= ab
1
a2 + ac + c2
+
1
b2 + bc + c2
≥
2ab(a + b)
ab(a + b) + bc(b + c) + ca(c + a)
= 2.
PWD if
a, b, c > 0
then the following inequ—lity holdsX
a2
(b + c)
b2 + bc + c2
+
b2
(c + a)
c2 + ca + a2
+
c2
(a + b)
a2 + ab + b2
≥ 2
a3 + b3 + c3
a + b + c
„his inequ—lity is equiv—lent to
a2
(b + c)(a + b + c)
b2 + bc + c2
≥ 2 (a3 + b3 + c3) (a + b + c)
or
a2
+
a2
(ab + bc + ca)
b2 + bc + c2
≥ 2 (a3 + b3 + c3) (a + b + c),
PP
˜e™—use
2 (a3 + b3 + c3) (a + b + c) ≤ a2
+ b2
+ c2
+
a3
+ b3
+ c3
(a + b + c)
a2 + b2 + c2
,
it suffi™es to prove th—t
a2
b2 + bc + c2
≥
a3
+ b3
+ c3
(a + b + c)
(a2 + b2 + c2) (ab + bc + ca)
,
˜y g—u™hyEƒ™hw—rz inequ—lityD ‡e h—ve
a2
b2 + bc + c2
≥
a2
+ b2
+ c2 2
a2 (b2 + bc + c2)
=
a2
+ b2
+ c2 2
2 a2b2 + a2bc
,
it suffi™es to prove th—t
a2
+ b2
+ c2 3
(ab + bc + ca) ≥ a3
+ b3
+ c3
(a + b + c) 2 a2
b2
+ a2
bc .
vet
A = a4
, B =
1
2
a3
b + ab3
, C = a2
b2
, D = a2
bc,
‡e h—ve
a2
+ b2
+ c2 2
= A + 2C,
a2
+ b2
+ c2
(ab + bc + ca) = 2B + D,
a3
+ b3
+ c3
(a + b + c) = A + 2B,
—nd
2 a2
b2
+ a2
bc = 2C + D.
„hereforeD it suffi™es to prove th—t
(A + 2C) (2B + D) ≥ (A + 2B) (2C + D) ,
or
2 (A − D) (B − C) ≥ 0,
whi™h is true ˜e™—use
A ≥ D
—nd
B ≥ C
QHD qiven
a, b, c ≥ 0
su™h th—t
a + b + c = 1
€rove th—t
2 a2b + b2c + c2a + ab + bc + ca ≤ 1
‚ewrite the inform inequ—lity —s
2 a2b + b2c + c2a + ab + bc + ca ≤ (a + b + c)2
PQ
2 (a2b + b2c + c2a) (a + b + c) ≤ a2
+ b2
+ c2
+ ab + bc + ca
essume th—t ˜ is the num˜er ˜etien — —nd ™F „henD ˜y —pplying the ewEqw inequ—lityD ‡e
get
2 (a2b + b2c + c2a) (a + b + c) ≤
a2
b + b2
c + c2
a
b
+ b(a + b + c)
it is thus suffi™ient to prove the stronger inequ—lity
a2
+ b2
+ c2
+ ab + bc + ca ≥
a2
b + b2
c + c2
a
b
+ b(a + b + c)
„his inequ—lity is equiv—lent to
c(a − b)(b − c)
b
≥ 0,
whi™h is o˜viously true —™™ording to the —ssumption of
b
row to prove
a4
+ 2 a3
c ≥ a2
b2
+ 2 a3
b
only ˜y ewEqw iquiv—lent to prove
(a − b)2
(a + b)2
≥ 4(a − b)(b − c)(a − c)(a + b + c)
‡vyq ‡e ™—n —ssume th—t
a ≥ b ≥ c, a − b = x, b − c = y
then ‡e need to prove th—t
x2
(2c + 2y + x)2
+ y2
(2c + y)2
+ (x + y)2
(2c + x + y)2
≥ xy(x + y)(3c + 2x + y)
˜y
(x + y)4
≥ xy(x + y)(x + 2y)
—nd
(x + y)3
≥ 3xy(x + y)
‡e h—ve ™ompleted the Solution
QID vet
a, b, c
˜e positive num˜ers su™h th—t
a2
b2
+ b2
c2
+ c2
a2
≥ a2
b2
c2
pind the minimum of e
A =
a2
b2
c3(a2 + b2)
+
b2
c2
a3(b2 + c2)
+
c2
a2
b3(c2 + a2)
xo one like this pro˜lemc ƒetting
x =
1
a
, y =
1
b
, z =
1
c
PR
‡e h—ve
x2
+ y2
+ z2
≥ 1
‡e will prove th—t
x3
y2 + z2
+
y3
x2 + z2
+
z3
x2 + y2
≥
√
3
2
…sing g—u™hyEƒ™hw—rzX
LHS ≥
(x2
+ y2
+ z2
)2
x(y2 + z2) + y(x2 + z2) + z(x2 + y2)
fy ewEqw ‡e h—veX
x(y2
+z2
)+y(x2
+z2
)+z(x2
+y2
) ≤
2
3
(x2
+y2
+z2
)(x+y+z) ≤
2
√
3
(x2
+y2
+z2
) x2 + y2 + z2
fe™—use
x2
+ y2
+ z2
≥ 1
ƒo
(x2
+ y2
+ z2
)2
2√
3
(x2 + y2 + z2) x2 + y2 + z2
≥
√
3
2
‡e done3
QPF
vet xDyDz ˜e non neg—tive re—l num˜ers su™h th—t x2
+ y2
+ z2
= 1
F find the minimum —nd m—ximum of f = x + y + z − xyz.
Solution IF
pirst ‡e fix z —nd let m = x+y = x+
√
1 − x2 − z2 = g(x)(0 ≤ x ≤
√
1 − z2), then ‡e h—ve
g (x) = 1 −
x
√
1 − x2 − z2
,
‡e get
g (x) > 0 ⇔ 0 ≤ x <
1 − z2
2
—nd
g (x) < 0 ⇔
1 − z2
2
< x ≤ 1 − z2,
so ‡e h—ve
mmin = min{g(0), g( 1 − z2)} = 1 − z2
—nd
mmax = g
1 − z2
2
= 2 − 2z2.
e™tu—llyD f —nd written —s
f = f(m) = −
z
2
m2
+ m + 1 − z2 z
2
+ z,
e—sy to prove th—t the —xis of symmetry
m =
1
z
> 2 − 2z2
PS
so f(m) is in™re—sing in the interv—l of mD thusD ‡e h—ve
f(m) ≥ f( 1 − z2) = 1 − z2 + z
—nd
f(m) ≤ f( 2 − 2z2) =
z3
2
+
z
2
+ 2 − 2z2.
ƒin™e
( 1 − z2 + z)2
= 1 + 2z 1 − z2 ≥ 1
‡e get f(m) ≥ 1 —nd when two of xDyDz —re zero ‡e h—ve f = 1, soWegetfmin = 1.
vet
h(z) =
z3
2
+
z
2
+ 2 − 2z2,
e—sy to prove th—t
h (z) > 0 ⇔ 0 ≤ z <
1
√
3
andh (z) < 0 ⇔
1
√
3
< z ≤ 1
then ‡e get
f(m) ≤ h
1
√
3
=
8
√
3
9
,
when x = y = z =
1
√
3
Wehavef =
8
√
3
9
D so ‡e getfmax =
8
√
3
9
.
honeF
Solution PF
‡hen two of xDyDz —re zero ‡e h—vef = 1D —nd ‡e will prove th—t f ≥ 1 then ‡e ™—n get
fmin = 1F e™tu—llyD ‡e h—ve
f ≥ 1 ⇔ x + y + z − xyz ≥ 1 ⇔ (x + y + z) x2
+ y2
+ z2
− xyz ≥
x2 + y2 + z2
3
⇔ (x + y + z) x2
+ y2
+ z2
− xyz
2
≥
x2
+ y2
+ z2 3
⇔ x2
y2
z2
+ 2
sym
x5
y + x3
y3
+ x3
y2
z ≥ 0,
the l—st inequ—lity is o˜vious trueD so ‡e got f ≥ 1; ‡henx = y = z =
1
√
3
‡e h—ve
f =
8
√
3
9
,
—nd ‡e will prove th—t
f ≤
8
√
3
9
then ‡e ™—n get
fmax =
8
√
3
9
e™tu—llyD ‡e h—ve
f ≤
8
√
3
9
⇔ x + y + z − xyz ≤
8
√
3
9
⇔ (x + y + z) x2
+ y2
+ z2
− xyz ≤
8
√
3
9
x2 + y2 + z2
3
⇔ 27 (x + y + z) x2
+ y2
+ z2
− xyz
2
≤ 64 x2
+ y2
+ z2 3
⇔
1
4 cyc
S (x, y, z) (y − z)
2
≥ 0,
PT
where
S(x, y, z) = 17y2
(2y−x)2
+17z2
(2z−x)2
+56y2
(z−x)2
++56z2
(y−x)2
+24x4
+6y4
+6z4
+57x2
(y2
+z2
)+104y2
z2
is o˜vious positiveD so the l—st inequ—lity is o˜vious trueD so ‡e gotfmax =
8
√
3
9
.
QQD por positive re—l num˜ersD show th—t
a3
(b + c − a)
a2 + bc
+
b3
(c + a − b)
b2 + ca
+
c3
(a + b − c)
c2 + ab
≤
ab + bc + ca
2
ineq
a2
+ b2
+ c2
+
ab + bc + ca
2
≥
a3
(b + c − a)
a2 + bc
+ a2
a2
+ b2
+ c2
+
ab + bc + ca
2
≥ (ab + bc + ca)(
a2
a2 + bc
)
a2
+ b2
+ c2
+ (ab + bc + ca)(
bc
a2 + bc
) ≥
5
2
(ab + bc + ca)
a2
+ b2
+ c2
ab + bc + ca
+
bc
a2 + bc
≥
5
2
…se two ineq
bc
a2 + bc
+
ab
c2 + ab
+
ac
b2 + ac
≥
4abc
(a + b)(b + c)(c + a)
+ 1(1)
it is e—sy to proveF
a2
+ b2
+ c2
ab + bc + ca
+
8abc
(a + b)(b + c)(c + a)
≥ 2(2)
ƒo e—sy to see th—t
a2
+ b2
+ c2
ab + bc + ca
+
bc
a2 + bc
≥
a2
+ b2
+ c2
ab + bc + ca
+
4abc
(a + b)(b + c)(c + a)
+ 1
≥
a2
+ b2
+ c2
2(ab + bc + ca)
+ 2 ≥
5
2
‡e h—ve done 3
a3
(b + c − a)
a2 + bc
+
b3
(c + a − b)
b2 + ca
+
c3
(a + b − c)
c2 + ab
≤
3abc(a + b + c)
2(ab + bc + ca)
Solution
a2
+ b2
+ c2
ab + bc + ca
+
bc
a2 + bc
+
3abc(a + b + c)
2(ab + bc + ca)
2 ≥ 3
end ‡e prove th—t
3abc(a + b + c)
2(ab + bc + ca)
2 ≥
4abc
(a + b)(b + c)(c + a)
3(a + b + c)(a + b)(b + c)(c + a) ≥ 8(ab + bc + ca)2
„his ineq is true ˜e™—use
3(a + b + c)(a + b)(b + c)(c + a) ≥
8
3
(a + b + c)2
(ab + bc + ca) ≥ 8(ab + bc + ca)2
PU
ƒo
LHS ≥
a2
+ b2
+ c2
ab + bc + ca
+
4abc
(a + b)(b + c)(c + a)
+
4abc
(a + b)(b + c)(c + a)
+ 1 ≥ 3
vet
a, b, c > 0
ƒhow th—t
a3
(b + c − a)
a2 + bc
+
b3
(c + a − b)
b2 + ca
+
c3
(a + b − c)
c2 + ab
≤
9abc
2(a + b + c)
pirstD‡e prove this lenm—X
a2
a2 + bc
+
b2
b2 + ca
+
c2
c2 + ab
≤
(a + b + c)2
2(ab + bc + ca)
bc
a2 + bc
+
ac
b2 + ac
+
ab
c2 + ab
+
a2
+ b2
+ c2
2(ab + bc + ca)
≥ 2
whi™h is true from
bc
a2 + bc
+
ac
b2 + ac
+
ab
c2 + ab
≥ 1 +
4abc
(a + b)(b + c)(c + a)
a2
+ b2
+ c2
2(ab + bc + ca)
+
4abc
(a + b)(b + c)(c + a)
≥ 1
equ—lity o™™ur if —nd if only
a = b = c
or
a = b, c = 0
or —ny ™y™li™ permutionF
‚eturn to your inequ—lityD‡e h—ve
(
a3
(b + c − a)
a2 + bc
+ a2
) ≤ a2
+ b2
+ c2
+
9abc
2(a + b + c)
or
(ab + bc + ca)
a2
a2 + bc
≤ a2
+ b2
+ c2
+
9abc
2(a + b + c)
prom
a2
a2 + bc
+
b2
b2 + ca
+
c2
c2 + ab
≤
(a + b + c)2
2(ab + bc + ca)
‡e only need to prove th—t
(a + b + c)2
2
≤ a2
+ b2
+ c2
+
9abc
2(a + b + c)
or
a2
+ b2
+ c2
+
9abc
a + b + c
≥ 2(ab + bc + ca)
‡hi™h is s™hur inequ—lityF yur Solution —re ™ompleted equ—lity o™™ur if —nd if only
a = b = c, a = b, c = 0
or —ny ™y™li™ permutionF
QQD vet
a, b, c > 0
PV
su™h th—t
a + b + c = 1
„hen
a3
+ bc
a2 + bc
+
b3
+ ca
b2 + ca
+
c3
+ ab
c2 + ab
≥ 2
prom the ™ondition
a − 1 = −(b + c)
it follows th—t
a3
+ bc
a2 + bc
= −
a2
(b + c)
a2 + bc
+ 1
„hus it suffi™es to prove th—t
a + b + c ≥
a2
(b + c)
a2 + bc
por
a, b, c
positive re—ls prove th—t
ab(a + b)
c2 + ab
≥ a
<=> a4
+ b4
+ c4
− b2
c2
− c2
a2
− a2
b2
≥ 0
ab(a + b)
c2 + ab
+
c2
(a + b)
c2 + ab
= 2 a
—nd our inequ—lity ˜e™omes
c2
(a + b)
(c2 + ab)
≤ a
˜ut
c2
(a + b)
(c2 + ab)
=
c2
(a + b)2
(c2 + ab)(a + b)
=
(ca + cb)2
a(b2 + c2) + b(a2 + c2)
≤
c2
a2
a(b2 + c2)
+
c2
b2
b(a2 + c2)
= a
QR
vet
a, b, c ≥ 0
su™h th—t
a + b + c = 1
„hen
6(a2
+ b2
+ c2
) ≥
a3
+ bc
a2 + bc
+
b3
+ ca
b2 + ca
+
c3
+ ab
c2 + ab
Solution
6(a2
+ b2
+ c2
) +
a2
(b + c)
a2 + bc
≥ 3
6(a2
+ b2
+ c2
) − 2(a + b + c)2
≥ (a−
a2
(b + c)
a2 + bc
)
4 (a − b)(a − c) ≥
a(a − b)(a − c)
a2 + bc
(a − b)(a − c)(4 −
a
a2 + bc
) ≥ 0
PW
essuming ‡vyq
a ≥ b ≥ c
then e—sy to see th—t
4 −
a
a2 + bc
≥ 0
—nd
4 −
c
c2 + ab
≥ 0
(c − a)(c − b)(4 −
c
c2 + ab
) ≥ 0and(a − b)(a − c)(4 −
a
a2 + bc
) ≥ 0
‡e h—ve two ™—ses
g—se I
4 −
b
b2 + ac
≤ 0
then
(b − c)(b − a)(4 −
b
b2 + ac
) ≥ 0
so this ineq is true
g—se P
4 −
b
b2 + ac
≤ 0
e—sy to see th—t
4 −
c
c2 + ab
≥ 4 −
b
b2 + ac
ƒo
LHS ≥ (c − b)2
(4 −
c
c2 + ab
) + (a − b)(b − c)(
b
b2 + ac
−
c
c2 + ab
) ≥ 0
FiFh
QSF vet
x, y, z
˜e re—l num˜ers s—tisfyX
x2
y2
+ 2yx2
+ 1 = 0
pind the m—ximum —nd minimum v—lues ofX
f(x, y) =
2
x2
+
1
x
+ y(y +
1
x
+ 2)
SolutionX €ut
t =
1
x
; k = y + 1
D ‡e h—veX
t2
+ k2
= 1
f(x, y) = t2
+ tk
€ut
t = cos α; k = sin α
then
f(x, y) = cos α2
+ cos α sinα =
sin 2α2 =
1
2
+
1
√
2
cos (2α −
π
4
)
max f(x, y) =
1
2
+
1
√
2
QH
min f(x, y) =
1
2
−
1
√
2
FiFh
F
QTF
ƒuppose —D˜D™Dd —re positive integers with ab + cd = 1.
„henD por We = 1, 2, 3, 4,let (xi)2
+ (yi)2
= 1, where xi —nd yi —re re—l num˜ersF
ƒhow th—t
(ay1 + by2 + cy3 + dy4)2
+ (ax4 + bx3 + cx2 + dx1)2
≤ 2(
a
b
+
b
a
+
c
d
+
d
c
).
ƒolitionX
…se g—u™hyEƒ™hw—rtz D ‡e h—ve
(ay1 + by2 + cy3 + dy4)2
≤
(ab + cd)(
(ay1 + by2)2
ab
+
(cy3 + dy4)2
cd
) =
(ay1 + by2)2
ab
+
(cy3 + dy4)2
cd
ƒimil—rX
(ax4 + bx3 + cx2 + dx1)2
≤
(ab + cd)(
(ax4 + bx3)2
ab
+
(cx2 + dx1)2
cd
)
=
(ax4 + bx3)2
ab
+
(cx2 + dx1)2
cd
futX
(ay1 + by2)2
≤ (ay1 + by2)2
+ (ax1 − bx2)2
= a2
+ b2
+ 2ab(y1y2 − x1x2)
ƒimil—rF
(cx2 + dx1)2
≤ c2
+ d2
+ 2cd(x1x2 − y1y2)
D then ‡e getX
(ay1 + by2)2
ab
+
(cx2 + dx1)2
cd
≤
a
b
+
b
a
+
c
d
+
d
c
@IA
„he s—me —rgument show th—tX
(cy3 + dy4)2
cd
+
(ax4 + bx3)2
ab
≤
a
b
+
b
a
+
c
d
+
d
c
@PA
gom˜ining @IAY@PA ‡e get F FiFh
QUF
in —ny ™onvex qu—dril—ter—l with sides
a ≤ b ≤ c ≤ d
QI
—nd —re— F
€rove th—t X
F ≤
3
√
3
4
c2
SolutionX
„he inequ—lity is rewritten —sX
(−a + b + c + d)(a − b + c + d)(a + b − c + d)(a + b + c − d) ≤ 27c4
.
‡e su˜stitute x = −a + b + c + dD y = a − b + c + dD z = a + b − c + dD t = a + b + c − dF
„hen
x + y − z + t
4
= c —nd x ≥ y ≥ z ≥ t.
„hus ‡e h—veX xyzt ≤ 27(
x + y − z + t
4
)4
.
„he left side of the inequ—lity is m—ximum when z = y
while the right side of the inequ—lity is minimum @‡e h—ve fixed xDy —nd tAF
„hen ‡e just prove th—t xy2
t ≤ 27(
x + t
4
)4
.
fe™—use xy2
t ≤ x3
tD ‡e just h—ve to prove
x3
t ≤ (
x + t
4
)4
end then it follows th—t the —˜ove inequ—lity is —lso trueF
x
3
+
x
3
+
x
3
+ t ≥
4 4
x
3
·
x
3
·
x
3
· t
hen™e
27(
x + y
4
)4
≥ x3
t
QVF
vet efg ˜e — tri—ngleF €rove th—tX
1
a
+
1
b
+
1
c
≤
1
a + b − c
+
1
b + c − a
+
1
c + a − b
SolutionX
IF
1
a + b − c
+
1
b + c − a
=
2b
(a + b − c)(b + c − a)
=
2b
b2 − (c − a)2
≥
2b
b2
=
2
b
ƒimil—rlyD ‡e h—ve
1
b + c − a
+
1
c + a − b
≥
2
c
1
c + a − b
+
1
a + b − c
≥
2
a
edd three inequ—lities together —nd divide ˜y P to get the desired resultF
PF
use u—r—m—t— for the num˜er —rr—ys (b + c − a; c + a − b; a + b − c) (a; b; c)
—nd the ™onvex fun™tion
f (x) =
1
x
QP
yr m—ke the su˜stitution x = 1
2 (b + c − a)D y = 1
2 (c + a − b)D
z = 1
2 (a + b − c) —nd get
a = y + z, b = z + x, c = x + y,
so th—t the inequ—lity in question ™—n ˜e rewritten —s
1
y + z
+
1
z + x
+
1
x + y
≤
1
2x
+
1
2y
+
1
2z
D
wh—t dire™tly follows from ewErwX
2
y + z
≤
1
2y
+
1
2z
,
2
z + x
≤
1
2z
+
1
2x
,
2
x + y
≤
1
2x
+
1
2y
QWF
vet a, b, c ˜e nonneg—tive re—l num˜ersF €rove th—t
a3
+ b3
+ c3
+ 3abc ≥
(a2
b + b2
c + c2
a)2
ab2 + bc2 + ca2
+
(ab2
+ bc2
+ ca)2
a2b + b2c + c2a
SolutionX
if a = 0 or b = 0 or c = 0 Dit9s trueFif
abc > 0
€ut x = a
b , y = b
c , z = c
a F‡e need prove
x
z
+
y
x
+
z
y
+ 3 ≥
(xy + yz + zx)2
xyz(x + y + z)
+
(x + y + z)2
xy + yz + zx
x
z
+
y
x
+
z
y
≥
x2
y2
+ y2
z2
+ z2
x2
xyz(x + y + z)
+
x2
+ y2
+ z2
xy + yz + zx
x2
z
+
z2
y
+
y2
x
≥
(x2
+ y2
+ z2
)(x + y + z)
xy + yz + zx
x3
y
z
+
y3
z
x
+
z3
x
y
≥ x2
y + y2
z + z2
x
fy using ew qw9inequ—lityD ‡e h—veX
x3
y
z
+ xyz ≥ 2x2
y,
y3
z
x
+ xyz ≥ 2y2
z
z3
x
y
+ xyz ≥ 2z2
y, x2
y + y2
z + z2
x ≥ 3xyz
‡e h—ve done
RHF
vet x, y, z ˜e positive re—l num˜ersF €rove th—tX
x +
1
y
− 1 y +
1
z
− 1 ≥ 3.
SolutionF
‡e rewrite the inequ—lity —s
y
x
+
1
xyz
− 2 x + 1 −
2
xyz
xy + 3 ≥ 0.
QQ
€utting xyz = k3
D then there exist a, b, c > 0 su™h th—t x = ka
b , y = kb
c , z = kc
a .
„he inequ—lity ˜e™omes
a2
bc
+
1
k2
− 2k
a
b
+ k2
−
2
k
b
a
+ 3 ≥ 0
f(k) = a3
+ k2
−
2
k
a2
b +
1
k2
− 2k ab2
+ 3abc ≥ 0
‡e h—ve th—t
f (k) =
2(k3
+ 1)
k3
k a2
b − ab2
f (k) = 0 ⇔ k =
ab2
a2b
.
prom nowD —™™ording to the †—ri—tion fo—rdD ‡e ™—n dedu™e our inequ—lity to show th—t
f
ab2
a2b
≥ 0
or equiv—lentlyD
a3
+ b3
+ c3
+ 3abc ≥
(a2
b + b2
c + c2
a)2
ab2 + bc2 + ca2
+
(ab2
+ bc2
+ ca2
)2
a2b + b2c + c2a
.
FiFh
RIF
qiven a, b, c ≥ 0F€rove th—tX
(a + b + c)2
2(ab + bc + ca)
≥
a2
a2 + bc
+
b2
b2 + ca
+
c2
c2 + ab
SolutionX
‡e h—ve
2a2
(a + b)(a + c)
−
a2
a2 + bc
=
a2
(a − b)(a − c)
(a + b)(a + c)(a2 + bc)
≥ 0
@e—sy to ™he™k ˜y †orni™u ƒ™hurA it suffi™es to prove th—t
(a + b + c)2
2(ab + bc + ca)
≥
2a2
(a + b)(a + c)
=
2 ab(a + b)
(a + b)(b + c)(c + a)
essume th—t a + b + c = 1 —nd put q = ab + bc + ca, r = abcD then the inequ—lity ˜e™omes
1
4q
≥
q − 3r
q − r
⇔
q − r
q − 3r
≥ 4q
⇔
2r
q − 3r
≥ 4q − 1
fy ƒ™hur9s inequ—lity for third degreeD ‡e h—ve r ≥ 4q−1
9 D then
2r
q − 3r
≥
2r
q − 4q−1
3
=
6r
1 − q
QR
it suffi™es to show th—t
6r ≥ (4q − 1)(1 − q)
fut this is just ƒ™hur9s inequ—lity for fourth degree
a4
+ abc a ≥ ab(a2
+ b2
)
‡e h—ve doneF
PF
ƒuppose a + b + c = 3F ‡e need to proveX
f(r) = 4q4
− 9q3
+ 24qr2
− 54q2
r − 72r2
− 243r + 216qr ≤ 0
f (r) = 48qr − 54q2
− 144r − 243 + 216q
f (r) = 48(q − 3) ≤ 0, sof (r) ≤ f (0) = −54q2
− 144 + 216q ≤ 0
ƒoD with q ≤ 9
4 , f(r) ≤ f(0) = q3
(4q − 9) ≤ 0
‡ith q ≥ 9
4 D ‡e h—veX f(r) ≤ f(4q−9
3 ) ≤ 0 @trues with q ≥ 9
4 A
RPF
vet a, b, c ˜e nonneg—tive re—l num˜ers su™h th—t a2
+ b2
+ c2
= 1F €rove th—t
a3
b2 − bc + c2
+
b3
+ c3
a2
≥
√
2
SolutionX
a3
b2 − bc + c2
+
b3
+ c3
a2
≥
(a2
+ b2
+ c2
)2
a[b2 − bc + c2 + a(b + c)]
≥
1
2.a[3−2a2
4 ]
=
1
2 a2(
3
2 −a2
2 )2
≥
√
2
RQF
vet ∆ABC —nd max(A, B, C) ≤ 90F €rove th—t X
cosAcosB
sin2C
+
cosBcosC
sin2A
+
cosCcosA
sin2B
≥
√
3
2
SolutionX
fut if A = 90◦
the left side does not existF
if max{A, B, C} < 90◦
F vet a2
+ b2
− c2
= z, a2
+ c2
− b2
= y —nd b2
+ c2
− a2
= x.
ren™eD x, y —nd z —re positive —nd
cyc
cos A cos B
sin 2C
=
cyc
b2
+c2
−a2
2bc · a2
+c2
−b2
2ac
2 · 2S
ab · a2+b2−c2
2ab
=
=
cyc
ab(b2
+ c2
− a2
)(a2
+ c2
− b2
)
8c2S(a2 + b2 − c2)
=
cyc
xy
(x + y)z
a2b2
2(a2b2 + a2c2 + b2c2) − a4 − b4 − c4
=
=
cyc
xy
(x + y)z
(x + z)(y + z)
4(xy + xz + yz)
.
„husD it rem—ins to prove th—t
cyc
xy
(x + y)z
(x + z)(y + z)
xy + xz + yz
≥
√
3,
QS
whi™h is equiv—lent to
cyc
x2
y2
(x + z)(y + z)
x + y
≥ xyz 3(xy + xz + yz).
fy g—u™hyEƒ™hw—rtz ‡e o˜t—inX
cyc
x2
y2
(x + z)(y + z)
x + y
·
cyc
x + y
(x + z)(y + z)
≥ (xy + xz + yz)2
.
ren™eD ‡e need to prove th—t
(xy + xz + yz)2
≥
cyc
x + y
(x + z)(y + z)
· xyz 3(xy + xz + yz).
‡e o˜t—inX
(xy + xz + yz)2
≥
cyc
x + y
(x + z)(y + z)
· xyz 3(xy + xz + yz) ⇔
⇔
√
xy + xz + yz
3
(x + y)(x + z)(y + z) ≥
cyc
xyz(x + y) 3(x + y) ⇔
⇔ (xy + xz + yz)3
(x + y)(x + z)(y + z) ≥ 3x2
y2
z2
cyc
(x + y)3
+
+6x2
y2
z2
cyc
(x + y)(x + z) (x + y)(x + z) ⇔
⇔ (xy + xz + yz)3
(x + y)(x + z)(y + z) ≥ 3x2
y2
z2
cyc
(2x3
+ 3x2
y + 3x2
z)+
+3x2
yz
cyc
(x + y)(x + z)2 z2(x + y)y2(x + z).
fy ewEqw 2 z2(x + y)y2(x + z) ≤ y2
x + y2
z + z2
x + z2
y.
ren™eD it rem—ins to prove th—t
⇔ (xy + xz + yz)3
(x + y)(x + z)(y + z) ≥ 3x2
y2
z2
cyc
(2x3
+ 3x2
y + 3x2
z)+
+3x2
yz cyc(x + y)(x + z)(y2
x + y2
z + z2
x + z2
y), whi™h is equiv—lent to
sym(x5
y4
+ x5
y3
z − 5x4
y3
z2
+ x4
y4
z + 2x3
y3
z3
) ≥ 0, whi™h is true ˜y ewEqw ˜e™—use
sym
(x5
y4
+ x5
y3
z − 5x4
y3
z2
+ x4
y4
z + 2x3
y3
z3
) ≥ 0 ⇔
⇔
sym
(x5
y4
+ x5
y3
z +
1
3
x4
y4
z +
2
3
x4
z4
y + 2x3
y3
z3
) ≥
sym
5x4
y3
z2
.
FiFh
RRF
vet a, b, c ˜e nonneg—tive re—l num˜ersD no two of whi™h —re zeroF €rove th—t
a
b + c
+
b
c + a
+
c
a + b
+
a2
b + b2
c + c2
a
ab2 + bc2 + ca2
≥
5
2
QT
Solution
IFFF—sume p = 1 —nd vemm— ab2
+ bc2
+ ca2
≤ 4
27 − r
⇔
a
b + c
+
b
c + a
+
c
a + b
+
ab(a + b)
ab2 + bc2 + ca2
≥
7
2
‡e h—ve
a
b + c
+
b
c + a
+
c
a + b
+
ab(a + b)
ab2 + bc2 + ca2
≥
1 − 2q + 3r
q − r
+
27q − 81r
4 − 27r
‡e need prove th—t
1 − 2q + 3r
q − r
+
27q − 81r
4 − 27r
≥
7
2
⇔
1 + r
q − r
− 2 +
27q − 12
4 − 27r
+ 3 ≥
7
2
⇔
1 + r
q − r
+
27q − 12
4 − 27r
≥
5
2
⇔ −135r2
+ r(81q + 2) + (54q2
+ 8 − 44q) ≥ 0
f(r) = −135r2
+ r(81q + 2) + (54q2
+ 8 − 44q)
f (r) = −270r + 81q + 2 ≥ 0 @˜e™—use q ≥ 9rA
⇒ f(r) ≥ f(
4q − 1
9
) =
570q2
− 349q + 55
9
≥ 0
PFFFFFFFF
a
b + c
+
b
c + a
+
c
a + b
+
a2
b + b2
c + c2
a
ab2 + bc2 + ca2
≥
5
2
⇔
⇔
cyc
a
b + c
−
1
2
≥
(a2
c − a2
b)
a2c
⇔
⇔
cyc
a − b − (c − a)
2(b + c)
≥
(a − b)(b − c)(c − a)
a2c + b2a + c2b
⇔
⇔
cyc
a − b
2
1
b + c
−
1
a + c
≥
(a − b)(b − c)(c − a)
a2c + b2a + c2b
⇔
⇔
cyc
(a − b)2
(a + c)(b + c)
≥
2(a − b)(b − c)(c − a)
a2c + b2a + c2b
.
if (a − b)(b − c)(c − a) ≤ 0 then the inequ—lity holdsF
vet (a − b)(b − c)(c − a) > 0 —nd a2
c+b2
a+c2
b
a2b+b2c+c2a = t. „hen t > 1.
fy ewEqw ‡e o˜t—inX
cyc
(a − b)2
(a + c)(b + c)
≥ 3 3 (a − b)2(b − c)2(c − a)2
(a + b)2(a + c)2(b + c)2
.
„husD it rem—ins to prove th—t
27(a2
c + b2
a + c2
b)3
≥ 8(a + b)2
(a + c)2
(b + c)2
(a − b)(b − c)(c − a).
QU
fut
(a + b)(a + c)(b + c) =
cyc
(a2
b + a2
c) + 2abc ≤
4
3 cyc
(a2
b + a2
c).
id estD it rem—ins to prove th—t 27t3
≥ 8 · 16
9 (t + 1)2
(t − 1), whi™h o˜viousF
RSF
por —ll nonneg—tive re—l num˜ers a, b —nd cD no two of whi™h —re zeroD
1
(a + b)2
+
1
(b + c)2
+
1
(c + a)2
≥
3 3abc(a + b + c)(a + b + c)2
4(ab + bc + ca)3
Solution ‚epl—™ing a, b, c ˜y 1
a , 1
b , 1
c respe™tivelyD ‡e h—ve to prove th—t
a2
b2
(a + b)2
≥
3 3(ab + bc + ca)(ab + bc + ca)2
4(a + b + c)3
.
xowD using g—u™hy ƒ™hw—rz inequ—lityD ‡e h—ve
a2
b2
(a + b)2
≥
(ab + bc + ca)2
(a + b)2 + (b + c)2 + (c + a)2
=
(ab + bc + ca)2
2(a2 + b2 + c2 + ab + bc + ca)
.
it suffi™es to prove th—t
(ab + bc + ca)2
2(a2 + b2 + c2 + ab + bc + ca)
≥
3 3(ab + bc + ca)(ab + bc + ca)2
4(a + b + c)3
or equiv—lentlyD
2(a + b + c)3
≥ 3 3(ab + bc + ca)(a2
+ b2
+ c2
+ ab + bc + ca),
th—t is
4(a + b + c)6
≥ 27(ab + bc + ca)(a2
+ b2
+ c2
+ ab + bc + ca)2
fy ewEqwD ‡e see th—t
27(ab + bc + ca)(a2
+ b2
+ c2
+ ab + bc + ca)2
≤
1
2
2(ab + bc + ca) + (a2
+ b2
+ c2
+ ab + bc + ca) + (a2
+ b2
+ c2
+ ab + bc + ca)
3
= 4(a+b+c)6
.
„hereforeD our Solution is ™ompleted
RTF
2
3
(
1
a2 + bc
+
1
b2 + ca
+
1
c2 + ab
) ≥
1
ab + bc + ca
+
2
a2 + b2 + c2
SolutionX
‚ewrite our inequ—lity —sX
1
a2 + bc
≥
3(a + b + c)2
2(a2 + b2 + c2)(ab + bc + ca)
.
‡e will ™onsider P ™—sesX g—se IF a2
+b2
+c2
≤ 2(ab+bc+ca), then —pplying g—u™hy ƒ™hw—rz
inequ—lityD ‡e ™—n redu™e our inequ—lity to
6
a2 + b2 + c2 + ab + bc + ca
≥
(a + b + c)2
(a2 + b2 + c2)(ab + bc + ca)
,
(a2
+ b2
+ c2
− ab − bc − ca)(2ab + 2bc + 2ca − a2
− b2
− c2
) ≥ 0, whi™h is trueF
g—se PF a2
+ b2
+ c2
≥ 2(ab + bc + ca), then (a + b + c)2
≤ 2(a2
+ b2
+ c2
), whi™h yields th—t
3(a + b + c)2
2(a2 + b2 + c2)(ab + bc + ca)
≤
3
ab + bc + ca
,
QV
—nd ‡e just need to prove th—t
1
a2 + bc
+
1
b2 + ca
+
1
c2 + ab
≥
3
ab + bc + ca
,
whi™h is just your very known @—nd ni™eA inequ—lityF
RUF
vet a, b, c ˜e nonneg—tive re—l num˜ersD no two of whi™h —re zeroF €rove th—t
(a)
1
2a2 + bc
+
1
2b2 + ca
+
1
2c2 + ab
≥
1
ab + bc + ca
+
2
a2 + b2 + c2
SolutionX
Ist Solution
fy g—u™hy inequ—lityD
cyc
(b + c)2
(2a2
+ bc)
cyc
1
2a2 + bc
≥ 4(a + b + c)2
it rem—ins to show th—t
cyc
(b + c)2
(2a2
+ bc) ≤ 4(a2
+ b2
+ c2
)(ab + bc + ca)
whi™h is e—syF Pnd SolutionF
ƒin™e
cyc
ab + bc + ca
2a2 + bc
=
cyc
bc
2a2 + bc
+
cyc
a(b + c)
2a2 + bc
‡e need to show
cyc
bc
2a2 + bc
≥ 1
—nd
cyc
a(b + c)
2a2 + bc
≥
2(ab + bc + ca)
a2 + b2 + c2
„he former is illEknownX if x, y, z ≥ 0 su™h th—t xyz = 1D then
1
2x + 1
+
1
2y + 1
+
1
2z + 1
≥ 1
„he l—terX ˜y g—u™hy inequ—lityD
cyc
a(b + c)(2a2
+ bc)
cyc
a(b + c)
2a2 + bc
≥ 4(ab + bc + ca)2
„he result then follows from the following identity
cyc
a(b + c)(2a2
+ bc) = 2(ab + bc + ca)(a2
+ b2
+ c2
)
Qrd SolutionF
LHS − RHS
a + b + c
=
2(a + b + c)(a − b)2
(b − c)2
(c − a)2
+ 3abc cyc(a2
+ ab + b2
)(a − b)2
(2a2 + bc)(2b2 + ca)(2c2 + ab)(ab + bc + ca)(a2 + b2 + c2)
Qrd SolutionF
essume th—t c = min{a, b, c}D then the g—u™hy ƒ™hw—rz inequ—lity yields
1
2a2 + bc
+
1
2b2 + ca
≥
4
2(a2 + b2) + c(a + b)
,
QW
then ‡e just need to prove th—t
4
2(a2 + b2) + c(a + b)
+
1
ab + 2c2
≥
1
ab + bc + ca
+
2
a2 + b2 + c2
,
or equiv—lently
c(a + b − 2c)
(ab + 2c2)(ab + bc + ca)
≥
2c(a + b − 2c)
(a2 + b2 + c2)(2a2 + 2b2 + ac + bc)
,
th—t is
(a2
+ b2
+ c2
)(2a2
+ 2b2
+ ac + bc) ≥ 2(ab + 2c2
)(ab + bc + ca),
whi™h is true sin™e a2
+ b2
+ c2
≥ ab + bc + ca —nd 2a2
+ 2b2
+ ac + bc ≥ 2(ab + 2c2
). honeF
RVF
vet a, b, c ˜e positive re—l num˜er F €rove th—tX
(c) 2(
1
a2 + 8bc
+
1
b2 + 8ca
+
1
c2 + 8ab
) ≥
1
ab + bc + ca
+
1
a2 + b2 + c2
SolutionX
‚epl—™ing a, b, c ˜y 1
a , 1
b , 1
c respe™tivelyD ‡e ™—n rewrite our inequ—lity —s
4(a + b + c)
a
8a2 + bc
+
b
8b2 + ca
+
c
8c2 + ab
≥ 2 +
2abc(a + b + c)
a2b2 + b2c2 + c2a2
.
xowD —ssume th—t c = mina, b, cD then ‡e h—ve the following estim—tionsX
a(4a + 4b + c)
8a2 + bc
+
b(4a + 4b + c)
8b2 + ca
− 2 =
(a − b)2
(32ab − 12ac − 12bc + c2
)
(8a2 + bc)(8b2 + ca)
≥ 0,
—nd
2abc(a + b + c)
a2b2 + b2c2 + c2a2
≤
2c(a + b + c)
ab + 2c2
.
‡ith these estim—tionsD ‡e ™—n redu™e our inequ—lity to
3ac
8a2 + bc
+
3bc
8b2 + ca
+
4c(a + b + c)
8c2 + ab
≥
2c(a + b + c)
ab + 2c2
,
or
3a
8a2 + bc
+
3b
8b2 + ca
≥
2(a + b + c)(4c2
− ab)
(ab + 2c2)(ab + 8c2)
.
e™™ording to g—u™hy ƒ™hw—rz inequ—lityD ‡e h—ve
3a
8a2 + bc
+
3b
8b2 + ca
≥
12
8(a + b) + c a
b + b
a
.
it suffi™es to show th—t
6
8(a + b) + c a
b + b
a
≥
(a + b + c)(4c2
− ab)
(ab + 2c2)(ab + 8c2)
.
if 4c2
≤ abD then it is trivi—lF ytherwiseD ‡e h—ve
a + b ≤ c + ab
c , —nd a
b + b
a ≤ ab
c2 + c2
ab . ‡e need to prove
6
8 c + ab
c + c ab
c2 + c2
ab
≥
2c + ab
c (4c2
− ab)
(ab + 2c2)(ab + 8c2)
,
RH
whi™h isD —fter exp—ndingD equiv—lent to
(9ab − 4c2
)(ab − c2
)2
c(ab + 8c2)(c4 + 8abc2 + 9a2b2)
≥ 0,
whi™h is true —s c = mina, b, c.
yur Solution is ™ompletedF
RWF
vet a, b, c > 0F€rove th—tX
5
3
(
1
4a2 + bc
+
1
4b2 + ca
+
1
4c2 + ab
) ≥
2
ab + bc + ca
+
1
a2 + b2 + c2
SolutionX
essume th—t c = min(a, b, c)D then ‡e h—ve the following estim—tionsX
1
4a2 + bc
+
1
4b2 + ca
−
4
8ab + ac + bc
=
(a − b)2
(32ab − 12ac − 12bc + c2
)
(4a2 + bc)(4b2 + ca)(8ab + ac + bc)
≥ 0,
—nd
1
a2 + b2 + c2
≤
1
2ab + c2
.
…sing theseD ‡e ™—n redu™e our inequ—lity to
20
8ab + ac + bc
+
5
ab + 4c2
≥
6
ab + ac + bc
+
3
2ab + c2
.
henote x = a + b ≥ 2
√
ab then this inequ—lity ™—n ˜e rewritten —s
f(x) =
20
cx + 8ab
−
6
cx + ab
+
5
ab + 4c2
−
3
2ab + c2
≥ 0.
‡e h—ve
f (x) =
6c
(cx + ab)2
−
20
(cx + 8ab)2
≥
20c
(cx + ab)(cx + 8ab)
−
20c
(cx + 8ab)2
=
140abc
(cx + ab)(cx + 8ab)2
≥ 0.
„his shows th—t f(x) is in™re—singD —nd ‡e just need to prove th—t f(2
√
ab) ≥ 0, whi™h is
equiv—lent to
7c(13t2
+ 6tc + 8c2
)(t − c)2
t(t + 2c)(4t + c)(2t2 + c2)(t2 + 4c2)
≥ 0,
‡here
t =
√
ab
„his is o˜viously nonneg—tiveD so our Solution is ™ompletedF
SHF
vet a, b —nd c re—l num˜ers su™h th—t a + b + c + d = e = 0F €rove th—tX
30(a4
+ b4
+ c4
+ d4
+ e4
) ≥ 7(a2
+ b2
+ c2
)2
SolutionX
xoti™e th—t there exitst three num˜ers —mong a, b, c, d, e h—vinh the s—me singF vet these
num˜er ˜e a, b, c, d, e F‡ithout loss of gener—lityD‡e m—y —ssume th—t a, b, c ≥ 0@it notD‡e
™—n t—ke −1, −b, −cAF
xow Dusing the g—u™hyEƒ™h—wrz inequ—lityD‡e h—veX
[(9(a4
+b4
+c4
)+2(d4
+e4
))+7d4
+7e4
)](84+63+63) ≥ [2 21(9(a4 + b4 + c4) + 2(d4 + e4))+21d2
+21e2
]2
.
RI
end thusDit suffi™es to prove th—tX
2 9(a4 + b4 + c4) + 2(d4 + e4)) ≥
√
21(a2
+ b2
+ c2
).
yr
36(a4
+ b4
+ c4
) + 8(d4
+ e4
) ≥ 21(a2
+ b2
+ c2
)2
.
ƒin™e
d4
+ e4
≥
(d2
+ e2
)2
2
≥
(d + e)4
8
=
(a + b + c)4
8
,
it is enough to show th—t
36(a4
+ b+
c4
) + (a + b + c + d)4
≥ 21(a2
+ b2
+ c2
)2
‡hi™h is true —nd it is e—sy to proveF
SIF
vet a, b, c > 0F€rove th—tX
a(b + c)
a2 + bc
+
b(c + a)
b2 + ca
+
c(a + b)
c2 + ab
≤
1
2
27 + (a + b + c)
1
a
+
1
b
+
1
c
Solution
„he inequ—lity is equiv—lent to
a2
(b + c)2
(a2 + bc)2
+ 2
ab(b + c)(c + a)
(a2 + bc)(b2 + ca)
≤
15
2
+
1
4
b + c
a
xoti™e th—t
(a2
+ bc)(b2
+ ca) − ab(b + c)(c + a) = c(a + b)(a − b)2
then
2
ab(b + c)(c + a)
(a2 + bc)(b2 + ca)
≤ 6
@IA yther h—ndD
a2
(b + c)2
(a2 + bc)2
≤
a2
(b + c)2
4a2bc
=
1
4
b
c
+
c
b
+ 2
@PA prom @IA —nd @PA ‡e h—ve done3
fesidesD ˜y the s—m w—ysD ‡e h—ve — ni™e Solution for —n old pro˜lemX
a(b + c)
a2 + bc
≤
√
a
1
√
a
SPF
por —ny positive re—l num˜ers a, b —nd cD
a(b + c)
a2 + bc
+
b(c + a)
b2 + ca
+
c(a + b)
c2 + ab
≤
√
a +
√
b +
√
c
1
√
a
+
1
√
b
+
1
√
c
SolutionX
‡e h—ve the inequ—lity is equiv—lent to
a(b + c)
a2 + bc
2
≤
√
a
1
√
a
RP
<=>
a(b + c)
a2 + bc
+ 2
ab(a + c)(b + c)
(a2 + bc)(b2 + ca)
≤
√
a
1
√
a
‡e ™—n e—sily prove th—t
ab(a + c)(b + c)
(a2 + bc)(b2 + ca)
≤ 3
ƒoD it suffi™es to prove th—t
<=>
a(b + c)
a2 + bc
+ 6 ≤
√
a
1
√
a
„o prove this ineqD ‡e only need to prove th—t
a + b
√
ab
−
c(a + b)
c2 + ab
− 1 ≥ 0
fut this is trivi—lD ˜e™—use
a + b
√
ab
−
c(a + b)
c2 + ab
− 1 = (a + b)
1
√
ab
−
c
c2 + ab
− 1
≥ 2
√
ab
1
√
ab
−
c
c2 + ab
− 1 =
c −
√
ab
2
c2 + ab
≥ 0
‡e —re doneF
SQF
vet a, b, c > 0F €rove th—tX
(a + b + c)3
3abc
+
ab2
+ bc2
+ ca2
a3 + b3 + c3
≥ 10
Solution
a3
+ b3
+ c3
3abc
+
ab2
+ bc2
+ ca2
a3 + b3 + c3
+
(a + b)(b + c)(c + a)
abc
≥ 10
…sing ewEqw9s inequ—lity D‡e h—veX
a3
+ b3
+ c3
3abc
+
ab2
+ bc2
+ ca2
a3 + b3 + c3
≥ 2
ab2 + bc2 + ca2
3abc
≥ 2
(a + b)(b + c)(c + a)
abc
≥ 8
SRF
in —ny tri—ngle efg show th—t
ama + bmb + cmc ≤
√
bcma +
√
camb +
√
abmc
SolutionX
‡e h—ve to prove the inequ—lity
ama + bmb + cmc ≤
√
bcma +
√
camb +
√
abmc
D where maD mbD mc —re the medi—ns of — tri—ngle efgF
ƒin™e 2bc
b+c ≤
√
bcD 2ca
c+a ≤
√
ca —nd 2ab
a+b ≤
√
ab
RQ
˜y the rwEqw inequ—lityD it will ˜e enough to show the stronger inequ—lity
ama + bmb + cmc ≤
2bc
b + c
ma +
2ca
c + a
mb +
2ab
a + b
mc
D
sin™e then ‡e will h—ve
ama + bmb + cmc ≤
2bc
b + c
ma +
2ca
c + a
mb +
2ab
a + b
mc
≤
√
bcma +
√
camb +
√
abmc
—nd the initi—l inequ—lity will ˜e provenF
ƒo in the followingD ‡e will ™on™entr—te on proving this stronger inequ—lityF
fe™—use the inequ—lity ‡e h—ve to prove is symmetri™D ‡e ™—n ‡vyq —ssume th—t a ≥ b ≥ cF
„henD ™le—rlyD bc ≤ ca ≤ abF
yn the other h—ndD using the formul—s m2
a = 1
4 2b2
+ 2c2
− a2
—nd m2
b = 1
4 2c2
+ 2a2
− b2
D
‡e ™—n get —s — result of — str—ightforw—rd ™omput—tionF
ma
b + c
2
−
mb
c + a
2
=
3ac + 3bc + a2
+ b2
+ 4c2
(a + b − c) (b − a)
4 (b + c)
2
(c + a)
2
xowD the fr—™tion on the right h—nd side is ≤ 0D sin™e 3ac + 3bc + a2
+ b2
+ 4c2
≥ 0 @this
is trivi—lAD
a + b − c > 0 @in f—™tD a + b > c ˜e™—use of the tri—ngle inequ—lityA —nd b − a ≤ 0 @sin™e
a ≥ bAF ren™eD
ma
b + c
2
−
mb
c + a
2
≤ 0
wh—t yields ma
b+c
2
≤ mb
c+a
2
—nd thus ma
b+c ≤ mb
c+a F ƒimil—rlyD using b ≥ cD ‡e ™—n find
mb
c+a ≤ mc
a+b F
„husD ‡e h—ve
ma
b + c
≤
mb
c + a
≤
mc
a + b
ƒin™e ‡e h—ve —lso bc ≤ ca ≤ abD the sequen™es
ma
b + c
;
mb
c + a
;
mc
a + b
—nd (bc; ca; ab) —re equ—lly sortedF „husD the ‚e—rr—ngement inequ—lity yields
ma
b + c
· bc +
mb
c + a
· ca +
mc
a + b
· ab ≥
ma
b + c
· ca +
mb
c + a
· ab +
mc
a + b
· bc
—nd
ma
b + c
· bc +
mb
c + a
· ca +
mc
a + b
· ab ≥
ma
b + c
· ab +
mb
c + a
· bc +
mc
a + b
· ca
ƒumming up these two inequ—litiesD ‡e get
2
ma
b + c
· bc + 2
mb
c + a
· ca + 2
mc
a + b
· ab
RR
≥
ma
b + c
· (ca + ab) +
mb
c + a
· (ab + bc) +
mc
a + b
· (bc + ca)
„his simplifies to
2bc
b + c
ma +
2ca
c + a
mb +
2ab
a + b
mc
≥
ma
b + c
· a (b + c) +
mb
c + a
· b (c + a) +
mc
a + b
· c (a + b)
iF eF to
2bc
b + c
ma +
2ca
c + a
mb +
2ab
a + b
mc ≥ ama + bmb + cmc
„husD ‡e h—ve
ama + bmb + cmc ≤
2bc
b + c
ma +
2ca
c + a
mb +
2ab
a + b
mc
—nd the Solution is ™ompleteF xote th—t in e—™h of the inequ—lities
ama + bmb + cmc ≤
√
bcma +
√
camb +
√
abmc
—nd
ama + bmb + cmc ≤
2bc
b + c
ma +
2ca
c + a
mb +
2ab
a + b
mc
equ—lity holds only if the tri—ngle efg is equil—ter—lF
SSF
por aD bD c positive re—ls prove th—t
a2
+ 3 b2
+ 3 c2
+ 3 ≥
4
3
3 √
abc (ab + bc + ca)
SolutionX
hivide abc for ˜oth term —nd t—ke x = bc
a ; y = ac
b ; z = ab
c —nd ‡e must prove th—tX
(xy + 3
xy ) ≥ (4
3 )3
(x + y + z) xote th—tX
LHS ≥ 3(x2
+y2
+z2
)+x2
y2
z2
+
4
x2y2z2
≥ (x+y+z)2
+4 ≥ 4(x+y+z) ≥ (
4
3
)3
(x+y+z).
STF veta, b, c > 0 F€rove th—tX
a + b
c
√
a2 + b2
+
b + c
a
√
b2 + c2
+
c + a
b
√
c2 + a2
≥
3
√
6
√
a2 + b2 + c2
.
Solution
IFFFeltern—tivelyD using ghe˜yshev —nd g—u™hyD
cycl
a + b
c
√
a2 + b2
≥
2(a + b + c)
3
·
9
cycl c
√
a2 + b2
=
6(a + b + c)
cycl c
√
a2 + b2
—nd
cycl
c a2 + b2 ≤
a + b + c
3
cycl
a2 + b2 ≤
a + b + c
3
6(a2 + b2 + c2)
gom˜ining ‡e get the desired resultF
SUF
vet a, b, c > 0 su™h th—t a2
+ b2
+ c2
+ abc = 4
€rove th—t
a2
b2
+ b2
c2
+ c2
a2
≤ a2
+ b2
+ c2
RS
SolutionX
vet a = 2 yz
(x+y)(x+z) , b = 2 xz
(x+y)(y+z) —nd c = 2 xy
(x+z)(y+z) ,
where x, y —nd z —re positive num˜ers @ e—sy to ™he™k th—t it exists AF
„husD it rem—ins to prove th—t
cyc
xy
(x + z)(y + z)
≥
cyc
4x2
yz
(x + y)(x + z)(y + z)2
,
whi™h equiv—lent to cyc(x4
y2
+x4
z2
−2x4
yz+2x3
y3
−2x2
y2
z2
) ≥ 0, whi™h true ˜y ewEqwF
SVF
vet a, b, c > 0 su™h th—t a + b + c = 1F€rove th—t
b2
a + b2
+
c2
b + c2
+
a2
c + a2
≥
3
4
Solution
‡e h—ve
b2
a + b2
+
c2
b + c2
+
a2
c + a2
≥
a2
+ b2
+ c2 2
(a4 + b4 + c4) + (ab2 + bc2 + ca2)
ren™e it suffi™es to prove th—t
a2
+ b2
+ c2 2
(a4 + b4 + c4) + (ab2 + bc2 + ca2)
≥
3
4
⇔ 4 a2
2
≥ 3 ab2
a + 3 a4
⇔ 4 a4
+ 8 a2
b2
≥ 3 a4
+ 3 a2
b2
+ abc2
+ a3
c
⇔ a4
+ 5 a2
b2
≥ 3abc a + 3 a3
c
ƒin™e ‡e —lw—ys h—ve
3 a3
c + b3
a + c3
b ≤ a2
+ b2
+ c2 2
= a4
+ b4
+ c4
+ 2 a2
b2
+ b2
c2
+ c2
a2
„herefor it suffi™es to prove th—t
3 a2
b2
+ b2
c2
+ c2
a2
≥ 3abc (a + b + c)
whi™h o˜viously trueF
SWF
vet a; b; c > 0F €rove th—t
ab+c
+ ba+c
+ ca+b
≥ 1
Solution
if a ≥ 1 or b ≥ 1 or c ≥ 1 then the inequ—lity is true
if 0 ≤ a, b, c ≤ 1 then suppose c = mina, b, c
C if a + b < 1 ‡e h—ve b + c < 1 Dc + a < 1
RT
epply fernoull‡e 9 inequ—lity
(
1
a
)b+c
) = (1 +
1 − a
a
)b+c
< 1 +
(b + c)(1 − a)
a
<
a + b + c
a
„herefore ab+c
> a
a+b+c
ƒimil—r for bc+a
—nd ca+b
dedu™e ab+c
+ ba+c
+ ca+b
> 1
C if a + b > 1 then ab+c
+ ba+c
+ ca+b
> ab+c
+ ba+c
≥ aa+b
+ ba+b
epply fernoull‡e 9 inequ—lity ‡e h—ve X haa+b
= (1 + (a − 1))a+b
> 1 + (a + b)(a − 1)
ƒimil—r forba+b
when™e ab+c
+ ba+c
+ ca+b
> 2 + (a + b)(a + b − 2) = (a + b − 1)2
+ 1 ≥ 1
THF
vet a, b, c ˜e the sidelengths of tri—ngle with perimeter 2 (⇒ a + b + c = 2). €rove th—t
a3
b
+
b3
c
+
c3
a
−
a3
c
−
b3
a
−
c3
b
< 3
SolutionX
„his ineq is equiv—lent toX
|a4
c + c4
b + b4
a − a4
b − bc
− c4
a| ≤ 3abc
<=> |(a − b)(b − c)(c − a)(a2
+ b2
+ c2
+ ab + bc + ca)| ≤ 3abc
fy ‚—v‡e ƒu˜stitution D denoteX a = x + y, b = y + z, c = z + xD so x + y + z = 1D this ineq
˜e™omesX
|(x − y)(y − z)(z − x)(3(x2
+ y2
+ z2
) + 5(xy + yz + zx)| ≤ 3(x + y)(y + z)(z + x)
i—sy to see th—t |(x − y)(y − z)(z − x)| ≤ (x + y)(y + z)(z + x)
ƒo ‡e need to prove (3(x2
+ y2
+ z2
) + 5(xy + yz + zx) ≤ 3 = 3(x + y + z)2
<=> xy + yz + zx ≥ 0
whi™h is o˜vious true
FiFh
F
TIF
qiven xDyDzbHF€rove th—t
x(y + z)2
2x + y + z
+
y(x + z)2
x + 2y + z
+
z(x + y)2
x + y + 2z
= (3xyz(x + y + z))
SolutionX
cyc
x(y + z)2
2x + y + z
− 3xyz(x + y + z) =
=
cyc
x(y + z)2
2x + y + z
− yz + xy + xz + yz − 3xyz(x + y + z) =
=
cyc
z2
(x − y) − y2
(z − x)
2x + y + z
+
cyc
z2
(x − y)2
2 xy + xz + yz + 3xyz(x + y + z)
=
RU
=
cyc
(x − y)
z2
2x + y + z
−
z2
2y + x + z
+
+
cyc
z2
(x − y)2
2 xy + xz + yz + 3xyz(x + y + z)
=
=
cyc
(x − y)2

 z2
2 xy + xz + yz + 3xyz(x + y + z)
−
z2
(2x + y + z)(2y + x + z)

 .
„husD it rem—ins to prove th—t
(2x + y + z)(2y + x + z) ≥ 2 xy + xz + yz + 3xyz(x + y + z) .
fut
(2x + y + z)(2y + x + z) ≥ 2 xy + xz + yz + 3xyz(x + y + z) ⇔
⇔ 2x2
+ 2y2
+ z2
+ 3xy + xz + yz ≥ 2 3xyz(x + y + z),
whi™h is true ˜e™—use
x2
+ y2
+ z2
≥ xy + xz + yz ≥ 3xyz(x + y + z).
it seems th—t the following inequ—lity is true tooF
vet x, y —nd z —re positive num˜ersF €rove th—tX
x(y + z)2
3x + 2y + 2z
+
y(x + z)2
2x + 3y + 2z
+
z(x + y)2
2x + 2y + 3z
≥
4
7
3xyz(x + y + z)
TPF
vet a, b ∈ R su™h th—t 9a2
+ 8ab + 7b2
≤ 6 €rove th—t X7a + 5b + 12ab ≤ 9
SolutionX
IFFF
fy ewEqw inequ—lityD ‡e see th—t
7a + 5b + 12ab ≤ 7 a2
+
1
4
+ 5 b2
+
1
4
+ 12ab
= (9a2
+ 8ab + 7b2
) − 2(a − b)2
+ 3 ≤ (9a2
+ 8ab + 7b2
) + 3 ≤ 6 + 3 = 9.
iqu—lity holds if —nd only if a = b = 1
2 .
TQF
vet x, y —nd z —re positive num˜ers su™h th—t x + y + z = 1
x + 1
y + 1
z .
€rove th—t xyz + yz + zx + xy ≥ 4.
SolutionX
xyz(x + y + z)
yz + zx + xy
+ x + y + z −
4xyz(x + y + z)2
(yz + zx + xy)2
≥
3xyz
yz + zx + xy
+ x + y + z −
4xyz(x + y + z)2
(yz + zx + xy)2
=
x3
(y − z)2
+ y3
(z − x)2
+ z3
(x − y)2
(yz + zx + xy)2
≥ 0 =⇒
1 +
1
x
+
1
y
+
1
z
≥
4
xyz
⇐⇒ xyz + yz + zx + xy ≥ 4
RV
TRF
vet a, b, ca0 s—tisfy a + b + c = 1
€rove th—t (a2
+ b2
)(b2
+ c2
)(c2
+ a2
)a 1
32
SolutionX
let f(a, b, c) = (a2
+ b2
)(b2
+ c2
)(c2
+ a2
)
let c = max(a, b, c);
‡e h—ve
f(a, b, c) ≤ f(a + b, 0, c)@whi™h is equiv—lent ab(−4abc2
+ a3
b + ab3
− 4a2
c2
− 4b2
c2
− 2c4
) ≤
0@trueA ‡e will prove th—t f(a + b, 0, c) = f(1 − c, 0, c) ≤ 1
2 whi™h is equiv—lent to
1
32
∗ (16c4
− 32c3
+ 20c2
− 4c − 1))(−1 + 2c)2
≤ 0
remem˜er th—t
16c4
− 32c3
+ 20c2
− 4c − 1 = 4(2c2
− 2c + 1−
√
5
4 )(2c2
− 2c + 1+
√
5
4 ) ≥ 0 for every c ∈ [0, 1]
TSF
vet a, b, c ˜e the sides of tri—ngleF €rove th—tX
a
2a − b + c
+
b
2b − c + a
+
c
2c − a + b
≥
3
2
SolutionX
the inequ—lity is equiv—lent to
1
1 + a
a+c−b
≤
3
2
fy g—u™hy ‡e h—ve X
a
a + c − b
+ 1 ≥ 2
a
a + c − b
ƒo ‡e need to prove
a + c − b
a
≤ 3
fe™—use a, b, c ˜e the sides of — tri—ngle so ‡e h—veX
a + c − b
a
=
sin A + sin C − sin B
sin A
=
2 sin C
2 cos B
2
cos A
2
it9s following th—t
a + c − b
a
=
cos B
2
√
sin C + cos C
2
√
sin A + cos A
2
√
sin B
cos A
2 cos B
2 cos C
2
≤
cos2 B
2 + cos2 C
2 + cos2 A
2 (sin C + sin A + sin B)
cos A
2 cos B
2 cos C
2
= 2 (cos B + cos C + cos A) + 6 ≤ 2.
3
2
+ 6 = 3
TTF
vet a, b, c, d > 0F€rove the following inequ—lityF‡hen does the equ—lity holdc
3
1
+
5
1 + a
+
7
1 + a + b
+
9
1 + a + b + c
+
36
1 + a + b + c + d
4 1 +
1
a
+
1
b
+
1
c
+
1
d
RW
SolutionX
‡e ™—n h—ve
(1 + a + b + c + d)(
4
25
+
16
25a
+
36
25b
+
64
25c
+
4
d
) ≥ (
2
5
+
4
5
+
6
5
+
8
5
+ 2)2
= 9
so
(
4
25
+
16
25a
+
36
25b
+
64
25c
+
4
d
) ≥ 36
1
1 + a + b + c + d
—nd
(1 + a + b + c)(
9
100
+
9
25a
+
81
100b
+
36
25c
) ≥ (
3
10
+
3
5
+
9
10
+
6
5
)
so ‡e h—ve
(
9
100
+
9
25a
+
81
100b
+
36
25c
) ≥ 9
1
1 + a + b + c
—nd
(1 + a + b)(
7
36
+
7
9a
+
7
4b
) ≥ 7
‡e get
(
7
36
+
7
9a
+
7
4b
) ≥
7
1 + a + b
—nd
(1 + a)(
5
9
+
20
9a
) ≥ 5
then
5
9
+
20
9a
≥
5
1 + a
—nd —dd these inequ—lity up ‡e ™—n solve the pro˜lemF
TUF
vet —D˜D™ ˜e positive re—l num˜er su™h th—t 9 + 3abc = 4(ab + bc + ca)
€rove th—t a + b + c ≥ 3
SolutionX
„—ke a = x + 1; b = y + 1; c = z + 1Dthen ‡e must prove th—tX
x + y + z ≥ 0 when 5(x + y + z) + xy + xz + yz = 3xyz
‡e ™onsider three ™—seX
g—se IXxyz ≥ 0 ⇒ (x+y+z)2
3 + 5(x + y + z)
≥ 5(x + y + z) + xy + xz + yz = 3xyz ≥ 0 ⇒ x + y + z ≥ 0
g—se PX x ≥ 0; y ≥ 0; z ≤ 0
essume th—t x + y + z ≤ 0 ⇒ −x ≥ y + zF
5(x + y + z) = yz(3x − 1) − x(y + z) ≥ −4yz + (y + z)2
= (y − z)2
≥ 0
g—se QX x ≤ 0; y ≤ 0; z ≤ 0Fy˜serve th—t −x, −y, −z ∈ [0, 1] thenX
0 = 5(x+y+z)+xy+yz+xz−3xyz ≤ 5(x+y+z)+2(xy+xz+yz) ≤ 5(x+y+z)+
2(x + y + z)2
3
⇒ x+y+z ≥ 0
ƒo ‡e h—ve doneF
TVF
if a, b, c, d —re nonEneg—tive re—l num˜ers su™h th—t a + b + c + d = 4D then
a2
b2 + 3
+
b2
c2 + 3
+
c2
d2 + 3
+
d2
a2 + 3
≥ 1.
SH
ƒyv…„iyxX
fy g—u™hyEƒ™hw—rz
[a2
(b2
+ 3) + b2
(c2
+ 3) + c2
(d2
+ 3)](
a2
b2 + 3
+
b2
c2 + 3
+
c2
d2 + 3
+
d2
a2 + 3
) ≥ (a2
+ b2
+ c2
)2
.
prom g—u™hy ‡e see th—t it is suffi™ient to prove th—t
(a2
+ b2
+ c2
+ d2
)2
≥ 3(a2
+ b2
+ c2
+ d2
) + a2
b2
+ b2
c2
+ c2
d2
+ d2
a2
whi™h ™—n ˜e rewritten —s
(a2
+ b2
+ c2
+ d2
)(a2
+ b2
+ c2
+ d2
− 3) ≥ a2
b2
+ b2
c2
+ c2
d2
+ d2
a2
xow you must homogeneize to h—ve r—r—z‡e form
(a2
+ b2
+ c2
+ d2
)(a2
+ b2
+ c2
+ d2
− 3(
a + b + c + d
4
)2
) ≥ a2
b2
+ b2
c2
+ c2
d2
+ d2
a2
whi™h follows from
a2
+ b2
+ c2
+ d2
≥ 4
—nd
(x + y + z + t)2
≥ 4(xy + yz + zt + tx)
with x = a2
—nd simil—rF
TWF
if a ≥ 2D b ≥ 2D c ≥ 2 —re re—lsD then prove th—t
8 a3
+ b b3
+ c c3
+ a ≥ 125 (a + b) (b + c) (c + a)
ƒyv…„iyxX
vets write LHS —s
8 ∗ (a3
b3
c3
+ abc + a4
b3
+ b4
c3
+ c4
a3
+ a4
c + b4
a + c4
b)
prom the wuirhe—ds inequ—lity ‡e h—ve th—t
a4
b3
+ b4
c3
+ c4
a3
≥ a3
b2
c2
= a2
b2
c2
(a + b + c)
—nd
a4
c + b4
a + c4
b ≥ a3
bc = abc(a2
+ b2
+ c2
)
F(∗∗)Nowletsseethat
a2
b2
c2
= (ab)(bc)(ca) ≥ (a + b)(b + c)(c + a)Thisiseasytoprove.
Fromthe@BBAWegetthat
LHS ≥ 8(a3
b3
c3
+ abc + a2
b2
c2
(a + b + c) + abc(a2
+ b2
+ c2
))
= 8a2
b2
c2
(abc +
1
abc
+ a + b + c +
a2
+ b2
+ c2
abc
)
so ‡e h—ve to prove th—t X
abc +
1
abc
+ a + b + c +
a2
+ b2
+ c2
abc
≥
125
8
SI
or
8a2
b2
c2
+ 8 + 8abc(a + b + c) + 8(a2
+ b2
+ c2
) ≥ 125abc
‡vyq let a ≥ b ≥ c
xow let abc = P ‡e will m—ke th—t following ™h—nge
a →
a
—nd b → b where ≥ 1 —nd a F
„he RHS doesnt ™h—ngeF in the LHS the first p—rt —lso doesn9t ™h—ngeF
a + b ≥
a
+ b
equiv—lent to ( − 1)(a − b ) whi™h is trueF elso ‡e get th—t
a2
+ b2
≥
a2
2
+ b2 2
ƒo —s ‡e get the num˜ers ™loser to e—™h other the LHS de™re—ses while the RHS rem—ins
the s—me so it is enough to prove the inequ—lity for the ™—se a = b = c whi™h is equiv—lent
to X
8a6
+ 8 + 24a4
+ 24a2
≥ 125a3
‡hi™h is pretty e—syF
UHD
vet a, b, c ≥ 0F €rove th—tX
1
√
2a2 + ab + bc
+
1
√
2b2 + bc + ca
+
1
√
2c2 + ca + ab
≥
9
2(a + b + c)
.
SolutionX
‡e h—veX
cyc
1
√
2a2 + ab + bc
=
cyc
2a + b + c
2 · 2a+b+c
2
√
2a2 + ab + bc
≥
≥
cyc
2a + b + c
2a+b+c
2
2
+ 2a2 + ab + bc
.
fut
cyc
2a + b + c
2a+b+c
2
2
+ 2a2 + ab + bc
≥
9
2(a + b + c)
⇔
⇔
cyc
(100a6
+ 600a5
b + 588a5
c + 1123a4
b2
− 357a4
c2
− 1842a3
b3
+
+1090a4
bc − 1414a3
b2
c + 1330a3
c2
b − 1218a2
b2
c2
) ≥ 0,
whi™h is e—syF
UIF vet —D˜D™ b H F€rove th—t X
a
a + 2b
+
b
b + 2c
+
c
c + 2a
≤
3(a2
+ b2
+ c2
)
(a + b + c)2
SP
SolutionX
⇐⇒ 1 −
a
a + 2b
≥ 3 −
3(a2
+ b2
+ c2
)
(a + b + c)2
=
6(ab + bc + ca)
(a + b + c)2
⇐⇒
b
a + 2b
+
c
b + 2c
+
a
c + 2a
≥
3(ab + bc + ca)
(a + b + c)2
fy g—u™hyEƒ™hw—rz ‡e get
b
a + 2b
b(a + 2b) ≥ (a + b + c)2
it suffi™e to show th—t
(a + b + c)4
≥ 3(ab + bc + ca)(ab + bc + ca + 2a2
+ 2b2
+ 2c2
)
‡ithout loss of generosityD—ssume th—t ab + bc + ca = 3Dthen it ˜e™omes
(a + b + c)2
− 9
2
≥ 0
whi™h is o˜viousF
UPF
vet a, b, c, d ˜e positive re—l num˜ersF €rove th—t the following inequ—lity holds
a4
+ b4
(a + b)(a2 + ab + b2)
+
b4
+ c4
(b + c)(b2 + bc + c2)
+
c4
+ d4
(c + d)(c2 + cd + d2)
+
d4
+ a4
(d + a)(d2 + da + a2)
≥
a2
+ b2
+ c2
+ d2
a + b + c + d
SolutionX
a4
+ b4
(a + b)(a2 + ab + b2)
≥
1
2 (a2
+ b2
)2
(a + b)(a2 + ab + b2)
„hus D it rem—ins to prove th—t
cyc
(a2
+ b2
)2
(a + b)(a2 + ab + b2)
≥
2(a2
+ b2
+ c2
+ d2
)
a + b + c + d
e™ording to g—u™hyEƒhw—rz inequ—lity ‡e h—ve X
LHS ≥
4(a2
+ b2
+ c2
+ d2
)2
A
where A = cyc(a + b)(a2
+ ab + b2
) = 2(a3
+ b3
+ c3
+ d3
) + 2 cyc ab(a + b)
it suffi™es to show th—t
2(a2
+ b2
+ c2
+ d2
)(a + b + c + d) ≥ 2(a3
+ b3
+ c3
+ d3
) + 2
cyc
ab(a + b)
⇔
cyc
2a3
+ 2a2
b + 2a2
c + 2a2
d − 2a3
− 2ab(a + b) =
cyc
2a2
c ≥ 0
UQF
if a, b, c —re nonneg—tive re—l num˜ersD then
a a2 + 4b2 + 4c2 ≥ (a + b + c)2
.
SolutionF
in the nontrivi—l ™—se when two of —D˜D™ —re nonzeroD ‡e t—ke squ—re ˜oth sides —nd write
SQ
the inequ—lity —s
a2
(a2
+ 4b2
+ 4c2
) + 2 ab (a2 + 4b2 + 4c2)(4a2 + b2 + 4c2) ≥≥ ( a)
4
.
epplying the g—u™hyEƒ™hw—rz inequ—lity in ™om˜in—tion with the trivi—l inequ—lity
(u + 4v)(v + 4u) ≥ 2u + 2v +
2uv
u + v
∀u, v ≥ 0, u + v > 0,
‡e get
ab (a2 + 4b2 + 4c2)(4a2 + b2 + 4c2) ≥ ab (a2 + 4b2)(b2 + 4a2) + 4c2
≥ ab 2a2
+ 2b2
+
2a2
b2
a2 + b2
+ 4c2
.
„hereforeD it suffi™es to prove th—t
a4
+ 8 a2
b2
+ 4 ab(a2
+ b2
) + 8abc a + 4 a3
b3
a2+b2 ≥
≥ ( a)
4
.
„his inequ—lity redu™es toF
URF
vet —D˜D™ ˜e nonneg—tive re—l num˜ersD no two of whi™h —re zeroF €rove th—t
b2
+ c2
a2 + bc
+
c2
+ a2
b2 + ca
+
a2
+ b2
c2 + ab
≥
2a
b + c
+
2b
c + a
+
2c
a + b
.
SolutionF
fy the g—u™hyEƒ™hw—rz inequ—lityD ‡e h—ve
b2
+ c2
a2 + bc
≥
(b2
+ c2
)
2
(b2 + c2)(a2 + bc)
=
4(a2
+ b2
+ c2
)2
ab(a2 + b2) + 2 a2b2
.
„hereforeD it suffi™es to prove th—t
2(a2
+ b2
+ c2
)2
≥
a ab(a2
+ b2
) + 2 a2
b2
b + c
.4
ƒin™e
ab(a2
+ b2
) + 2 a2
b2
=
= (b + c)[a3
+ 2a2
(b + c) + bc(b + c) + a(b2
− bc + c2
)] − 4a2
bc,
this inequ—lity ™—n ˜e written —s
2 a2
2
+ 4abc
a2
b + c
≥
≥ a[a3
+ 2a2
(b + c) + bc(b + c) + a(b2
− bc + c2
)],
or equiv—lentlyD
a4
+ 2 a2
b2
+ 4abc
a2
b + c
≥ abc a + 2 a3
(b + c).
xowD ˜y ghe˜yshev9s inequ—lityD ‡e h—ve
a2
b + c
≥
3(a2
+ b2
+ c2
)
2(a + b + c)
,
SR
—nd thusD it suffi™es to show th—t
a4
+ 2 a2
b2
+
6abc a2
a
≥ abc a + 2 a3
(b + c).
efter some simple ™omput—tionsD ‡e ™—n write this inequ—lity —s
a3
(a − b)(a − c) + b3
(b − c)(b − a) + c3
(c − a)(c − b) ≥ 0,
whi™h is ƒ™hur9s inequ—lityF „he Solution is ™ompletedF iqu—lity holds if —nd only if a =
b = c, ora = b —nd c = 0, or —ny ™y™li™ permut—tionFF
a2
b2
+ 2
a3
b3
a2 + b2
≥ 2abc a,
or
a2
b2
(a + b)2
a2 + b2
≥ 2abc a.
fy the g—u™hyEƒ™hw—rz inequ—lityD ‡e h—ve
a2
b2
(a + b)2
a2 + b2
≥
[ab(a + b) + bc(b + c) + ca(c + a)]2
(a2 + b2) + (b2 + c2) + (c2 + a2)
,
—nd thusD it is enough to to ™he™k th—t
[ab(a + b) + bc(b + c) + ca(c + a)]2
≥ 4abc(a + b + c)(a2
+ b2
+ c2
).
‡ithout loss of gener—lityD —ssume th—t b is ˜etien a—nd cF prom the ewEqw inequ—lityD ‡e
h—ve
4abc(a + b + c)(a2
+ b2
+ c2
) ≤ [ac(a + b + c) + b(a2
+ b2
+ c2
)]2
.
yn the other h—ndD one h—s
ac(a + b + c) + b(a2
+ b2
+ c2
) − [ab(a + b) + bc(b + c) + ca(c + a)] =
= −b(a − b)(b − c) ≤ 0.
gom˜ining these two inequ—litiesD the ™on™lusion followsF iqu—lity o™™urs if —nd only if
—a˜a™D or a = b = 0, orb = c = 0, orc = a = 0.
USF vet a, b, c ˜e positive re—l num˜erF €rove th—tX
2(b + c)
a
≥
27(a + b)(b + c)(c + a)
4(a + b + c)(ab + bc + ca)
SolutionF
fy ewEqw inequ—lity ‡e h—ve
1
a 2(a2 + bc)
=
√
b + c
√
2a. (ab + ac)(a2 + bc)
≥
2(b + c)
√
a.(a + b)(a + c)
it suffi™es to show th—t
(b + c)
2(b + c)
a
≥
9(a + b)(b + c)(c + a)
2(ab + bc + ca)
fy ghe˜yselv inequ—lity ‡e h—ve
(b + c)
2(b + c)
a
≥
2
3
(a + b + c).
2(b + c)
a
SS
ren™eD it suffi™es to show th—t
2(b + c)
a
≥
27(a + b)(b + c)(c + a)
4(a + b + c)(ab + bc + ca)
fy g—u™hyEƒ™hw—rz inequ—lityD ‡e get
2(b + c)
a
2
a(b + c)2
≥ 16(a + b + c)3
end ˜y ewEqw inequ—lityD
27(a + b)(b + c)(c + a) ≤ 8(a + b + c)3
pin—llyD ‡e need to show th—t
16(a + b + c)3
a(b + c)2
≥
27.8(a + b + c)3
(a + b)(b + c)(c + a)
16(a + b + c)2(ab + bc + ca)2
or
32(a + b + c)2
(ab + bc + ca)2
≥ 27(a + b)(b + c)(c + a) ((a + b)(b + c)(c + a) + 4abc)
or
5x2
+ 32y2
≥ 44xy
where ‡e setting x = (a + b)(b + c)(c + a)D y = abc —nd using the equ—lity (a + b + c)(ab +
bc + ca) = x + y
„he l—st inequ—lity is true ˜e™—use it equiv—lent (x − 8y)(5x − 4y) ≥ 0D o˜viouslyF
UTF
if —D˜D™ —re positive re—l num˜ersD then
1
a 2(a2 + bc)
≥
9
2(ab + bc + ca)
.
pirst SolutionF
fy rolder9s inequ—lityD ‡e h—ve
1
a
√
a2 + bc
2
a2
+ bc
a
≥
1
a
3
.
it follows th—t
1
a
√
a2 + bc
2
≥
(ab + bc + ca)3
a2
b2
c2
a2
b2
+ a2
bc
,
—nd hen™eD it suffi™es to prove th—t
2(ab + bc + ca)5
≥ 81a2
b2
c2
a2
b2
+ a2
bc .
ƒetting x = bc, y = ca —nd z = abD this inequ—lity ˜e™omes
2(x + y + z)5
≥ 81xyz(x2
+ y2
+ z2
+ xy + yz + zx).
…sing the illEknown inequ—lity
xyz ≤
(x + y + z)(xy + yz + zx)
9
,
ST
‡e see th—t it is enough to ™he™k th—t
2(x + y + z)4
≥ 9(xy + yz + zx)(x2
+ y2
+ z2
+ xy + yz + zx),
whi™h is equiv—lent to the o˜vious inequ—lity
(x2
+ y2
+ z2
− xy − yz − zx)(2x2
+ 2y2
+ 2z2
+ xy + yz + zx) ≥ 0.
„he Solution is ™ompletedF iqu—lity holds if —nd only if a = b = c.
ƒe™ond SolutionF fy the ewEqw inequ—lityD ‡e h—ve
1
a 2(a2 + bc)
=
√
b + c
√
2a (ab + ac)(a2 + bc)
≥
2(b + c)
√
a(a + b)(a + c)
.
„hereforeD it suffi™es to prove th—t
b + c
2a
·
1
(a + b)(a + c)
≥
9
4(ab + bc + ca)
.
‡ithout loss of gener—lityD —ssume th—t a ≥ b ≥ cF ƒin™e
b + c
2a
≤
c + a
2b
≤
a + b
2c
—nd
1
(a + b)(a + c)
≤
1
(b + c)(b + a)
≤
1
(c + a)(c + b)
,
˜y ghe˜yshev9s inequ—lityD ‡e get
b + c
2a
·
1
(a + b)(a + c)
≥
1
3
b + c
2a
1
(a + b)(a + c)
=
2(a + b + c)
3(a + b)(b + c)(c + a)
b + c
2a
.
ƒoD it suffi™es to show th—t
b + c
2a
≥
27(a + b)(b + c)(c + a)
8(a + b + c)(ab + bc + ca)
.
ƒetting
t =
6 (a + b)(b + c)(c + a)
8abc
t ≥ 1. fy the ewEqw inequ—lityD ‡e h—ve
b + c
2a
≥ 3t.
elsoD it is e—sy to verify th—t
27(a + b)(b + c)(c + a)
8(a + b + c)(ab + bc + ca)
=
27t6
8t6 + 1
.
ƒoD it is enough to ™he™k th—t
3t ≥
27t6
8t6 + 1
,
SU
or
8t6
− 9t5
+ 1 ≥ 0.
ƒin™e t ≥ 1D this inequ—lity is true —nd the Solution is ™ompletedF
UTF
qive a1, a2, ..., an ≥ 0—re num˜ers h—ve sum is IF €rove th—t if n > 3 so
a1a2 + a2a3 + ... + ana1 ≤
1
4
SolutionX
vet n = 2kD where k ∈ N —nd a1 + a3 + ... + a2k−1 = xF
ren™eD
a1a2 + a2a3 + ... + ana1 ≤ (a1 + a3 + ... + a2k−1) (a2 + a4 + ... + a2k) =
= x(1 − x) ≤
1
4
vet n = 2k − 1 —nd a1 = mini{ai}F
ren™eD
a1a2 + a2a3 + ... + ana1 ≤
≤ a1a2 + a2a3 + ... + ana2 ≤ (a1 + a3 + ... + a2k−1) (a2 + a4 + ... + a2k−2) =
= x(1 − x) ≤
1
4
UUF
vet a, b, c ˜e nonEneg—tive re—l num˜ersF €rove th—t
a3
+ 2abc
a3 + (b + c)3
+
b3
+ 2abc
b3 + (c + a)3
+
c3
+ 2abc
c3 + (a + b)3
≥ 1
Solution
‡e h—ve
1−
a3
a3 + (b + c)3
=
a3
a3 + b3 + c3
−
a3
a3 + (b + c)3
=
3a3
bc(b + c)
(a3 + b3 + c3)(a3 + (b + c)3)
ren™eD it suffi™es to show th—t
2
a3
+ b3
+ c3
a3 + (b + c)3
≥
3a2
(b + c)
a3 + (b + c)3
⇔
2a3
− 3a2
(b + c) + 2b3
+ 2c3
a3 + (b + c)3
≥ 0
⇔
(a − b)(a2
− 2ab − 2b2
) + (a − c)(a2
− 2ac − 2c2
)
a3 + (b + c)3
≥ 0
⇔
(a − b)3
− (c − a)3
+ 3c2
(c − a) − 3b2
(a − b)
a3 + (b + c)3
≥ 0
it suffi™es to show th—t
c2
(c − a) − b2
(a − b)
a3 + (b + c)3
≥ 0
—nd
(a − b)3
− (c − a)3
a3 + (b + c)3
≥ 0
SV
„he first inequ—lity is equiv—lent to
⇔ (a − b)
a2
b3 + (c + a)3
−
b2
a3 + (b + c)3
≥ 0
pin—llyD to finish the SolutionD ‡e will show th—t if a ≥ bD then
a2
b3 + (c + a)3
≥
b2
a3 + (b + c)3
⇔ a5
− b5
≥ b2
(c + a)3
− a2
(b + c)3
⇔ a5
− b5
≥ a2
b2
(a − b) + c3
(b2
− a2
) + 3c2
ab(b − a)
whi™h is o˜viously true sin™e a ≥ b —nd c ≥ 0F
end the se™ond inequ—lity is equiv—lent to
(a − b)3 1
a3 + (b + c)3
−
1
b3 + (c + a)3
≥ 0
⇔
(a − b)4
(3c(a + b) + 3c2
)
(a3 + (b + c)3)(b3 + (c + a)3)
≥ 0
whi™h is o˜viously trueF
iqu—lity holds for a = b = c or abc = 0
UVF
vet a, b, c —re positive re—l num˜ersD prove th—t
3
a
b
+
b
c
+
c
a
+ 2
ab + bc + ca
a2 + b2 + c2
≥ 5
SolutionX
fy using the ill known
2
a
b
+
b
c
+
c
a
+ 1 ≥
21 a2
+ b2
+ c2
(a + b + c)
2
ƒetting x = ab+bc+ca
a2+b2+c2 ≤ 1F it suffi™es to show th—t
3 (10 − x2)
2x2 + 1
+ 2x ≥ 5
⇔
3 10 − x2
2x2 + 1
≥ 4x2
− 20x + 25
⇔
−8x4
+ 40x3
− 57x2
+ 20x + 5
2x2 + 1
≥ 0
⇔
(x − 1) −8x3
+ 32x2
− 25x − 5
2x2 + 1
≥ 0
whi™h is ™le—rly trueF
a
b + b
c + c
a
3
a
b + b
c + c
a
3
ab + bc + ca
a2 + b2 + c2
≥ 1
end ‡e —lso note th—t the folloid is not true
a
b + b
c + c
a
3
ab + bc + ca
a2 + b2 + c2
≥ 1
SW
UWF
vet a, b, c ˜e the sideElengths of — tri—ngle su™h th—t a2
+ b2
+ c2
= 3. €rove th—t
bc
1 + a2
+
ca
1 + b2
+
ab
1 + c2
≥
3
2
.
SolutionF
‡rite the inequ—lity —s
2bc
4a2 + b2 + c2
≥ 1.
ƒin™e 1 = b2
c2
a2b2+b2c2+c2a2 , this inequ—lity is equiv—lent to
2bc
4a2 + b2 + c2
−
b2
c2
a2b2 + b2c2 + c2a2
≥ 0,
or
abc
a2b2 + b2c2 + c2a2
(2a2
− bc)(b − c)2
a(4a2 + b2 + c2)
≥ 0.
‡ithout loss of gener—lityD —ssume th—t a ≥ b ≥ c. ƒin™e
(2a2
− bc)(b − c)2
a(4a2 + b2 + c2)
≥ 0,
it suffi™es to prove th—t
(2b2
− ca)(c − a)2
b(4b2 + c2 + a2)
+
(2c2
− ab)(a − b)2
c(4c2 + a2 + b2)
≥ 0.
ƒin™e a, b, c —re the sideElengths of — tri—ngle —nd a ≥ b ≥ c, ‡e h—ve
2b2
− ca ≥ c(b + c) − ca = c(b + c − a) ≥ 0,
—nd
a − c −
b
c
(a − b) =
(b − c)(b + c − a)
c
≥ 0.
„hereforeD
(2b2
− ca)(c − a)2
b(4b2 + c2 + a2)
≥
b(2b2
− ca)(a − b)2
c2(4b2 + c2 + a2)
.
it suffi™es to show th—t
b(2b2
− ca)
4b2 + c2 + a2
+
c(2c2
− ab)
4c2 + a2 + b2
≥ 0,
or
b(2b2
− ca)
4b2 + c2 + a2
≥
c(ab − 2c2
)
4c2 + a2 + b2
.
ƒin™e ab − 2c2
− (2b2
− ca) = a(b + c) − 2(b2
+ c2
) ≤ a(b + c) − (b + c)2
≤ 0, it is enough to
™he™k th—t
b
4b2 + c2 + a2
≥
c
4c2 + a2 + b2
,
whi™h is true ˜e™—use
b(4c2
+ a2
+ b2
) − c(4b2
+ c2
+ a2
) = (b − c)[(b − c)2
+ (a2
− bc)] ≥ 0.
TH
„he Solution is ™ompletedF
VHF
vet a, b, c, d > 0 su™h th—t a2
+ b2
+ c2
+ d2
= 4Dthen
1
a
+
1
b
+
1
c
+
1
d
≤ 2 +
2
abcd
Solution
write the inequ—lity —s
abc + bcd + cda + dab ≤ 2abcd + 2.
‡ithout loss of gener—lityD —ssume th—t a ≥ b ≥ c ≥ d.
vet
t =
a2 + b2
2
,
=> 1 ≤ t ≤
√
2. ƒin™e
2
1
c
+
1
d
≥
1
a
+
1
b
+
1
c
+
1
d
≥
16
a + b + c + d
≥
16
4(a2 + b2 + c2 + d2)
= 4,
‡e h—vec + d ≥ 2cd.
„hereforeD
abc + bcd + cda + dab − 2abcd = ab(c + d − 2cd) + cd(a + b)
≤
a2
+ b2
2
(c + d − 2cd) + cd 2(a2 + b2)
= t2
(c + d − 2cd) + 2tcd.
it suffi™es to prove th—t
t2
(c + d − 2cd) + 2tcd ≤ 2,
or
2tcd(1 − t) + t2
(c + d) ≤ 2.
…sing the ewEqw inequ—lityD ‡e get
c + d ≤
(c + d)2
+ 4
4
=
(4 − 2t2
+ 2cd) + 4
4
=
4 − t2
+ cd
2
.
ƒoD it is enough to ™he™k th—t
4tcd(1 − t) + t2
(4 − t2
+ cd) ≤ 4,
or
tcd(4 − 3t) ≤ (2 − t2
)2
.
ƒin™e 2 − t2
= c2
+d2
2 ≥ cd, ‡e h—ve
(2 − t2
)2
− tcd(4 − 3t) ≥ cd(2 − t2
) − tcd(4 − 3t) = 2cd(t − 1)2
≥ 0.
TI
„he Solution is ™ompletedF
VIF
vet a, b, c ˜e positive re—l num˜er F€roveX
cyc
3
(a2 + ab + b2)2 ≤ 3
3
cyc
(2a2 + bc)
2
SolutionX
cyc
3
(a2 + ab + b2)2 ≤ 3
3
cyc
(2a2 + bc)
2
↔
cyc
3
3(a2 + ab + b2)2 ≤ 3
9
cyc
(2a2 + bc)
2
fy holder9s inequ—lityX
cyc
3
3(a2 + ab + b2)2 ≤ 3 9(
cyc
(a2 + ab + b2))2
ƒo ‡e must proveX
3 9(
cyc
(a2 + ab + b2))2 ≤ 3 (
cyc
(2a2 + bc))
2
<=> (
cyc
(a2
+ ab + b2
))2
≤ 3
cyc
(2a2
+ bc)2
…sing g—u™hyEƒ™h—wrz9s inequ—lityD
3
cyc
(2a2
+ bc)2
≥ (
cyc
(2a2
+ bc))2
=
cyc
(a2
+ ab + b2
)2
FiFh
F
VPF
vet a, b, c > 0Fprove th—tX
cyc
1
a2 + bc
≥
cyc
1
a2 + 2bc
+
ab + bc + ca
2(a2b2 + b2c2 + c2a2)
SolutionX
IFFF
(a2
b2
+ b2
c2
+ c2
a2
)
cyc
1
a2 + bc
=
cyc
bc +
a2
(b2
+ c2
− bc)
a2 + bc
<=> 2
cyc
a2
(b2
− bc + c2
)
a2 + bc
≥ ab + bc + ca
<=> 2
cyc
a2
1 +
b2
− bc + c2
a2 + bc
≥ 2(a2
+ b2
+ c2
) + ab + bc + ca
<=>
cyc
a2
a2 + bc
≥ 1 +
ab + bc + ca
2(a2 + b2 + c2)
TP
fy g—u™hyEƒ™hw—rzD
cyc
a2
a2 + bc
≥
(a + b + c)2
cyc a2 + cyc bc
= 1 +
ab + bc + ca
cyc a2 + cyc bc
≥ 1 +
ab + bc + ca
2(a2 + b2 + c2)
FiFh
PFF
„— ™âX
(a2
b2
+ b2
c2
+ c2
a2
)
cyc
1
a2 + bc
=
cyc
b2
+ c2
− bc
b2
+ c2
− bc
a2 + bc
<=> 2(a2
+ b2
+ c2
) −
cyc
bc
b2
+ c2
− bc
a2 + bc
≤
3
2
(a2
+ b2
+ c2
)
r—y l
2
cyc
bc
b2
+ c2
− bc
a2 + bc
≥ a2
+ b2
+ c2
fy ewEqw9s inequ—lityX
2(b2
+ c2
− bc) ≥ b2
+ c2
end ‡e will proveX
cyc
bc(b2
+ c2
)
a2 + bc
≥ a2
+ b2
+ c2
fy g—u™hyEƒ™hw—rz9s inequ—lityX
LHS ≥
( ab
√
a2 + b2)2
bc(a2 + bc)
( bc b2 + c2)2
≥ ( a2
)(abc a + a2
b2
)
…sing g—u™hyEƒ™hw—rzD
a2 + b2 a2 + c2 ≥ a2
+ bc
<=> abc(a3
+ b3
+ c3
+ 3abc − a2
b ≥ 0)
it is trueF
VQF
if —D˜D™ —re positive re—l num˜ersD then
a2
(b + c)
b2 + c2
+
b2
(c + a)
c2 + a2
+
c2
(a + b)
a2 + b2
≥ a + b + c.
pirst SolutionF
‡e h—ve
a2
(b + c)
b2 + c2
− a =
ab(a − b) − ca(c − a)
b2 + c2
= ab(a − b)
1
b2 + c2
−
1
c2 + a2
=
ab(a + b)(a − b)2
(a2 + c2)(b2 + c2)
≥ 0.
TQ
„husD it follows th—t
a2
(b + c)
b2 + c2
− a ≥ 0,
or
a2
(b + c)
b2 + c2
+
b2
(c + a)
c2 + a2
+
c2
(a + b)
a2 + b2
≥ a + b + c,
whi™h is just the desired inequ—lityF iqu—lity holds if —nd only if a = b = c.
ƒe™ond SolutionF
r—ving in view of the identity
a2
(b + c)
b2 + c2
=
(b + c)(a2
+ b2
+ c2
)
b2 + c2
− b − c,
‡e ™—n write the desired inequ—lity —s
b + c
b2 + c2
+
c + a
c2 + a2
+
a + b
a2 + b2
≥
3(a + b + c)
a2 + b2 + c2
.
‡ithout loss of gener—lityD —ssume th—t a ≥ b ≥ cF ƒin™e a2
+ c2
≥ b2
+ c2
—nd
b + c
b2 + c2
−
a + c
a2 + c2
=
(a − b)(ab + bc + ca − c2
)
(a2 + c2)(b2 + c2)
≥ 0,
˜y ghe˜yshev9s inequ—lityD ‡e h—ve
[(b2
+ c2
) + (a2
+ c2
)]
b + c
b2 + c2
+
a + c
a2 + c2
≥ 2[(b + c) + (a + c)],
or
b + c
b2 + c2
+
a + c
a2 + c2
≥
2(a + b + 2c)
a2 + b2 + 2c2
.
„hereforeD it suffi™es to prove th—t
2(a + b + 2c)
a2 + b2 + 2c2
+
a + b
a2 + b2
≥
3(a + b + c)
a2 + b2 + c2
,
whi™h is equiv—lent to the o˜vious inequ—lity
c(a2
+ b2
− 2c2
)(a2
+ b2
− ac − bc)
(a2 + b2)(a2 + b2 + c2)(a2 + b2 + 2c2)
≥ 0.
Solution Q
xote th—t from g—u™hyEƒ™hw—rtz inequ—lity ‡e h—ve
cyc
a2
(b + c)
(b2 + c2)
≥
cyc
cyc a2
(b + c)
2
cyc a2(b + c)(b2 + c2)
„herefore it suffi™es to show th—t
cyc
a2
(b + c)
2
≥ (a + b + c)
cyc
a2
(b + c)(b2
+ c2
)
efter exp—nsion —nd using the ™onvention p = a + b + c; q = ab + bc + ca; r = abc this is
equiv—lent to withX
⇔ r(2p3
+ 9r − 7pq) ≥ 0
TR
futD sin™e @from trivi—l inequ—lityA ‡e h—vep2
− 3q ≥ 0D hen™e it suffi™es to show th—t
p3
+ 9r ≥ 4pq, whi™h follows from ƒ™hur9s inequ—lityF
iqu—lity o™™urs if —nd only if —a˜a™ or when a = b; c = 0 —nd its ™y™li™ permut—tionsF
VRF
if —D˜D™ —re positive num˜ers su™h th—t a + b + c = 3 then
a
3a + b2
+
b
3b + c2
+
c
3c + a2
≤
3
4
.
SolutionX
is equiv—lent to
a
3a + b2
≤
3
4
3a
3a + b2
− 1 ≤ −
3
4
or
b2
b2 + 3a
≥
3
4
fy g—u™hy ƒ™hw—rz inequ—lityD ‡e h—ve
LHS ≥
(a2
+ b2
+ c2
)2
a4 + (a + b + c) ab2
it suffi™es to prove
4(a2
+ b2
+ c2
)2
≥ 3 a4
+ 3 a2
b2
+ 3 ab3
+ 3 a2
bc
⇔ (a2
+ b2
+ c2
)2
− 3 ab3
+ 3( a2
b2
− a2
bc) ≥ 0
fy †—sg9s inequ—lityD ‡e h—ve
(a2
+ b2
+ c2
)2
− 3 ab3
≥ 0
fy em Eqw inequ—lityD
a2
b2
− a2
bc ≥ 0
VSF
if a ≥ b ≥ c ≥ d ≥ 0 —nd a + b + c + d = 2, then
ab(b + c) + bc(c + d) + cd(d + a) + da(a + b) ≤ 1.
SolutionX
pirstDlet us prove — lemm—X
vemm—X
por —ny a + b + c + d = 2 —nd a ≥ b ≥ c ≥ d ≥ 0
a2
b + b2
c + c2
d + d2
a ≥ ab2
+ bc2
+ cd2
+ da2
Solution of lemm—X
vet
F(a) = (b − d)a2
+ (d2
− b2
)a + b2
c + c2
d − bc2
− cd2
F (a) = (b − d)(2a − b − d) ≥ 0 <=> F(a) ≥ F(b) = (c − d)b2
+ (d2
− c2
)b + cd(c − d)
TS
F (b) = (c − d)(2b − c − d) ≥ 0 <=> F(b) ≥ F(c) = 0
xowDlet us turn ˜—™k the Solution of the pro˜lemF prom lemm— ‡e h—veX
a2
b+b2
c+c2
d+d2
a+ab2
+bc2
+cd2
+da2
+2(abc+bcd+cda+dab) ≥ 2(ab2
+bc2
+cd2
+da2
+abc+bcd+cda+dab
it follows th—tY
(a + b + c + d)(a + c)(b + d) ≥ 2(ab(b + c) + bc(c + d) + cd(d + a) + da(a + b))
futD ˜y ewEqw inequ—lityX
(a + c)(b + d) = (a + c)(b + d) ≤
(a + b + c + d)2
4
= 1
VTF
if x, y, z, p, q ˜e nonneg—tive re—l num˜ers su™h th—t
(p + q)(yz + zx + xy) > 0
€rove th—tX
2(p + q)
(y + z)(py + qz)
+
2(p + q)
(z + x)(pz + qx)
+
2(p + q)
(x + y)(px + qy)
≥
9
yz + xy + zx
SolutionX
cyc
2(p + q)
(y + z)(py + qz)
−
9
yz + zx + xy
≡
F(x, y, z)
(y + z)(z + x)(x + y)(py + qz)(pz + qx)(px + qy)(yz + zx + xy)
.
F(x, y, z) = F(x, x + s, x + t)
= 16x4
p3
+ q3
s2
− st + t2
+x3
16(p + q) p2
+ q2
(s + t)(s − t)2
+ (p − 2q)2
(15p + 7q)
+5q2
(p + q)s2
t + (q − 2p)2
(15q + 7p) + 5p2
(q + p) st2
+x2 2(s − t)2
[p2
(p + 47q)s2
+ 51pq(p + q)st + q2
(q + 47p)t2
]
3
+
2[(5p + q)(5p − 12q)2
+ 6q3
]s4
75
+
[(77p − 145q)2
(7918p + 6699q) + 3003p3
+ 14297pq2
]s3
t
5633859
+
34(p + q)(p − q)2
s2
t2
3
+
[(77q − 145p)2
(7918q + 6699p) + 3003q3
+ 14297qp2
]st3
5633859
+
2[(5q + p)(5q − 12p)2
+ 6p3
]t4
75
+x [(2p + 5q)s − (2q + 5p)t]
2 4pq(p + q)s3
(2p + 5q)2
+
(4p4
+ 40p3
q + 65p2
q2
+ 113pq3
+ 30q4
)s2
t
(2p + 5q)3
+
(4q4
+ 40q3
p + 65q2
p2
+ 113qp3
+ 30p4
)st2
(2q + 5p)3
+
4qp(q + p)t3
(2q + 5p)2
+
s3
t2
(2p + 5q)2(2q + 5p)3
(2p − 3q)2
1505p6
+ 9948p5
q + 19439p4
q2
TT
+16869p3
q3
+ 6709p2
q4
+ 852pq5
+ 117q6
+4960p7
q + 6800p6
q2
+ p5
q3
+ 4p4
q4
+ 5p3
q5
+ 8q6
p2
+ 4pq7
+ 7q8
+
s2
t3
(2p + 5q)3(2q + 5p)2
(2q−3p)2
1505q6
+ 9948q5
p + 19439q4
p2
+ 16869q3
p3
+ 6709q2
p4
+ 852qp5
+ 117p6
+4960q7
p + 6800q6
p2
+ q5
p3
+ 4q4
p4
+ 5q3
p5
+ 8p6
q2
+ 4qp7
+ 7p8
+st [(13p + 47q)s − (13q + 47p)t]2 2pq(p + q)s2
(13p + 47q)2
+
49pq(p + q)st
6(743p2 + 6914pq + 743q2)
+
2qp(q + p)t2
(13q + 47p)2
+(p − q)2
st
q(324773p3
+ 3233274p2
q + 836101pq2
+ 419052q3
)s2
6(13p + 47q)(743p2 + 6914pq + 743q2)
+
2(p + q)(p − q)2
(832132509p4
+ 9284734492p3
q + 9070265998p2
q2
+ 9284734492pq3
+ 832132509q4
)st
3(13p + 47q)2(13q + 47p)2(743p2 + 6914pq + 743q2)
+
p(324773q3
+ 3233274q2
p + 836101qp2
+ 419052p3
)t2
6(13q + 47p)(743p2 + 6914pq + 743q2)
≥ 0,
whi™h is ™le—rly true for x = min{x, y, z}.
VUF
vet —D˜D™ ˜e positive num˜ers su™h th—t ab + bc + ca = 3. €rove th—t
1
a + b
+
1
b + c
+
1
c + a
≥ 1 +
3
2(a + b + c)
.
SolutionX
IFFFFFvet f(a, b, c) = 1
a+b + 1
b+c + 1
c+a − 1 − 3
2(a+b+c) —nd a = min{a, b, c}. „hen
f(a, b, c) − f a, (a + b)(a + c) − a, (a + b)(a + c) − a =
=
√
a + b −
√
a + c
2 1
(a + b)(a + c)
−
1
2(b + c) (a + b)(a + c) − a
+
3
2(a + b + c) 2 · (a + b)(a + c) − a
≥
≥
√
a + b −
√
a + c
2 1
4bc
−
1
2(b + c) ·
√
bc
+
2
3(b + c)2
≥ 0
sin™eD (a + b)(a + c) ≥ a +
√
bc —nd 2 · (a + b)(a + c) − a ≤ a + b + c ≤ 3(b+c)
2 . „husD
rem—in to prove th—t f(a, b, b) ≥ 0, whi™h equiv—lent to
(a − b)2
(2a3
+ 9a2
b + 12ab2
+ b3
) ≥ 0.
PFFFFFFF
„he inequ—lity is equiv—lent toX
1
a + b
+
1
b + c
+
1
c + a
≥
3
3(ab + bc + ca)
+
3
2(a + b + c)
↔
1
a + b
+
1
b + c
+
1
c + a
−
9
2(a + b + c)
≥
3
3(ab + bc + ca)
−
3
a + b + c
TU
↔
(a − b)2
2(a + c)(b + c)(a + b + c)
+
(b − c)2
2(a + b)(a + c)(a + b + c)
+
(c − a)2
2(b + c)(a + b)(a + b + c)
≥
3[(a − b)2
+ (b − c)2
+ (c − a)2
]
2(a + b + c + 3(ab + bc + ca))(a + b + c)( 3(ab + bc + ca))
↔ (a − b)2
.M + (b − c)2
.N + (c − a)2
.P ≥ 0
with X
M = (a + b)[(a + b + c) + 3(ab + bc + ca)] 3(ab + bc + ca) − 3(a + b)(b + c)(c + a)
N = (b + c)[(a + b + c) + 3(ab + bc + ca)] 3(ab + bc + ca) − 3(a + b)(b + c)(c + a)
P = (c + a)[(a + b + c) + 3(ab + bc + ca)] 3(ab + bc + ca) − 3(a + b)(b + c)(c + a)
ƒuppose th—tXa ≥ b ≥ cF
ƒo ‡e h—veX
M = (a + b)([(a + b + c) + 3(ab + bc + ca)] 3(ab + bc + ca) − 3(b + c)(c + a)) ≥ 0
fe™—use (a + b + c) 3(ab + bc + ca) ≥ 3c2
P = (a + c)([(a + b + c) + 3(ab + bc + ca)] 3(ab + bc + ca) − 3(a + b)(b + c)) ≥ 0
fe™—use (a + b + c) 3(ab + bc + ca) ≥ 3b2
ƒo ‡e must proveX
N + P ≥ 0
it –s equiv—lent toX
X = [(a + b + c) + 3(ab + bc + ca)] 3(ab + bc + ca)(a + b + 2c) − 6(a + b)(b + c)(c + a) ≥ 0
€ut
x = a + b + c; y = 3(ab + bc + ca)
X ≥ [(a + b + c) + 3(ab + bc + ca)] 3(ab + bc + ca)(a + b + c) − 6(a + b + c)(ab + bc + ca)
↔ x2
y ≥ xy2
↔ x ≥ y
@it –s true for —ll positive num˜ers —D˜D™AF
VVF
vet a, b, c˜e positive re—l num˜er F €rove th—tX
1
a + b
+
1
b + c
+
1
c + a
≥
a + b + c
2(ab + bc + ca)
+
3
a + b + c
SolutionX vet put p = a + b + c, q = ab + bc + ca, r = abcD „his inequ—lity is equiv—lent toX
p2
+ q
pq − r
≥
p
2q
+
3
p
⇐⇒
p2
+ 3
3p − r
≥
p
6
+
3
p
fy exp—nding expression ‡e h—veX
(p2
+ 3)6p − p2
(3p − r) − 18(3p − r) ≥ 0
TV
⇐⇒ 3p3
+ p2
r − 36p + 18r ≥ 0
prom the illEknown inequ—lityD the third degree ƒ™hur9s inequ—lity st—tesX
p3
− 4pq + 9r ≥ 0 ⇐⇒ p3
− 12p + 9r ≥ 0
‡e h—veX
⇐⇒ 3p3
+ p2
r − 36p + 18r ≥ 0
⇐⇒ 3(p3
− 12p + 9r) + r(p2
− 9) ≥ 0
yn the other h—ndD ‡e h—veX
r(p2
− 9) ≥ 0 ⇐⇒ (a − b)2
+ (b − c)2
+ (c − a)2
≥ 0
VWF if x, y, z —re nonneg—tive re—l num˜ers su™h th—t x + y + z = 3D then
4(
√
x +
√
y +
√
z) + 15 ≤ 9(
x + y
2
+
y + z
2
+
z + x
2
)
SolutionX „he inequ—lity9s true when x = 3, y = z = 0F if no two of x, y, z —re HD set x = a2
et™F it ˜e™omes
8(a + b + c) + 10 3a2 + 3b2 + 3c2 ≤ 9 2a2 + 2b2 + 2b2 + 2c2 + 2c2 + 2a2
⇐⇒ 10 3a2 + 3b2 + 3c2 − (a + b + c) ≤ 9
cyc
2a2 + 2b2 − (a + b)
⇐⇒ 10
cyc
(a − b)2
a + b + c +
√
3a2 + 3b2 + 3c2
≤ 9
cyc
(a − b)2
a + b +
√
2a2 + 2b2
⇐⇒
cyc
9
a + b +
√
2a2 + 2b2
−
10
a + b + c +
√
3a2 + 3b2 + 3c2
(a − b)2
≥ 0
xow e—™h term is nonneg—tiveD for in f—™tD
9(a + b + 3a2 + 3b2) ≥ 10(a + b + 2a2 + 2b2)
˜e™—use
9
√
3
√
2
− 10 2a2 + 2b2 > 2a2 + 2b2 ≥ a + b
WHF
if a, b, c —re nonneg—tive re—l num˜ers su™h th—t a + b + c = 3D then
1
4a2 + b2 + c2
+
1
4b2 + c2 + a2
+
1
4c2 + a2 + b2
≤
1
2
SolutionX
1
4a2 + b2 + c2
+
1
4b2 + c2 + a2
+
1
4c2 + a2 + b2
≤
1
2
⇔
⇔
sym
(a6
− 4a5
b + 13a4
b2
− 2a4
bc − 6a3
b3
− 12a3
b2
c + 10a2
b2
c2
) ≥ 0 ⇔
⇔
cyc
(a − b)2
(2c4
+ 2(a2
− 4ab + b2
)c2
+ a4
− 2a3
b + 4a2
b2
− 2ab3
+ b4
) ≥ 0,
whi™h true ˜e™—use
(a2
− 4ab + b2
)2
− 2(a4
− 2a3
b + 4a2
b2
− 2ab3
+ b4
) =
TW
= −(a − b)2
(a2
+ 6ab + b2
) ≤ 0.
WIF
vet —D˜D™ ˜e nonneg—tive re—l num˜ers su™h th—ta + b + c = 3. €rove th—t
a2
b
4 − bc
+
b2
c
4 − ca
+
c2
a
4 − ab
≤ 1.
SolutionF
ƒin™e
4a2
b
4 − bc
= a2
b +
a2
b2
c
4 − bc
the inequ—lity ™—n ˜e written —s
abc
ab
4 − bc
≤ 4 − a2
b.
…sing the illEknown inequ—lity a2
b + b2
c + c2
a + abc ≤ 4D ‡e get
4 − (a2
b + b2
c + c2
a) ≥ abc,
—nd hen™eD it suffi™es to prove th—t
abc
ab
4 − bc
≤ abc,
or equiv—lentlyD
ab
4 − bc
+
bc
4 − ca
+
ca
4 − ab
≤ 1.
ƒin™e
ab + bc + ca ≤
(a + b + c)2
3
= 3,
‡e get
ab
4 − bc
≤
ab
4
3
(ab + bc + ca) − bc
=
3ab
4ab + bc + 4ca
.
„hereforeD it is enough to ™he™k th—t
x
4x + 4y + z
+
y
4y + 4z + x
+
z
4z + 4x + y
≤
1
3
,
where x = ab, y = ca —nd z = bcF „his is — illEknown inequ—lityF WPF
if a, b, c —re nonneg—tive re—l num˜ersD then
a3
+ b3
+ c3
+ 12abc ≤ a2
a2 + 24bc + b2
b2 + 24ca + c2
c2 + 24ab
SolutionX
a2
+ 24bc 7 a2
+ b2
+ c2
+ 8(bc + ca + ab)
2
− 7a3
+ 8a2
(b + c) + 7a b2
+ c2
+ 92abc + 48bc(b + c)
2
= 24bc (b − c)2
109a2
+ 77ab + 77ac + 49b2
+ 89bc + 49c2
+(b + c − 2a)2
(25bc + 7ab + 7ca) ≥ 0 =⇒
a2
a2 + 24bc
≥
a2
[7a3
+ 8a2
(b + c) + 7a(b2
+ c2
) + 92abc + 48bc(b + c)]
7(a2 + b2 + c2) + 8(bc + ca + ab)
UH
= a3
+ b3
+ c3
+ 12abc.
WQF where a, b, c, d —re nonneg—tive re—l num˜ersF€rove the inequ—lityX
a 9a2 + 7b2 + b 9b2 + 7c2 + c 9c2 + 7d2 + d 9d2 + 7a2 ≥ (a + b + c + d)2
SolutionX
fy g—u™hyƒ™hw—rzD ‡e h—ve
4 a 9a2 + 7b2 ≥ a(9a + 7b)
it suffi™es to prove th—t
9(a2
+b2
+c2
+d2
)+7(a+c)(b+d) ≥ 4(a+b+c+d)2
= 4(a+c)2
+4(b+d)2
+8(a+c)(b+d)
⇔ 9(a2
+ b2
+ c2
+ d2
) ≥ 4(a + c)2
+ 4(b + d)2
+ (a + c)(b + d)
whi™h is true ˜e™—use
a2
+ b2
+ c2
+ d2
≥ (a + c)(b + d)
2(a2
+ b2
+ c2
+ d2
) = 2(a2
+ c2
) + 2(b2
+ d2
) ≥ (a + c)2
+ (b + d)2
WRF vet a, b, c ˜e positive F €rove th—tF
3a2
7a2 + 5(b + c)2
≤ 1 ≤
a2
a2 + 2(b + c)2
SolutionX
„he right h—nd is trivi—l ˜y the rolder inequ—lity sin™e

 a
a2 + 2 (b + c)
2


2
a a2
+ 2 (b + c)
2
≥ a
3
end ( a)
3
≥ a a2
+ 2 (b + c)
2
⇔ ab (a + b) ≥ 6abcF por the left h—nd ˜y the
g—u™hy ƒ™hw—rz inequ—lity ‡e h—ve

 a
7a2 + 5 (b + c)
2


2
≤ a
a
7a2 + 5 (b + c)
2
essume a + b + c = 3D denote ab + bc + ca = 9−q2
3 , r = abc then ‡e will prove
a
12a2 − 30a + 45
≤
1
9
⇔ f (r) = 48r2
+ 222 + 52q2
r + 20q4
+ 75q2
− 270 ≥ 0
‡e h—ve
r ≥ max 0,
(3 + q)
2
(3 − 2q)
27
„hereforD if
q ≥
3
2
UI
then get r ≥ 0 —nd
f (r) ≥ f (0) = 20 q −
3
2
q +
3
2
q2
+ 6 ≥ 0
if q ≤ 3
2 then get r ≥ (3+q)2
(3−2q)
27 —nd
f (r) ≥ f
(3 + q)
2
(3 − 2q)
27
=
q2
(2q − 3) 96q3
− 396q2
+ 2322q − 5103
729
≥ 0
‡e h—ve doneF iqu—lity holds if —n only if a = b = c or a = b, c = 0 or —ny ™y™li™ permut—E
tionsF
WSF
if a, b, c, d, e —re positive re—l num˜ers su™h th—t a + b + c + d + e = 5D then
1
a
+
1
b
+
1
c
+
1
d
+
1
e
+
20
a2 + b2 + c2 + d2 + e2
≥ 9
SolutionD sym f(a, b) me—ns f(a, b)+f(a, c)+f(a, d)+f(a, e)+f(b, c)+f(b, d) +f(b, e)+
f(c, d) + f(c, e) + f(d, e)F ‡e will firstly rewrite the inequ—lity —s
1
a
+
1
b
+
1
c
+
1
d
+
1
e
−
25
a + b + c + d + e
≥ 4 −
4(a + b + c + d + e)2
5(a2 + b2 + c2 + d2 + e2)
.
…sing the identities
(a + b + c + d + e)
1
a
+
1
b
+
1
c
+
1
d
+
1
e
− 25 =
sym
(a − b)2
ab
—nd 5(a2
+ b2
+ c2
+ d2
+ e2
) − (a + b + c + d + e)2
= sym(a − b)2
‡e ™—n rewrite —g—in
the inequ—lity —s
1
a + b + c + d + e sym
(a − b)2
ab
≥
4
5
×
sym(a − b)2
a2 + b2 + c2 + d2 + e2
or sym Sab(a − b)2
≥ 0 where
Sxy =
1
xy
−
4
a2 + b2 + c2 + d2 + e2
for —ll x, y ∈ {a, b, c, d, e}F essume th—t a ≥ b ≥ c ≥ d ≥ e > 0F ‡e will show th—t
Sbc + Sbd ≥ 0 —nd Sab + Sac + Sad + Sae ≥ 0F indeedD ‡e h—ve
Sbc + Sbd =
1
bc
+
1
bd
−
8
a2 + b2 + c2 + d2 + e2
>
1
bc
+
1
bd
−
8
b2 + b2 + c2 + d2
≥
1
bc
+
1
bd
−
8
2bc + 2bd
≥ 0
—nd
Sab+Sac+Sad+Sae =
1
ab
+
1
ac
+
1
ad
+
1
ae
−
16
a2 + b2 + c2 + d2 + e2
≥
16
a(b + c + d + e)
−
16
a2 + 1
4 (b + c + d + e)2
≥ 0.
ren™eD with noti™e th—t
Sbd ≥ Sbc —nd Sae ≥ Sad ≥ Sac ≥ Sab
UP
‡e h—ve Sbd ≥ 0 —nd Sae ≥ 0, Sae + Sad ≥ 0, Sae + Sad + Sac ≥ 0F
„husD
Sbd(b − d)2
+ Sbc(b − c)2
≥ (Sbd + Sbc)(b − c)2
≥ 0(1)
—nd
Sae(a−e)2
+Sad(a−d)2
+Sac(a−c)2
+Sab(a−b)2
≥ (Sae+Sad)(a−d)2
+Sac(a−c)2
+Sab(a−b)2
≥ (Sae + Sad + Sac)(a − c)2
+ Sab(a − b)2
≥ (Sae + Sad + Sac + Sab)(a − b)2
≥ 0(2)
yn the other h—ndD Sbe ≥ Sbd ≥ 0 —nd Sde ≥ Sce ≥ Scd ≥ Sbd ≥ 0(3)F
„hereforeD from @IAD @PA —nd @QA ‡e get sym Sab(a − b)2
≥ 0.
iqu—lity o™™urs when a = b = c = d = e or a = 2b = 2c = 2d = 2eF
WTF
vet a, b, c ˜e nonneg—tive re—l num˜ersF €rove th—t
1 +
3abc
a2b + b2c + c2a
≥
2(ab + bc + ca)
a2 + b2 + c2
SolutionX
‡e ™—n prove it —s followX
‚ewriting the inequ—lity —s
3abc
a2b + b2c + c2a
≥
2(ab + bc + ca) − a2
− b2
− c2
a2 + b2 + c2
if 2(ab + bc + ca) ≤ a2
+ b2
+ c2
D it is trivi—lF
if 2(ab + bc + ca) ≥ a2
+ b2
+ c2
D —pplying ƒ™hur9s inequ—lityX
3abc ≥
(a + b + c)[2(ab + bc + ca) − a2
− b2
− c2
]
3
it suffi™es to show th—t
(a + b + c)[2(ab + bc + ca) − a2
− b2
− c2
]
3(a2b + b2c + c2a)
≥
2(ab + bc + ca) − a2
− b2
− c2
a2 + b2 + c2
(a + b + c)(a2
+ b2
+ c2
) ≥ 3(a2
b + b2
c + c2
a)
b(a − b)2
+ c(b − c)2
+ a(c − a)2
≥ 0
@„rueA
WUF
vet a, b, c ˜e positive re—l num˜ers su™h th—t abc = 1F €rove th—t
1
a
+
1
b
+
1
c
+
6
a + b + c
≥ 5.
SolutionX
IFFFF‡vyq —ssume a ≥ b ≥ cF
vet
f(a, b, c) =
1
a
+
1
b
+
1
c
+
6
a + b + c
f(a, b, c) ≥ f(a,
√
bc,
√
bc)
<=>
(
√
b −
√
c)2
bc(a + b + c)(a + 2
√
bc)
((a + b + c)(a + 2
√
bc) − 6bc) ≥ 0 ⇔
UQ
(a + b + c)(a + 2
√
bc) ≥ 6bc.
es
a ≥
b + c
2
≥
√
bc
so
(a + b + c)(a + 2
√
bc) ≥ 9bc ≥ 6bc
hen™e the —˜ove inequ—lity is trueF
f(
1
x2
, x, x) ≥ 5 ⇔
(x − 1)2
(2x4
+ 4x3
− 4x2
− x + 2) ≥ 0.
es 2x4
+ 4x3
− 4x2
− x + 2 > 0 if x > 0D so the —˜ove inequ—lity is trueF
„herefore
f(a, b, c) ≥ f(a,
√
bc,
√
bc) = f(
1
bc
,
√
bc,
√
bc) ≥ 5.
ab FiFh
PFFFFFFFFF
essume th—t a ≥ b, cF ‡rite x =
√
a, y = b
c F „hen x ≥ 1 —nd the inequ—lity
(ab + bc + ca)(a + b + c) + 6 ≥ 5(a + b + c)
˜e™omes
x3
(y + y−1
) − 5x2
+ (y2
+ 9 + y−2
) − 5x−1
(y + y−1
) + x−3
(y + y−1
) ≥ 0
„his ™—n ˜e seper—ted —s
2x3
− 5x2
+ 11 − 10x−1
+ 2x−3
≥ 0
—nd
x3
(y + y−1
− 2) + (y2
+ y−2
− 2) − 5x−1
(y + y−1
− 2) + x−3
(y + y−1
− 2) ≥ 0
„he first one is e—syF e˜out the se™ond oneD
xote th—t x3
+ x−3
≥ 2 ≥ 2x−1
—nd (y2
+ y−2
− 2) ≥ 3(y1
+ y−1
− 2) ≥ 3x−1
(y1
+ y−1
− 2)
sin™e
y2
− 3y + 4 − 3y−1
+ y−2
= (y − 1)2
(y − 1 + y−1
)
QFFFFFFFFF
vem— of †—ile girto—je
(a + b) (b + c) (c + a) + 7 ≥ 5 (a + b + c)
@g—n e—sy prove ˜y w†A
fut
(a + b) (b + c) (c + a) = a2
b + a2
c + b2
c + b2
a + c2
a + c2
b + 2abc
= a2
b + a2
c + b2
c + b2
a + c2
a + c2
b + 3abc − abc
UR
= (a + b + c) (bc + ca + ab) − abc = (a + b + c) (bc + ca + ab) − 1
where ‡e h—ve used th—t —˜™ a I in the l—st step of our ™—l™ul—tionF „husD ‡e h—ve
((a + b + c) (bc + ca + ab) − 1) + 7 ≥ 5 (a + b + c)
Y in other wordsD
(a + b + c) (bc + ca + ab) + 6 ≥ 5 (a + b + c)
…pon division ˜y — C ˜ C ™D this ˜e™omes
(bc + ca + ab) +
6
a + b + c
≥ 5
pin—llyD sin™e abc = 1D
‡e h—ve bc = 1
a D ca = 1
b —nd ab = 1
c D —nd thus ‡e get
1
a
+
1
b
+
1
c
+
6
a + b + c
≥ 5
WVF
vet a, b, c > 0 —nd with —ll k ≥ −3/2 F €rove the inequ—lityX
a3
+ (k + 1)abc
b2 + kbc + c2
≥ a + b + c
SolutionX
yur inequ—lity is equiv—lent to
a(a2
+ bc − b2
− c2
)
b2 + kbc + c2
+
b(b2
+ ca − c2
− a2
)
c2 + kca + a2
+
c(c2
+ ab − a2
− b2
)
a2 + kab + b2
≥ 0,
(a2
−b2
)
a
b2 + kbc + c2
−
b
a2 + kac + c2
+c
a(b − c)
b2 + kbc + c2
+
b(a − c)
a2 + kac + c2
+
c2
+ ab − a2
− b2
a2 + kab + b2
≥ 0.
prom nowD ‡e see th—t
a
b2 + kbc + c2
−
b
a2 + kac + c2
=
(a − b)(a2
+ b2
+ c2
+ ab + kac + kbc)
(b2 + kbc + c2)(a2 + kac + c2)
,
c2
+ ab − a2
− b2
a2 + kab + b2
=
(c − a)(c − b) − a(b − c) − b(a − c) − (a − b)2
a2 + kab + b2
,
a(b − c)
b2 + kbc + c2
−
a(b − c)
a2 + kab + b2
=
a(a − c)(b − c)(a + c + kb)
(a2 + kab + b2)(b2 + kbc + c2)
,
b(a − c)
a2 + kac + c2
−
b(a − c)
a2 + kab + b2
=
a(a − c)(b − c)(b + c + ka)
(a2 + kab + b2)(a2 + kac + c2)
.
„hereforeD the inequ—lity ™—n ˜e rewritten —s
A(a − b)2
+
c(a − c)(b − c)
a2 + kab + b2
B ≥ 0,
where
A =
(a + b)(a2
+ b2
+ c2
+ ab + kac + kbc)
(a2 + kac + c2)(b2 + kbc + c2)
−
c
a2 + kab + b2
,
—nd
B =
a(a + c + kb)
b2 + kbc + c2
+
b(b + c + ka)
a2 + kac + c2
+ 1.
US
567 nice and_hard_inequality
567 nice and_hard_inequality
567 nice and_hard_inequality
567 nice and_hard_inequality
567 nice and_hard_inequality
567 nice and_hard_inequality
567 nice and_hard_inequality
567 nice and_hard_inequality
567 nice and_hard_inequality
567 nice and_hard_inequality
567 nice and_hard_inequality
567 nice and_hard_inequality
567 nice and_hard_inequality
567 nice and_hard_inequality
567 nice and_hard_inequality
567 nice and_hard_inequality
567 nice and_hard_inequality
567 nice and_hard_inequality
567 nice and_hard_inequality
567 nice and_hard_inequality
567 nice and_hard_inequality
567 nice and_hard_inequality
567 nice and_hard_inequality
567 nice and_hard_inequality
567 nice and_hard_inequality
567 nice and_hard_inequality
567 nice and_hard_inequality
567 nice and_hard_inequality
567 nice and_hard_inequality
567 nice and_hard_inequality
567 nice and_hard_inequality
567 nice and_hard_inequality
567 nice and_hard_inequality
567 nice and_hard_inequality
567 nice and_hard_inequality
567 nice and_hard_inequality
567 nice and_hard_inequality
567 nice and_hard_inequality
567 nice and_hard_inequality
567 nice and_hard_inequality
567 nice and_hard_inequality
567 nice and_hard_inequality
567 nice and_hard_inequality
567 nice and_hard_inequality
567 nice and_hard_inequality
567 nice and_hard_inequality
567 nice and_hard_inequality
567 nice and_hard_inequality
567 nice and_hard_inequality
567 nice and_hard_inequality
567 nice and_hard_inequality
567 nice and_hard_inequality
567 nice and_hard_inequality
567 nice and_hard_inequality
567 nice and_hard_inequality
567 nice and_hard_inequality
567 nice and_hard_inequality
567 nice and_hard_inequality
567 nice and_hard_inequality
567 nice and_hard_inequality
567 nice and_hard_inequality
567 nice and_hard_inequality
567 nice and_hard_inequality
567 nice and_hard_inequality
567 nice and_hard_inequality
567 nice and_hard_inequality
567 nice and_hard_inequality
567 nice and_hard_inequality
567 nice and_hard_inequality
567 nice and_hard_inequality
567 nice and_hard_inequality
567 nice and_hard_inequality
567 nice and_hard_inequality
567 nice and_hard_inequality
567 nice and_hard_inequality
567 nice and_hard_inequality
567 nice and_hard_inequality
567 nice and_hard_inequality
567 nice and_hard_inequality
567 nice and_hard_inequality
567 nice and_hard_inequality
567 nice and_hard_inequality
567 nice and_hard_inequality
567 nice and_hard_inequality
567 nice and_hard_inequality
567 nice and_hard_inequality
567 nice and_hard_inequality
567 nice and_hard_inequality
567 nice and_hard_inequality
567 nice and_hard_inequality
567 nice and_hard_inequality
567 nice and_hard_inequality
567 nice and_hard_inequality
567 nice and_hard_inequality
567 nice and_hard_inequality
567 nice and_hard_inequality
567 nice and_hard_inequality
567 nice and_hard_inequality
567 nice and_hard_inequality
567 nice and_hard_inequality
567 nice and_hard_inequality
567 nice and_hard_inequality
567 nice and_hard_inequality
567 nice and_hard_inequality
567 nice and_hard_inequality
567 nice and_hard_inequality
567 nice and_hard_inequality
567 nice and_hard_inequality
567 nice and_hard_inequality
567 nice and_hard_inequality
567 nice and_hard_inequality
567 nice and_hard_inequality
567 nice and_hard_inequality
567 nice and_hard_inequality
567 nice and_hard_inequality
567 nice and_hard_inequality
567 nice and_hard_inequality
567 nice and_hard_inequality
567 nice and_hard_inequality
567 nice and_hard_inequality
567 nice and_hard_inequality
567 nice and_hard_inequality
567 nice and_hard_inequality
567 nice and_hard_inequality
567 nice and_hard_inequality
567 nice and_hard_inequality
567 nice and_hard_inequality
567 nice and_hard_inequality
567 nice and_hard_inequality
567 nice and_hard_inequality
567 nice and_hard_inequality
567 nice and_hard_inequality
567 nice and_hard_inequality
567 nice and_hard_inequality
567 nice and_hard_inequality
567 nice and_hard_inequality
567 nice and_hard_inequality
567 nice and_hard_inequality
567 nice and_hard_inequality
567 nice and_hard_inequality
567 nice and_hard_inequality
567 nice and_hard_inequality
567 nice and_hard_inequality
567 nice and_hard_inequality
567 nice and_hard_inequality
567 nice and_hard_inequality
567 nice and_hard_inequality
567 nice and_hard_inequality
567 nice and_hard_inequality
567 nice and_hard_inequality
567 nice and_hard_inequality
567 nice and_hard_inequality
567 nice and_hard_inequality
567 nice and_hard_inequality
567 nice and_hard_inequality
567 nice and_hard_inequality
567 nice and_hard_inequality
567 nice and_hard_inequality
567 nice and_hard_inequality
567 nice and_hard_inequality
567 nice and_hard_inequality
567 nice and_hard_inequality
567 nice and_hard_inequality
567 nice and_hard_inequality
567 nice and_hard_inequality
567 nice and_hard_inequality
567 nice and_hard_inequality
567 nice and_hard_inequality
567 nice and_hard_inequality
567 nice and_hard_inequality
567 nice and_hard_inequality
567 nice and_hard_inequality
567 nice and_hard_inequality
567 nice and_hard_inequality
567 nice and_hard_inequality
567 nice and_hard_inequality
567 nice and_hard_inequality
567 nice and_hard_inequality
567 nice and_hard_inequality
567 nice and_hard_inequality
567 nice and_hard_inequality
567 nice and_hard_inequality
567 nice and_hard_inequality
567 nice and_hard_inequality
567 nice and_hard_inequality
567 nice and_hard_inequality
567 nice and_hard_inequality
567 nice and_hard_inequality
567 nice and_hard_inequality
567 nice and_hard_inequality
567 nice and_hard_inequality
567 nice and_hard_inequality
567 nice and_hard_inequality
567 nice and_hard_inequality
567 nice and_hard_inequality
567 nice and_hard_inequality
567 nice and_hard_inequality
567 nice and_hard_inequality
567 nice and_hard_inequality
567 nice and_hard_inequality
567 nice and_hard_inequality
567 nice and_hard_inequality
567 nice and_hard_inequality
567 nice and_hard_inequality
567 nice and_hard_inequality
567 nice and_hard_inequality
567 nice and_hard_inequality
567 nice and_hard_inequality
567 nice and_hard_inequality
567 nice and_hard_inequality
567 nice and_hard_inequality
567 nice and_hard_inequality
567 nice and_hard_inequality
567 nice and_hard_inequality
567 nice and_hard_inequality
567 nice and_hard_inequality
567 nice and_hard_inequality
567 nice and_hard_inequality
567 nice and_hard_inequality
567 nice and_hard_inequality
567 nice and_hard_inequality
567 nice and_hard_inequality
567 nice and_hard_inequality
567 nice and_hard_inequality
567 nice and_hard_inequality
567 nice and_hard_inequality
567 nice and_hard_inequality
567 nice and_hard_inequality
567 nice and_hard_inequality
567 nice and_hard_inequality
567 nice and_hard_inequality
567 nice and_hard_inequality
567 nice and_hard_inequality
567 nice and_hard_inequality
567 nice and_hard_inequality
567 nice and_hard_inequality
567 nice and_hard_inequality
567 nice and_hard_inequality
567 nice and_hard_inequality
567 nice and_hard_inequality
567 nice and_hard_inequality
567 nice and_hard_inequality
567 nice and_hard_inequality
567 nice and_hard_inequality
567 nice and_hard_inequality
567 nice and_hard_inequality
567 nice and_hard_inequality
567 nice and_hard_inequality
567 nice and_hard_inequality
567 nice and_hard_inequality
567 nice and_hard_inequality
567 nice and_hard_inequality
567 nice and_hard_inequality
567 nice and_hard_inequality
567 nice and_hard_inequality
567 nice and_hard_inequality
567 nice and_hard_inequality
567 nice and_hard_inequality
567 nice and_hard_inequality
567 nice and_hard_inequality
567 nice and_hard_inequality
567 nice and_hard_inequality
567 nice and_hard_inequality
567 nice and_hard_inequality
567 nice and_hard_inequality
567 nice and_hard_inequality
567 nice and_hard_inequality
567 nice and_hard_inequality
567 nice and_hard_inequality
567 nice and_hard_inequality
567 nice and_hard_inequality
567 nice and_hard_inequality
567 nice and_hard_inequality
567 nice and_hard_inequality
567 nice and_hard_inequality
567 nice and_hard_inequality
567 nice and_hard_inequality
567 nice and_hard_inequality
567 nice and_hard_inequality
567 nice and_hard_inequality
567 nice and_hard_inequality
567 nice and_hard_inequality
567 nice and_hard_inequality
567 nice and_hard_inequality
567 nice and_hard_inequality
567 nice and_hard_inequality
567 nice and_hard_inequality
567 nice and_hard_inequality
567 nice and_hard_inequality
567 nice and_hard_inequality

More Related Content

What's hot

Solucion de problemas de ecuaciones difrenciales hasta 19
Solucion de problemas de ecuaciones difrenciales hasta 19Solucion de problemas de ecuaciones difrenciales hasta 19
Solucion de problemas de ecuaciones difrenciales hasta 19
JAVIERTELLOCAMPOS
 
Solo edo hasta 20
Solo edo hasta 20Solo edo hasta 20
Solo edo hasta 20
JAVIERTELLOCAMPOS
 
Single page-integral-table
Single page-integral-tableSingle page-integral-table
Single page-integral-table
Monique Anderson
 
Math resources trigonometric_formulas
Math resources trigonometric_formulasMath resources trigonometric_formulas
Math resources trigonometric_formulas
Er Deepak Sharma
 
Multiple Choice Questions_Successive Differentiation (CALCULUS)
Multiple Choice Questions_Successive Differentiation (CALCULUS)Multiple Choice Questions_Successive Differentiation (CALCULUS)
Multiple Choice Questions_Successive Differentiation (CALCULUS)
sanjay gupta
 
System dynamics 3rd edition palm solutions manual
System dynamics 3rd edition palm solutions manualSystem dynamics 3rd edition palm solutions manual
System dynamics 3rd edition palm solutions manual
SextonMales
 
F.Y.B.Sc(2013 pattern) Old Question Papers:Dr.Kshirsagar
F.Y.B.Sc(2013 pattern) Old Question Papers:Dr.KshirsagarF.Y.B.Sc(2013 pattern) Old Question Papers:Dr.Kshirsagar
F.Y.B.Sc(2013 pattern) Old Question Papers:Dr.Kshirsagar
Dr.Ravindra Kshirsagar P.G Department of Zoology Modern College Ganeshkhind Pune
 
Integral table
Integral tableIntegral table
Integral table
zzzubair
 
Integral table
Integral tableIntegral table
Integral table
Ankitcos0
 
Factorise quadratics lesson 1
Factorise quadratics lesson 1Factorise quadratics lesson 1
Factorise quadratics lesson 1Angela Phillips
 
12 cbse-maths-2014-solution set 1
12 cbse-maths-2014-solution set 1 12 cbse-maths-2014-solution set 1
12 cbse-maths-2014-solution set 1
vandna123
 
Integral table
Integral tableIntegral table
Integral table
bags07
 
Paper mathematics
Paper mathematics  Paper mathematics
Paper mathematics
Dr.Faisal Ahmed Yousafzai
 

What's hot (16)

Solucion de problemas de ecuaciones difrenciales hasta 19
Solucion de problemas de ecuaciones difrenciales hasta 19Solucion de problemas de ecuaciones difrenciales hasta 19
Solucion de problemas de ecuaciones difrenciales hasta 19
 
Solo edo hasta 20
Solo edo hasta 20Solo edo hasta 20
Solo edo hasta 20
 
Single page-integral-table
Single page-integral-tableSingle page-integral-table
Single page-integral-table
 
Single page-integral-table
Single page-integral-table Single page-integral-table
Single page-integral-table
 
Math resources trigonometric_formulas
Math resources trigonometric_formulasMath resources trigonometric_formulas
Math resources trigonometric_formulas
 
Multiple Choice Questions_Successive Differentiation (CALCULUS)
Multiple Choice Questions_Successive Differentiation (CALCULUS)Multiple Choice Questions_Successive Differentiation (CALCULUS)
Multiple Choice Questions_Successive Differentiation (CALCULUS)
 
System dynamics 3rd edition palm solutions manual
System dynamics 3rd edition palm solutions manualSystem dynamics 3rd edition palm solutions manual
System dynamics 3rd edition palm solutions manual
 
F.Y.B.Sc(2013 pattern) Old Question Papers:Dr.Kshirsagar
F.Y.B.Sc(2013 pattern) Old Question Papers:Dr.KshirsagarF.Y.B.Sc(2013 pattern) Old Question Papers:Dr.Kshirsagar
F.Y.B.Sc(2013 pattern) Old Question Papers:Dr.Kshirsagar
 
Integral table
Integral tableIntegral table
Integral table
 
3vherleidenenbreuken
3vherleidenenbreuken3vherleidenenbreuken
3vherleidenenbreuken
 
Integral table
Integral tableIntegral table
Integral table
 
Factorise quadratics lesson 1
Factorise quadratics lesson 1Factorise quadratics lesson 1
Factorise quadratics lesson 1
 
12 cbse-maths-2014-solution set 1
12 cbse-maths-2014-solution set 1 12 cbse-maths-2014-solution set 1
12 cbse-maths-2014-solution set 1
 
Integral table
Integral tableIntegral table
Integral table
 
Paper mathematics
Paper mathematics  Paper mathematics
Paper mathematics
 
Integral table
Integral tableIntegral table
Integral table
 

Similar to 567 nice and_hard_inequality

100 Inequalities Problems
100 Inequalities Problems100 Inequalities Problems
100 Inequalities Problems
Kimberly Pulley
 
Algebra formulas
Algebra formulas Algebra formulas
Algebra formulas
Matthew McKenzie
 
Productos notables
Productos notablesProductos notables
Productos notables
PROFTEBA
 
Important maths formulas
Important maths formulasImportant maths formulas
Important maths formulaskrsraju
 
3d. Pedagogy of Mathematics (Part II) - Algebra (Ex 3.4)
3d. Pedagogy of Mathematics (Part II) - Algebra (Ex 3.4)3d. Pedagogy of Mathematics (Part II) - Algebra (Ex 3.4)
3d. Pedagogy of Mathematics (Part II) - Algebra (Ex 3.4)
Dr. I. Uma Maheswari Maheswari
 
The sum of the triangle sides lengths reciprocals vs a cyclic sum of a specif...
The sum of the triangle sides lengths reciprocals vs a cyclic sum of a specif...The sum of the triangle sides lengths reciprocals vs a cyclic sum of a specif...
The sum of the triangle sides lengths reciprocals vs a cyclic sum of a specif...
Mikołaj Hajduk
 
xy2.5 3.0 3.5-1.0 6 7 81.0 0 1 23.0 -6 -5 .docx
xy2.5 3.0 3.5-1.0 6 7 81.0 0 1 23.0 -6 -5 .docxxy2.5 3.0 3.5-1.0 6 7 81.0 0 1 23.0 -6 -5 .docx
xy2.5 3.0 3.5-1.0 6 7 81.0 0 1 23.0 -6 -5 .docx
ericbrooks84875
 
log and algebra rules
log and algebra ruleslog and algebra rules
log and algebra rules
sriam99
 
Pembahasan Soal Matematika Kelas 10 Semester 1
Pembahasan Soal Matematika Kelas 10 Semester 1Pembahasan Soal Matematika Kelas 10 Semester 1
Pembahasan Soal Matematika Kelas 10 Semester 1
Pillar Adhikusumah
 
cl 9 polynomial.pdf
cl 9 polynomial.pdfcl 9 polynomial.pdf
cl 9 polynomial.pdf
TanishqAgarwal27
 
Class XII CBSE Mathematics Sample question paper with solution
Class XII CBSE Mathematics Sample question paper with solutionClass XII CBSE Mathematics Sample question paper with solution
Class XII CBSE Mathematics Sample question paper with solution
Pratima Nayak ,Kendriya Vidyalaya Sangathan
 
Maths formulae
Maths formulaeMaths formulae
Maths formulae
chirag patil
 
Bt0063 mathematics fot it
Bt0063 mathematics fot itBt0063 mathematics fot it
Bt0063 mathematics fot itnimbalkarks
 
Presentation1
Presentation1Presentation1
Presentation1
leemessika
 
Presentation1[1]
Presentation1[1]Presentation1[1]
Presentation1[1]
efratmarco
 
567 bdt hay va kho
567 bdt hay va kho567 bdt hay va kho
567 bdt hay va khovannhonbclt
 

Similar to 567 nice and_hard_inequality (20)

100 Inequalities Problems
100 Inequalities Problems100 Inequalities Problems
100 Inequalities Problems
 
Algebra formulas
Algebra formulas Algebra formulas
Algebra formulas
 
number theory chandramowliswaran theorem
number theory chandramowliswaran theoremnumber theory chandramowliswaran theorem
number theory chandramowliswaran theorem
 
Appendex
AppendexAppendex
Appendex
 
Productos notables
Productos notablesProductos notables
Productos notables
 
Important maths formulas
Important maths formulasImportant maths formulas
Important maths formulas
 
3d. Pedagogy of Mathematics (Part II) - Algebra (Ex 3.4)
3d. Pedagogy of Mathematics (Part II) - Algebra (Ex 3.4)3d. Pedagogy of Mathematics (Part II) - Algebra (Ex 3.4)
3d. Pedagogy of Mathematics (Part II) - Algebra (Ex 3.4)
 
The sum of the triangle sides lengths reciprocals vs a cyclic sum of a specif...
The sum of the triangle sides lengths reciprocals vs a cyclic sum of a specif...The sum of the triangle sides lengths reciprocals vs a cyclic sum of a specif...
The sum of the triangle sides lengths reciprocals vs a cyclic sum of a specif...
 
Sect3 7
Sect3 7Sect3 7
Sect3 7
 
xy2.5 3.0 3.5-1.0 6 7 81.0 0 1 23.0 -6 -5 .docx
xy2.5 3.0 3.5-1.0 6 7 81.0 0 1 23.0 -6 -5 .docxxy2.5 3.0 3.5-1.0 6 7 81.0 0 1 23.0 -6 -5 .docx
xy2.5 3.0 3.5-1.0 6 7 81.0 0 1 23.0 -6 -5 .docx
 
log and algebra rules
log and algebra ruleslog and algebra rules
log and algebra rules
 
Unexpected ineq
Unexpected ineqUnexpected ineq
Unexpected ineq
 
Pembahasan Soal Matematika Kelas 10 Semester 1
Pembahasan Soal Matematika Kelas 10 Semester 1Pembahasan Soal Matematika Kelas 10 Semester 1
Pembahasan Soal Matematika Kelas 10 Semester 1
 
cl 9 polynomial.pdf
cl 9 polynomial.pdfcl 9 polynomial.pdf
cl 9 polynomial.pdf
 
Class XII CBSE Mathematics Sample question paper with solution
Class XII CBSE Mathematics Sample question paper with solutionClass XII CBSE Mathematics Sample question paper with solution
Class XII CBSE Mathematics Sample question paper with solution
 
Maths formulae
Maths formulaeMaths formulae
Maths formulae
 
Bt0063 mathematics fot it
Bt0063 mathematics fot itBt0063 mathematics fot it
Bt0063 mathematics fot it
 
Presentation1
Presentation1Presentation1
Presentation1
 
Presentation1[1]
Presentation1[1]Presentation1[1]
Presentation1[1]
 
567 bdt hay va kho
567 bdt hay va kho567 bdt hay va kho
567 bdt hay va kho
 

Recently uploaded

Chapter 3 - Islamic Banking Products and Services.pptx
Chapter 3 - Islamic Banking Products and Services.pptxChapter 3 - Islamic Banking Products and Services.pptx
Chapter 3 - Islamic Banking Products and Services.pptx
Mohd Adib Abd Muin, Senior Lecturer at Universiti Utara Malaysia
 
The French Revolution Class 9 Study Material pdf free download
The French Revolution Class 9 Study Material pdf free downloadThe French Revolution Class 9 Study Material pdf free download
The French Revolution Class 9 Study Material pdf free download
Vivekanand Anglo Vedic Academy
 
Instructions for Submissions thorugh G- Classroom.pptx
Instructions for Submissions thorugh G- Classroom.pptxInstructions for Submissions thorugh G- Classroom.pptx
Instructions for Submissions thorugh G- Classroom.pptx
Jheel Barad
 
Acetabularia Information For Class 9 .docx
Acetabularia Information For Class 9  .docxAcetabularia Information For Class 9  .docx
Acetabularia Information For Class 9 .docx
vaibhavrinwa19
 
The Challenger.pdf DNHS Official Publication
The Challenger.pdf DNHS Official PublicationThe Challenger.pdf DNHS Official Publication
The Challenger.pdf DNHS Official Publication
Delapenabediema
 
The Accursed House by Émile Gaboriau.pptx
The Accursed House by Émile Gaboriau.pptxThe Accursed House by Émile Gaboriau.pptx
The Accursed House by Émile Gaboriau.pptx
DhatriParmar
 
Unit 8 - Information and Communication Technology (Paper I).pdf
Unit 8 - Information and Communication Technology (Paper I).pdfUnit 8 - Information and Communication Technology (Paper I).pdf
Unit 8 - Information and Communication Technology (Paper I).pdf
Thiyagu K
 
Lapbook sobre os Regimes Totalitários.pdf
Lapbook sobre os Regimes Totalitários.pdfLapbook sobre os Regimes Totalitários.pdf
Lapbook sobre os Regimes Totalitários.pdf
Jean Carlos Nunes Paixão
 
Additional Benefits for Employee Website.pdf
Additional Benefits for Employee Website.pdfAdditional Benefits for Employee Website.pdf
Additional Benefits for Employee Website.pdf
joachimlavalley1
 
"Protectable subject matters, Protection in biotechnology, Protection of othe...
"Protectable subject matters, Protection in biotechnology, Protection of othe..."Protectable subject matters, Protection in biotechnology, Protection of othe...
"Protectable subject matters, Protection in biotechnology, Protection of othe...
SACHIN R KONDAGURI
 
Thesis Statement for students diagnonsed withADHD.ppt
Thesis Statement for students diagnonsed withADHD.pptThesis Statement for students diagnonsed withADHD.ppt
Thesis Statement for students diagnonsed withADHD.ppt
EverAndrsGuerraGuerr
 
The Roman Empire A Historical Colossus.pdf
The Roman Empire A Historical Colossus.pdfThe Roman Empire A Historical Colossus.pdf
The Roman Empire A Historical Colossus.pdf
kaushalkr1407
 
How to Make a Field invisible in Odoo 17
How to Make a Field invisible in Odoo 17How to Make a Field invisible in Odoo 17
How to Make a Field invisible in Odoo 17
Celine George
 
TESDA TM1 REVIEWER FOR NATIONAL ASSESSMENT WRITTEN AND ORAL QUESTIONS WITH A...
TESDA TM1 REVIEWER  FOR NATIONAL ASSESSMENT WRITTEN AND ORAL QUESTIONS WITH A...TESDA TM1 REVIEWER  FOR NATIONAL ASSESSMENT WRITTEN AND ORAL QUESTIONS WITH A...
TESDA TM1 REVIEWER FOR NATIONAL ASSESSMENT WRITTEN AND ORAL QUESTIONS WITH A...
EugeneSaldivar
 
Welcome to TechSoup New Member Orientation and Q&A (May 2024).pdf
Welcome to TechSoup   New Member Orientation and Q&A (May 2024).pdfWelcome to TechSoup   New Member Orientation and Q&A (May 2024).pdf
Welcome to TechSoup New Member Orientation and Q&A (May 2024).pdf
TechSoup
 
Honest Reviews of Tim Han LMA Course Program.pptx
Honest Reviews of Tim Han LMA Course Program.pptxHonest Reviews of Tim Han LMA Course Program.pptx
Honest Reviews of Tim Han LMA Course Program.pptx
timhan337
 
Supporting (UKRI) OA monographs at Salford.pptx
Supporting (UKRI) OA monographs at Salford.pptxSupporting (UKRI) OA monographs at Salford.pptx
Supporting (UKRI) OA monographs at Salford.pptx
Jisc
 
1.4 modern child centered education - mahatma gandhi-2.pptx
1.4 modern child centered education - mahatma gandhi-2.pptx1.4 modern child centered education - mahatma gandhi-2.pptx
1.4 modern child centered education - mahatma gandhi-2.pptx
JosvitaDsouza2
 
Francesca Gottschalk - How can education support child empowerment.pptx
Francesca Gottschalk - How can education support child empowerment.pptxFrancesca Gottschalk - How can education support child empowerment.pptx
Francesca Gottschalk - How can education support child empowerment.pptx
EduSkills OECD
 
CLASS 11 CBSE B.St Project AIDS TO TRADE - INSURANCE
CLASS 11 CBSE B.St Project AIDS TO TRADE - INSURANCECLASS 11 CBSE B.St Project AIDS TO TRADE - INSURANCE
CLASS 11 CBSE B.St Project AIDS TO TRADE - INSURANCE
BhavyaRajput3
 

Recently uploaded (20)

Chapter 3 - Islamic Banking Products and Services.pptx
Chapter 3 - Islamic Banking Products and Services.pptxChapter 3 - Islamic Banking Products and Services.pptx
Chapter 3 - Islamic Banking Products and Services.pptx
 
The French Revolution Class 9 Study Material pdf free download
The French Revolution Class 9 Study Material pdf free downloadThe French Revolution Class 9 Study Material pdf free download
The French Revolution Class 9 Study Material pdf free download
 
Instructions for Submissions thorugh G- Classroom.pptx
Instructions for Submissions thorugh G- Classroom.pptxInstructions for Submissions thorugh G- Classroom.pptx
Instructions for Submissions thorugh G- Classroom.pptx
 
Acetabularia Information For Class 9 .docx
Acetabularia Information For Class 9  .docxAcetabularia Information For Class 9  .docx
Acetabularia Information For Class 9 .docx
 
The Challenger.pdf DNHS Official Publication
The Challenger.pdf DNHS Official PublicationThe Challenger.pdf DNHS Official Publication
The Challenger.pdf DNHS Official Publication
 
The Accursed House by Émile Gaboriau.pptx
The Accursed House by Émile Gaboriau.pptxThe Accursed House by Émile Gaboriau.pptx
The Accursed House by Émile Gaboriau.pptx
 
Unit 8 - Information and Communication Technology (Paper I).pdf
Unit 8 - Information and Communication Technology (Paper I).pdfUnit 8 - Information and Communication Technology (Paper I).pdf
Unit 8 - Information and Communication Technology (Paper I).pdf
 
Lapbook sobre os Regimes Totalitários.pdf
Lapbook sobre os Regimes Totalitários.pdfLapbook sobre os Regimes Totalitários.pdf
Lapbook sobre os Regimes Totalitários.pdf
 
Additional Benefits for Employee Website.pdf
Additional Benefits for Employee Website.pdfAdditional Benefits for Employee Website.pdf
Additional Benefits for Employee Website.pdf
 
"Protectable subject matters, Protection in biotechnology, Protection of othe...
"Protectable subject matters, Protection in biotechnology, Protection of othe..."Protectable subject matters, Protection in biotechnology, Protection of othe...
"Protectable subject matters, Protection in biotechnology, Protection of othe...
 
Thesis Statement for students diagnonsed withADHD.ppt
Thesis Statement for students diagnonsed withADHD.pptThesis Statement for students diagnonsed withADHD.ppt
Thesis Statement for students diagnonsed withADHD.ppt
 
The Roman Empire A Historical Colossus.pdf
The Roman Empire A Historical Colossus.pdfThe Roman Empire A Historical Colossus.pdf
The Roman Empire A Historical Colossus.pdf
 
How to Make a Field invisible in Odoo 17
How to Make a Field invisible in Odoo 17How to Make a Field invisible in Odoo 17
How to Make a Field invisible in Odoo 17
 
TESDA TM1 REVIEWER FOR NATIONAL ASSESSMENT WRITTEN AND ORAL QUESTIONS WITH A...
TESDA TM1 REVIEWER  FOR NATIONAL ASSESSMENT WRITTEN AND ORAL QUESTIONS WITH A...TESDA TM1 REVIEWER  FOR NATIONAL ASSESSMENT WRITTEN AND ORAL QUESTIONS WITH A...
TESDA TM1 REVIEWER FOR NATIONAL ASSESSMENT WRITTEN AND ORAL QUESTIONS WITH A...
 
Welcome to TechSoup New Member Orientation and Q&A (May 2024).pdf
Welcome to TechSoup   New Member Orientation and Q&A (May 2024).pdfWelcome to TechSoup   New Member Orientation and Q&A (May 2024).pdf
Welcome to TechSoup New Member Orientation and Q&A (May 2024).pdf
 
Honest Reviews of Tim Han LMA Course Program.pptx
Honest Reviews of Tim Han LMA Course Program.pptxHonest Reviews of Tim Han LMA Course Program.pptx
Honest Reviews of Tim Han LMA Course Program.pptx
 
Supporting (UKRI) OA monographs at Salford.pptx
Supporting (UKRI) OA monographs at Salford.pptxSupporting (UKRI) OA monographs at Salford.pptx
Supporting (UKRI) OA monographs at Salford.pptx
 
1.4 modern child centered education - mahatma gandhi-2.pptx
1.4 modern child centered education - mahatma gandhi-2.pptx1.4 modern child centered education - mahatma gandhi-2.pptx
1.4 modern child centered education - mahatma gandhi-2.pptx
 
Francesca Gottschalk - How can education support child empowerment.pptx
Francesca Gottschalk - How can education support child empowerment.pptxFrancesca Gottschalk - How can education support child empowerment.pptx
Francesca Gottschalk - How can education support child empowerment.pptx
 
CLASS 11 CBSE B.St Project AIDS TO TRADE - INSURANCE
CLASS 11 CBSE B.St Project AIDS TO TRADE - INSURANCECLASS 11 CBSE B.St Project AIDS TO TRADE - INSURANCE
CLASS 11 CBSE B.St Project AIDS TO TRADE - INSURANCE
 

567 nice and_hard_inequality

  • 2. IF —A if a, b, c —re positive re—l num˜ersD then a b + b c + c a ≥ a2 + 1 b2 + 1 + b2 + 1 c2 + 1 + c2 + 1 a2 + 1 . ˜Avet a, b, c, d ˜e positive re—l num˜ersF€rove th—t a2 − bd b + 2c + d + b2 − ca c + 2d + a + c2 − db d + 2a + b + d2 − ac a + 2b + c ≥ 0. SolutionX —Afy g—u™hyEƒ™hw—rz9s inequ—lityD ‡e h—veX a2 + b2 (a2 + 1) (b2 + 1) ≥ a2 + b2 (ab + 1) = ab a2 + b2 + a2 + b2 ≥ ab a2 + b2 + 2 ⇒ a b + b a = a2 + b2 ab ≥ a2 + b2 + 2 (a2 + 1) (b2 + 1) = a2 + 1 b2 + 1 + b2 + 1 a2 + 1 fy ghe˜yshev9s inequ—lityD ‡e h—ve a2 b2 = a2 b2 + 1 + a2 b2 (b2 + 1) ≥ a2 b2 + 1 + b2 b2 (b2 + 1) = a2 + 1 b2 + 1 . „herefore 1 + a b 2 = 1 + 2 a b + b a + a2 b2 ≥ 1 + 2 a2 + 1 b2 + 1 + b2 + 1 a2 + 1 + a2 + 1 b2 + 1 = 1 + a2 + 1 b2 + 1 2 . „herefore a b + b c + c a ≥ a2 + 1 b2 + 1 + b2 + 1 c2 + 1 + c2 + 1 a2 + 1 —s requireF ˜Axoti™e th—t 2(a2 − bd) b + 2c + d + b + d = 2a2 + b2 + d2 + 2c(b + d) b + 2c + d = (a − b)2 + (a − d)2 + 2(a + c)(b + d) b + 2c + d (1) end simil—rlyD 2(c2 − db) d + 2a + b + b + d = (c − d)2 + (c − b)2 + 2(a + c)(b + d) d + 2a + b (2) …sing g—u™hyEƒ™hw—rz9s inequ—lityDwe get (a − d)2 b + 2c + d + (c − d)2 d + 2a + b ≥ [(a − b)2 + (c − d)2 ] (b + 2c + d) + (d + 2a + b) (3) Q
  • 3. (a − d)2 b + 2c + d + (c − b)2 d + 2a + b ≥ [(a − d)2 + (c − b)2 ]2 (b + 2c + d) + (d + 2a + b) (4) 2(a + c)(b + d) b + 2c + d + 2(a + c)(b + d) d + 2a + b ≥ 8(a + c)(b + d) (b + 2c + d) + (d + 2a + b) (5) prom @IAD@PAD@QAD@RA —nd @SAD we get 2( a2 − bd b + 2c + d + c2 − db d + 2a + b ) + b + d ≥ (a + c − b − d)2 + 4(a + c)(b + d) a + b + c + d = a + b + c + d. or a2 − bd b + 2c + d + c2 − db d + 2a + b ≥ a + c − b − d 2 sn the s—me m—nnerDwe ™—n —lso show th—t b2 − ca c + 2d + a + d2 − ac a + 2b + c ≥ b + d − a − c 2 —nd ˜y —dding these two inequ—litiesDwe get the desired resultF inqu—lity holds if —nd only if a = c —nd b = dF PD vet a, b, c ˜e positive re—l num˜ers su™h th—t a + b + c = 1 €rove th—t the following inequ—lity holds ab 1 − c2 + bc 1 − a2 + ca 1 − b2 ≤ 3 8 SolutionX prom the given ™ondition „he inequ—lity is equiv—lent to 4ab a2 + b2 + 2(ab + bc + ca) ≤ 3 2 ˜ut from g—uhy ƒhw—rz inequ—lity 4ab a2 + b2 + 2(ab + bc + ca) ≤ ab a2 + ab + bc + ca + ab b2 + ab + bc + ca = ab (a + b)(a + c) + ab (b + c)(a + b) = a(b + c)2 (a + b)(b + c)(c + a) „hus ‡e need prove th—t 3(a + b)(b + c)(c + a) ≥ 2 a(b + c)2 whi™h redu™es to the o˜vious inequ—lity ab(a + b) ≥ 6abc „he Solution is ™ompletedFwith equ—lity if —nd only if a = b = c = 1 3 R
  • 4. yr ‡e ™—n use the f—™t th—t 4ab a2 + b2 + 2(ab + bc + ca) ≤ 4ab (2ab + 2ac) + (2ab + 2bc) ≤ ab 2a(b + c) + ab 2b(a + c) = 1 2 b b + c + a a + c = 1 2 b b + c + c b + c = 3 2 QD vet a, b, c ˜e the positive re—l num˜ersF €rove th—t 1 + ab2 + bc2 + ca2 (ab + bc + ca)(a + b + c) ≥ 4. 3 (a2 + ab + bc)(b2 + bc + ca)(c2 + ca + ab) (a + b + c)2 SolutionX wultiplying ˜oth sides of the —˜ove inequ—lity with (a + b + c)2 it9s equiv—lent to prove th—t (a + b + c)2 + (a + b + c)(ab2 + bc2 + ca2 ) ab + bc + ca ≥ 4. 3 (a2 + ab + bc)(b2 + bc + ca)(c2 + ca + ab) ‡e h—ve (a + b + c)2 + (a + b + c)(ab2 + bc2 + ca2 ) ab + bc + ca = (a2 + ab + bc)(c + a)(c + b) ab + bc + ca fy using ewEqw inequ—lity ‡e get (a2 + ab + bc)(c + a)(c + b) ab + bc + ca ≥ 3. 3 (a2 + ab + bc)(b2 + bc + ca)(c2 + ca + ab)[(a + b)(b + c)(c + a)]2 ab + bc + ca ƒin™e it9s suffi™es to show th—t √ 3. 3 (a + b)(b + c)(c + a) ≥ 2. √ ab + bc + ca whi™h is ™le—rly true ˜y ewEqw inequ—lity —g—inF „he Solution is ™ompletedF iqu—lity holds for a = b = c RD vet a0, a1, . . . , an ˜e positive re—l num˜ers su™h th—t ak+1 −ak ≥ 1 for —ll k = 0, 1, . . . , n−1. €rove th—t 1 + 1 a0 1 + 1 a1 − a0 · · · 1 + 1 an − a0 ≤ 1 + 1 a0 1 + 1 a1 · · · 1 + 1 an SolutionX ‡e will prove it ˜y indu™tionF por n = 1 ‡e need to ™he™k th—t 1 + 1 a0 1 + 1 a1 − a0 ≤ 1 + 1 a0 1 + 1 a1 whi™h is equiv—lent to a0(a1 − a0 − 1) ≥ 0, whi™h is true ˜y given ™onditionF vet 1 + 1 a0 1 + 1 a1 − a0 · · · 1 + 1 ak − a0 ≤ 1 + 1 a0 1 + 1 a1 · · · 1 + 1 ak S
  • 5. it rem—ins to prove th—tX 1 + 1 a0 1 + 1 a1 − a0 · · · 1 + 1 ak+1 − a0 ≤ ≤ 1 + 1 a0 1 + 1 a1 · · · 1 + 1 ak+1 fy our hypothesis 1 + 1 a0 1 + 1 a1 · · · 1 + 1 ak+1 ≥ ≥ 1 + 1 ak+1 1 + 1 a0 1 + 1 a1 − a0 · · · 1 + 1 ak − a0 id estD it rem—ins to prove th—tX 1 + 1 ak+1 1 + 1 a0 1 + 1 a1 − a0 · · · 1 + 1 ak − a0 ≥ ≥ 1 + 1 a0 1 + 1 a1 − a0 · · · 1 + 1 ak+1 − a0 fut 1 + 1 ak+1 1 + 1 a0 1 + 1 a1 − a0 · · · 1 + 1 ak − a0 ≥ ≥ 1 + 1 a0 1 + 1 a1 − a0 · · · 1 + 1 ak+1 − a0 ⇔ ⇔ 1 ak+1 + 1 ak+1a0 1 + 1 a1 − a0 · · · 1 + 1 ak − a0 ≥ ≥ 1 (ak+1 − a0)a0 1 + 1 a1 − a0 · · · 1 + 1 ak − a0 ⇔ ⇔ 1 ≥ 1 ak+1 − a0 1 + 1 a1 − a0 · · · 1 + 1 ak − a0 fut ˜y our ™onditions ‡e o˜t—inX 1 ak+1 − a0 1 + 1 a1 − a0 · · · 1 + 1 ak − a0 ≤ ≤ 1 k 1 + 1 1 · · · 1 + 1 k − 1 = 1. „husD the inequ—lity is provenF SD qiven a, b, c > 0F €rove th—t 3 a2 + bc b2 + c2 ≥ 9. 3 √ abc (a + b + c) Solution X „his ineq is equiv—lent toX a2 + bc 3 abc(a2 + bc) 2 (b2 + c2) ≥ 9 (a + b + c) 3 fy ewEqw ineq D ‡e h—ve a2 + bc 3 abc(a2 + bc) 2 (b2 + c2) = T
  • 6. = a2 + bc 3 (a2 + bc)c(a2 + bc)b(b2 + c2)a ≥ 3(a2 + bc) sym a2b ƒimil—rlyD this ineq is true if ‡e prove th—tX 3(a2 + b2 + c2 + ab + bc + ca) sym a2b ≥ 9 (a + b + c) 3 a3 + b3 + c3 + 3abc ≥ sym a2 b ‡hi™h is true ˜y ƒ™hur ineqF iqu—lity holds when a = b = c TD vet a, b, c ˜e nonneg—tive re—l num˜ers su™h th—t ab + bc + ca > 0F €rove th—t 1 2a2 + bc + 1 2b2 + ca + 1 2c2 + ab ≥ 2 ab + bc + ca . „he inequ—lity is equiv—lent to ab + bc + ca 2a2 + bc ≥ 2, (1) or a(b + c) 2a2 + bc + bc bc + 2a2 ≥ 2.(2) …sing the g—u™hyEƒ™hw—rz inequ—lityD ‡e h—ve bc bc + 2a2 ≥ ( bc) 2 bc(bc + 2a2) = 1.(3) „hereforeD it suffi™es to prove th—t a(b + c) 2a2 + bc ≥ 1.(4) ƒin™e a(b + c) 2a2 + bc ≥ a(b + c) 2(a2 + bc) it is enough to ™he™k th—t a(b + c) a2 + bc ≥ 2, (5) whi™h is — known resultF ‚em—rkX 2ca + bc 2a2 + bc + 2bc + ca 2b2 + ca ≥ 4c a + b + c . UD vet a, b, c ˜e non neg—tive re—l num˜ers su™h th—t ab + bc + ca > 0F €rove th—t 1 2a2 + bc + 1 2b2 + ca + 1 2c2 + ab + 1 ab + bc + ca ≥ 12 (a + b + c)2 . SolutionX IA ‡e ™—n prove this inequ—lity using the following —uxili—ry result if 0 ≤ a ≤ min{a, b}D then 1 2a2 + bc + 1 2b2 + ca ≥ 4 (a + b)(a + b + c) . U
  • 7. in f—™tD this is used to repl—™ed for 4no two of whi™h —re zero4D so th—t the fr—™tions 1 2a2 + bc , 1 2b2 + ca , 1 2c2 + ab , 1 ab + bc + ca h—ve me—ningsF fesidesD the i—ker —lso works for itX 1 2a2 + bc + 1 2b2 + ca + 1 2c2 + ab ≥ 2(ab + bc + ca) a2b2 + abc(a + b + c) fut our Solution for ˜oth of them is exp—nd vet a, b, c ˜e non neg—tive re—l num˜ers su™h th—t ab + bc + ca > 0F €rove th—t 1 2a2 + bc + 1 2b2 + ca + 1 2c2 + ab + 1 ab + bc + ca ≥ 12 (a + b + c)2 . PA gonsider ˜y ewEqw inequ—lityD ‡e h—ve 2 a2 + ab + b2 (a + b + c) = (2b + a) 2a2 + bc + (2a + b) 2b2 + ca ≥ 2 (2a + b)(2b + a) (2a2 + bc) (2b2 + ca). end ˜y ewEqw inequ—lityD ‡e h—ve c2 (2a + b) 2a2 + bc + c2 (2b + a) 2b2 + ca ≥ 2 c4(2a + b)(2b + a) (2a2 + bc) (2b2 + ca) ≥ 2c2 (2a + b)(2b + a) (a2 + ab + b2) (a + b + c) = 4c2 a + b + c + 6abc a + b + c c a2 + ab + b2 2c2 a + bc2 + 2ab2 + b2 c 2a2 + bc = c2 (2a + b) 2a2 + bc + c2 (2b + a) 2b2 + ca ≥ 4c2 a + b + c + 6abc a + b + c c a2 + ab + b2 = 4 a2 + b2 + c2 a + b + c + 6abc a + b + c c a2 + ab + b2 ≥ 4 a2 + b2 + c2 a + b + c + 6abc a + b + c (a + b + c)2 c (a2 + ab + b2) = 4 a2 + b2 + c2 ab + c + 6abc ab + bc + ca ⇒ 2a2 b + 2ab2 + 2b2 c + 2bc2 + 2c2 a + 2ca2 2a2 + bc V
  • 8. = (b + c) + 2c2 a + bc2 + 2ab2 + b2 c 2a2 + bc ≥ (b + c) + 4 a2 + b2 + c2 a + b + c + 6abc ab + bc + ca = 8 a2 + b2 + c2 + ab + bc + ca a + b + c − 2 a2 b + ab2 ab + bc + ca ⇒ 1 2a2 + bc + 1 ab + bc + ca ≥ 4 a2 + b2 + c2 + ab + bc + ca (a + b + c) ( (a2b + ab2)) ≥ 12 (a + b + c)2 . <=> (a + b)(a + c) 2a2 + bc + a2 + bc 2a2 + bc − 2 ≥ 12(ab + bc + ca) (a + b + c)2 prom 2a2 + 2bc 2a2 + bc − 3 = bc 2a2 + bc ≥ 1 ‡e get a2 + bc 2a2 + bc − 2 ≥ 0 xowD ‡e will prove the stronger (a + b)(a + c) 2a2 + bc ≥ 12(ab + bc + ca) (a + b + c)2 prom ™—u™hyEs™h—rztD ‡e h—ve (a + b)(a + c) 2a2 + bc = (a+b)(b+c)(c+a)( 1 (2a2 + bc)(b + c) ≥ 3(a + b)(b + c)(c + a) ab(a + b) + bc(b + c) + ca(c + a) pin—llyD ‡e only need to prove th—t (a + b)(b + c)(c + a) ab(a + b) + bc(b + c) + ca(c + a) ≥ 4(ab + bc + ca) (a + b + c)2 (a + b + c)2 ab + bc + ca ≥ 4[ab(a + b) + bc(b + c) + ca(c + a) (a + b)(b + c)(c + a) = 4 − 8abc (a + b)(b + c)(c + a) a2 + b2 + c2 ab + bc + ca + 8abc (a + b)(b + c)(c + a) ≥ 2 whi™h is old pro˜lemF yur Solution —re ™ompleted equ—lity o™™ur if —nd if only a = b = c, a = b, c = 0 or —ny ™y™li™ permutionF VD vet a, b, c ˜e positive re—l num˜ers su™h th—t 16(a + b + c) ≥ 1 a + 1 b + 1 c F €rove th—t 1 a + b + 2(a + c) 3 ≤ 8 9 . SolutionX „his pro˜lem is r—ther e—syF …sing the ewEqw inequ—lityD ‡e h—veX a + b + 2(c + a) = a + b + c + a 2 + c + a 2 ≥ 3 3 (a + b)(c + a) 2 . W
  • 9. ƒo th—tX 1 a + b + 2(c + a) 3 ≤ 2 27(a + b)(c + a) . „husD it9s enough to ™he™k th—tX 1 3(a + b)(c + a) ≤ 4 ⇐⇒ 6(a + b)(b + c)(c + a) ≥ a + b + c, whi™h is true sin™e 9(a + b)(b + c)(c + a) ≥ 8(a + b + c)(ab + bc + ca) —nd 16abc(a + b + c) ≥ ab + bc + ca ⇒ 16(ab + bc + ca)2 3 ≥ ab + bc + ca ⇐⇒ ab + bc + ca ≥ 3 16 . „he Solution is ™ompletedF iqu—lity holds if —nd only if a = b = c = 1 4 F WD vet x, y, z ˜e positive re—l num˜ers su™h th—t xyz = 1F €rove th—t x3 + 1 x4 + y + z + y3 + 1 y4 + z + x + z3 + 1 z4 + x + y ≥ 2 √ xy + yz + zx. SolutionX …sing the ewEqw inequ—lityD ‡e h—ve 2 (x4 + y + z)(xy + yz + zx) = 2 [x4 + xyz(y + z)](xy + yz + zx) = 2 (x3 + y2z + yz2)(x2y + x2z + xyz) ≤ (x3 + y2 z + yz2 ) + (x2 y + x2 z + xyz) = (x + y + z)(x2 + yz) = (x + y + z)(x3 + 1) x . it follows th—t x3 + 1 x4 + y + z ≥ 2x √ xy + yz + zx x + y + z . edding this —nd it —n—logous inequ—litiesD the result followsF IHD vet a, b, c ˜e nonneg—tive re—l num˜ers s—tisfying a + b + c = √ 5F €rove th—t (a2 − b2 )(b2 − c2 )(c2 − a2 ) ≤ √ 5 SolutionX por this oneD ‡e ™—n —ssume ‡vyq th—t c ≥ b ≥ a so th—t ‡e h—ve P = (a2 − b2 )(b2 − c2 )(c2 − a2 ) = (c2 − b2 )(c2 − a2 )(b2 − a2 ) ≤ b2 c2 (c2 − b2 ). elso note th—t √ 5 = a + b + c ≥ b + c sin™e a ≥ 0F xowD using the ewEqw inequ—lity ‡e h—ve (c + b) · √ 5 2 − 1 · c 2 · √ 5 2 + 1 b 2 · (c − b) ≤ (c + b) √ 5(b + c) 5 5 ≤ √ 5; ƒo th—t ‡e get P ≤ √ 5F end hen™e ‡e —re doneF iqu—lity holds if —nd only if (a, b, c) =√ 5 2 + 1; √ 5 2 − 1; 0 —nd —ll its ™y™li™ permut—tionsF 2 IH
  • 10. IID vet a, b, c > 0 —nd a + b + c = 3F €rove th—t 1 3 + a2 + b2 + 1 3 + b2 + c2 + 1 3 + c2 + a2 ≤ 3 5 SolutionX ‡e h—veX 1 3 + a2 + b2 + 1 3 + b2 + c2 + 1 3 + c2 + a2 ≤ 3 5 <=> 3 3 + a2 + b2 + 3 3 + b2 + c2 + 3 3 + c2 + a2 ≤ 9 5 a2 + b2 3 + a2 + b2 ≥ 6 5 …sing g—u™hyEƒ™hw—rz9s inequ—lityX a2 + b2 3 + a2 + b2 ( 3 + a2 + b2 ) ≥ ( a2 + b2)2 „h—t me—ns ‡e h—ve to prove ( a2 + b2)2 ≥ 6 5 ( (3 + a2 + b2 )) (a2 + b2 ) + 2 (a2 + b2)(a2 + c2) ≥ 54 5 + 12 5 a2 8 a2 + 10 ab ≥ 54 <=> 5(a + b + c)2 + 3 a2 ≥ 54 it is true with a + b + c = 3F IPD qiven a, b, c > 0 su™h th—t ab + bc + ca = 1F €rove th—t 1 4a2 − bc + 1 + 1 4b2 − ca + 1 + 1 4c2 − ab + 1 ≥ 1 SolutionX in f—™tD the sh—rper inequ—lity holds 1 4a2 − bc + 1 + 1 4b2 − ca + 1 + 1 4c2 − ab + 1 ≥ 3 2 . „he inequ—lity is equiv—lent to 1 a(4a + b + c) + 1 b(4b + c + a) + 1 c(4c + a + b) ≥ 3 2 . …sing the g—u™hyEƒ™hw—rz inequ—lityD ‡e h—ve 1 a(4a + b + c) 4a + b + c a ≥ 1 a 2 = 1 a2b2c2 . „hereforeD it suffi™es to prove th—t 2 3a2b2c2 ≥ 4a + b + c a + 4b + c + a b + 4c + a + b c . ƒin™e 4a + b + c a = 3 + a + b + c a = 9 + (a + b + c)(ab + bc + ca) abc = 9 + a + b + c abc , II
  • 11. this inequ—lity ™—n ˜e written —s 9a2 b2 c2 + abc(a + b + c) ≤ 2 3 , whi™h is true ˜e™—use a2 b2 c2 ≤ ab + bc + ca 3 3 = 1 27 , —nd abc(a + b + c) ≤ (ab + bc + ca)2 3 = 1 3 . IQD qiven a, b, c ≥ 0 su™h th—t ab + bc + ca = 1F €rove th—t 1 4a2 − bc + 2 + 1 4b2 − ca + 2 + 1 4c2 − ab + 2 ≥ 1 SolutionX xoti™e th—t the ™—se abc = 0 is trivi—l so let us ™onsider now th—t abc > 0F …sing the ewEqw inequ—lityD ‡e h—ve 4a2 − bc + 2(ab + bc + ca) = (2a + b)(2a + c) ≤ [c(2a + b) + b(2a + c)]2 4bc = (ab + bc + ca)2 bc = 1 bc . it follows th—t 1 4a2 − bc + 2 ≥ bc. edding this —nd its —n—logous inequ—litiesD ‡e get the desired resultF IRD qiven a, b, c —re positive re—l num˜ersF €rove th—t ( 1 a + 1 b + 1 c )( 1 1 + a + 1 1 + b + 1 1 + c ) ≥ 9 1 + abc . SolutionX „he origin—l inequ—lity is equiv—lent to abc + 1 a + abc + 1 b + abc + 1 c 1 a + 1 + 1 b + 1 + 1 c + 1 ≥ 9 or cyc 1 + a2 c a 1 a + 1 + 1 b + 1 + 1 c + 1 ≥ 9 fy g—u™hy ƒ™hw—rz ineq —nd ewEqw ineqD cyc 1 + a2 c a ≥ cyc c(1 + a)2 a(1 + c) ≥ 3 3 (1 + a)(1 + b)(1 + c) —nd 1 a + 1 + 1 b + 1 + 1 c + 1 ≥ 3 3 (1 + a)(1 + b)(1 + c) wultiplying these two inequ—litiesD the ™on™lusion followsF iqu—lity holds if —nd only if a = b = c = 1F ISF qiven a, b, c —re positive re—l num˜ersF €rove th—tX a(b + 1) + b(c + 1) + c(a + 1) ≤ 3 2 (a + 1)(b + 1)(c + 1) IP
  • 12. SolutionX g—seIFif a + b + c + ab + bc + ca ≤ 3abc + 3 <=> 4(ab + bc + ca + a + b + c) ≤ 3(a + 1)(b + 1)(c + 1) …sing g—u™hyEƒ™h—wrz9s inequ—lity D‡e h—veX ( a(b + 1) + b(c + 1) + c(a + 1))2 ≤ 3(ab + bc + ca + a + b + c) ≤ 9(a + 1)(b + 1)(c + 1) 4 „he inequ—lity is trueF g—sePF ifa + b + c + ab + bc + ca ≤ 3abc + 3. <=> 9(a + 1)(b + 1)(c + 1) 4 ≥ 2(a + b + c + ab + bc + ca) + 3abc + 3 fy ewEqw9s inequ—lity X 2 ab(b + 1)(c + 1) ≤ [ab(c + 1) + (b + 1)] = a + b + c + ab + bc + ca + 3abc + 3 => ab + bc + ca + a + b + c + 2 ab(b + 1)(c + 1) ≤ 9 4(a + 1)(b + 1)(c + 1) => ( a(b + 1) + b(c + 1) + c(a + 1))2 ≤ [ 3 2 (a + 1)(b + 1)(c + 1)]2 => Q.E.D inqu—lity holds when a = b = c = 1. ITD qiven a, b, c —re positive re—l num˜ersF €rove th—tX 1 a2 + b2 + 1 b2 + c2 + 1 c2 + a2 ≥ 10 (a + b + c)2 SolutionX essume c = min{a, b, c}F „hen 1 a2 + c2 + 1 b2 + c2 ≥ 2 ab + c2 ⇐⇒ (ab − c2 )(a − b)2 ≥ 0 end ˜y g—u™hyEs™hw—rz ((a2 + b2 ) + 8(ab + c2 )) 1 a2 + b2 + 2 ab + c2 ≥ 25 ren™e ‡e need only to proveX 5(a + b + c)2 ≥ 2((a2 + b2 ) + 8(ab + c2 )) ⇐⇒ 3(a − b)2 + c(10b + 10a − 11c) ≥ 0 iqu—lity for a = b, c = 0 or permut—tionsF IUD vet a, b —nd c —re nonEneg—tive num˜ers su™h th—t ab + ac + bc = 0F €rove th—t a2 (b + c)2 a2 + 3bc + b2 (a + c)2 b2 + 3ac + c2 (a + b)2 c2 + 3ab ≤ a2 + b2 + c2 Solution: fy g—u™hyEƒ™hw—rz ineq D ‡e h—ve a2 (b + c) 2 a2 + bc = a2 (b + c) 3 (a2 + bc)(b + c) = a2 (b + c) 3 b(a2 + c2) + c(a2 + b2) ≤ a2 (b + c) 4 ( b2 b(a2 + c2) + c2 c(a2 + b2) ) = a2 (b + c) 4 ( b a2 + c2 + c a2 + b2 ) ƒimil—rlyD ‡e h—ve LHS ≤ a2 (b + c)( b a2 + c2 + c a2 + b2 ) = c(a2 (b + c) + b2 (c + a)) a2 + b2 IQ
  • 13. = a2 + b2 + c2 + abc(a + b) a2 + b2 ≤ a2 + b2 + c2 + abc(a + b) a2 + b2 ≤ a2 + b2 + c2 + ab + bc + ca whi™h is true ˜y ewEqw ineq „he origin—l inequ—lity ™—n ˜e written —s (a + b)2 (a + c)2 a2 + bc ≤ 8 3 (a + b + c)2 . ƒin™e (a + b)(a + c) = (a2 + bc) + a(b + c) ‡e h—ve (a + b)2 (a + c)2 a2 + bc = (a2 + bc)2 + 2a(b + c)(a2 + bc) + a2 (b + c)2 a2 + bc = a2 + bc + 2a(b + c) + a2 (b + c)2 a2 + bc , —nd thus the —˜ove inequ—lity is equiv—lent to a2 (b + c)2 a2 + bc ≤ 8 3 (a + b + c)2 − a2 − 5 ab, or a2 (b + c)2 a2 + bc ≤ 5(a2 + b2 + c2 ) + ab + bc + ca 3 . ƒin™e 5(a2 + b2 + c2 ) + ab + bc + ca 3 ≥ a2 + b2 + c2 + ab + bc + ca it is enough show th—t a2 (b + c)2 a2 + bc ≤ a2 + b2 + c2 + ab + bc + ca. FiFh IVD qiven a1 ≥ a2 ≥ . . . ≥ an ≥ 0, b1 ≥ b2 ≥ . . . ≥ bn ≥ 0 n i=1 ai = 1 = n i=1 bi pind the m—xmium of n i=1 (ai − bi)2 ‡SolutionXithout loss of gener—lityD —ssume th—t a1 ≥ b1 xoti™e th—t for a ≥ x ≥ 0, b, y ≥ 0 ‡e h—ve (a − x)2 + (b − y)2 − (a + b − x)2 − y2 = −2b(a − x + y) ≤ 0. e™™ording to this inequ—lityD ‡e h—ve (a1 − b1)2 + (a2 − b2)2 ≤ (a1 + a2 − b1)2 + b2 2, (a1 + a2 − b1)2 + (a3 − b3)2 ≤ (a1 + a2 + a3 − b1)2 + b2 3, · · · · · · IR
  • 14. (a1 + a2 + · · · + an−1 − b1)2 + (an − bn)2 ≤ (a1 + a2 + · · · + an − b1)2 + b2 n. edding these inequ—litiesD ‡e get n i=1 (ai − bi)2 ≤ (1 − b1)2 + b2 2 + b2 3 + · · · + b2 n ≤ (1 − b1)2 + b1(b2 + b3 + · · · + bn) = (1 − b1)2 + b1(1 − b1) = 1 − b1 ≤ 1 − 1 n . iqu—lity holds for ex—mple when a1 = 1, a2 = a3 = · · · = an = 0 —nd b1 = b2 = · · · = bn = 1 n IWD qiven a, b, c ≥ 0 su™h th—t a2 + b2 + c2 = 1 €rove th—t 1 − ab 7 − 3ac + 1 − bc 7 − 3ba + 1 − ca 7 − 3cb ≥ 1 3 SolutionX pirstD ‡e will show th—t 1 7 − 3ab + 1 7 − 3bc + 1 7 − 3ca ≤ 1 2 . …sing the g—u™hyEƒ™hw—rz inequ—lityD ‡e h—ve 1 7 − 3ab = 1 3(1 − ab) + 4 ≤ 1 9 1 3(1 − ab) + 1 . it follows th—t 1 7 − 3ab ≤ 1 27 1 1 − ab + 1 3 , —nd thusD it is enough to show th—t 1 1 − ab + 1 1 − bc + 1 1 − ca ≤ 9 2 , whi™h is †—s™9s inequ—lityF xowD ‡e write the origin—l inequ—lity —s 3 − 3ab 7 − 3ac + 3 − 3bc 7 − 3ba + 3 − 3ca 7 − 3cb ≥ 1, or 7 − 3ab 7 − 3ac + 7 − 3bc 7 − 3ba + 7 − 3ca 7 − 3cb ≥ 1 + 4 1 7 − 3ab + 1 7 − 3bc + 1 7 − 3ca . ƒin™e 4 1 7 − 3ab + 1 7 − 3bc + 1 7 − 3ca ≤ 2 IS
  • 15. it is enough to show th—t 7 − 3ab 7 − 3ac + 7 − 3bc 7 − 3ba + 7 − 3ca 7 − 3cb ≥ 3, whi™h is true —™™ording to the ewEqw inequ—lityF PID vet a, b, c ≥ 0 su™h th—t a + b + c > 0 —nd b + c ≥ 2a por x, y, z > 0 su™h th—t xyz = 1 €rove th—t the following inequ—lity holds 1 a + x2(by + cz) + 1 a + y2(bz + cx) + 1 a + z2(bx + cy) ≥ 3 a + b + c SolutionX ƒetting u = 1 x , v = 1 y —nd w = 1 z —nd using the ™ondition uvw = 1 the inequ—lity ™—n ˜e rewritten —s u au + cv + bw = u2 au2 + cuv + bwu 3 a + b + c F epplying g—u™hyD it suffi™es to prove (u + v + w) 2 a u2 + (b + c) uv 3 a + b + c 1 2 · (b + c − 2a) (x − y)2 0D whi™h is o˜vious due to the ™ondition for a, b, c PPD qiven x, y, z > 0 su™h th—t xyz = 1 IT
  • 16. €rove th—t 1 (1 + x2)(1 + x7) + 1 (1 + y2)(1 + y7) + 1 (1 + z2)(1 + z7) ≥ 3 4 SolutionX pirst ‡e prove this ineq e—sy 1 (1 + x2)(1 + x7) ≥ 3 4(x9 + x 9 2 + 1) end this ineq ˜e™—meX 1 x9 + x 9 2 + 1 + 1 y9 + y 9 2 + 1 + 1 z9 + z 9 2 + 1 ≥ 1 with xyz = 1 it9s —n old result PQD vet a, b, c ˜e positive re—l num˜ers su™h th—t 3(a2 + b2 + c2 ) + ab + bc + ca = 12 €rove th—t a √ a + b + b √ b + c + c √ c + a ≤ 3 √ 2 . SolutionX vet A = a2 + b2 + c2 , B = ab + bc + ca 2A + B = 2 a2 + ab ≤ 3 4 3 a2 + ab = 9. fy g—u™hy ƒ™hw—rz inequ—lityD ‡e h—ve a √ a + b = √ a a a + b ≤ √ a + b + c a a + b . fy g—u™hy ƒ™hw—rz inequ—lity —g—inD ‡e h—ve b a + b = b2 b(a + b) ≥ (a + b + c)2 b(a + b) = A + 2B A + B a a + b = 3 − b a + b ≤ 3 − A + 2B A + B = 2A + B A + B hen™eD it suffi™es to prove th—t (a + b + c) · 2A + B A + B ≤ 9 2 IU
  • 17. gonsider (a + b + c) √ 2A + B = (A + 2B) (2A + B) ≤ (A + 2B) + (2A + B) 2 = 3 2 (A + B) ⇒ (a + b + c) · 2A + B A + B ≤ 3 2 √ 2A + B ≤ 9 2 —s requireF fy ewEqw ineq e—sy to see th—t 3 ≤ a2 + b2 + c2 ≤ 4 fy g—u™hyEƒ™hw—rz ineqD ‡e h—ve LHS2 = ( a √ a + c (a + b)(a + c) ) ≤ (a2 + b2 + c2 + ab + bc + ca)( a (a + b)(a + c) ) …sing the f—mili—r ineq 9(a + b)(b + c)(c + a) ≥ 8(a + b + c)(ab + bc + ca) ‡e h—ve a (a + b)(a + c) = 2(ab + bc + ca) (a + b)(b + c)(c + a) ≤ 9 4(a + b + c) end ‡e need to prove th—t 9(a2 + b2 + c2 + ab + bc + ca) 4(a + b + c) ≤ 9 2 ⇔ 6 − (a2 + b2 + c2 ) 24 − 5(a2 + b2 + c2) ≤ 1 ⇔ (6 − (a2 + b2 + c2 ))2 ≤ 24 − 5(a2 + b2 + c2 ) ⇔ (3 − (a2 + b2 + c2 ))(4 − (a2 + b2 + c2 )) ≤ 0 ‡hi™h is true ‡e —re done equ—lity holds when a = b = c = 1 PRF qiven a, b, c ≥ 0 €rove th—t 1 (a2 + bc)(b + c)2 ≤ 8(a + b + c)2 3(a + b)2(b + c)2(c + a)2 SolutionX in f—™tD the sh—rper —nd ni™er inequ—lity holdsX a2 (b + c)2 a2 + bc + b2 (c + a)2 b2 + ca + c2 (a + b)2 c2 + ab ≤ a2 + b2 + c2 + ab + bc + ca. a2 (b + c)2 a2 + bc + b2 (c + a)2 b2 + ca + c2 (a + b)2 c2 + ab ≤ a2 + b2 + c2 + ab + bc + ca IV
  • 18. PSF qiven a, b, c ≥ 0 su™h th—t ab + bc + ca = 1 €rove th—t 1 8 5 a2 + bc + 1 8 5 b2 + ca + 1 8 5 c2 + ab ≥ 9 4 essume ‡vyq a ≥ b ≥ c this ineq 1 8 5 a2 + bc − 5 8 + 1 8 5 b2 + ca − 5 8 + 1 8 5 c2 + ab − 1 ≥ 0 8 − 8a2 − 5bc 8a2 + 5bc + 8 − 8b2 − 5ca 8b2 + 5ca + 1 − 8 5 c2 − ab c2 + 8 5 ab ≥ 0 8a(b + c − a) + 3bc 8a2 + 5bc + 8b(a + c − b) + 5ac 8b2 + 5ca + c(a + b − 8 5 c) c2 + 8 5 ab ≥ 0 xoti™e th—t ‡e only need to prove this ineq when a ≥ b + c ˜y the w—y ‡e need to prove th—t 8b 8b2 + 5ca ≥ 8a 8a2 + 5bc (a − b)(8ab − 5ac − 5bc) ≥ 0 i—sy to see th—tX if a ≥ b + c then 8ab = 5ab + 3ab ≥ 5ac + 6bc ≥ 5ac + 5ac ƒo this ineq is trueD ‡e h—ve qFdFe D equ—lity hold when (a, b, c) = (1, 1, 0) PTD qive a, b, c ≥ 0 €rove th—tX a b2 + c2 + b a2 + c2 + c a2 + b2 ≥ a + b + c ab + bc + ca + abc(a + b + c) (a3 + b3 + c3)(ab + bc + ca) a b2 + c2 = a2 ab2 + c2a ≥ (a + b + c)2 (ab2 + c2a) , it suffi™es to prove th—t a + b + c (ab2 + c2a) ≥ 1 ab + bc + ca + abc (ab + bc + ca) (a3 + b3 + c3) , IW
  • 19. ˜e™—use a + b + c (ab2 + c2a) − 1 ab + bc + ca = 3abc (ab + bc + ca) (ab2 + ca2) , it suffi™es to prove th—t 3 a3 + b3 + c3 ≥ ab2 + c2 a , whi™h is true ˜e™—use 2 a3 + b3 + c3 ≥ ab2 + c2 a . ‚em—rkX a b2 + c2 + b c2 + a2 + c a2 + b2 ≥ a + b + c ab + bc + ca + 3abc(a + b + c) 2(a3 + b3 + c3)(ab + bc + ca) . qive a, b, c ≥ 0 €rove th—t 1 a2 + bc + 1 b2 + ca + 1 c2 + ab ≥ 3 ab + bc + ca + 81a2 b2 c2 2(a2 + b2 + c2)4 iqu—lity o™™ur if —nd if only a = b = c, a = b, c = 0 or —ny ™y™li™ permutionF it is true ˜e™—use (1) 1 a2 + bc + 1 b2 + ca + 1 c2 + ab ≥ 3 a2 + b2 + c2 a3b + ab3 + b3c + bc3 + c3a + ca3 —nd (2) 3 a2 + b2 + c2 a3b + ab3 + b3c + bc3 + c3a + ca3 ≥ 3 ab + bc + ca + 81a2 b2 c2 2(a2 + b2 + c2)4 . fe™—use a2 (a3b + ab3) − 1 ab + bc + ca = abc(a + b + c) (ab + bc + ca) ( (a3b + ab3)) , it suffi™es to prove th—t 2(a + b + c) a2 + b2 + c2 4 ≥ 27abc(ab + bc + ca) a3 b + ab3 , whi™h is true ˜e™—use (a) (a + b + c) a2 + b2 + c2 ≥ 9abc, (b) a2 + b2 + c2 ≥ ab + bc + ca, (c) 2 a2 + b2 + c2 2 ≥ 3 a3 b + ab3 , whi™h (c) PH
  • 20. is equiv—lent to a2 − ab + b2 (a − b)2 ≥ 0, whi™h is trueF PUD vet a, b, c ˜e nonneg—tive num˜ersD no two of whi™h —re zeroF €rove th—t a2 (b + c) b2 + bc + c2 + b2 (c + a) c2 + ca + a2 + c2 (a + b) a2 + ab + b2 2(a2 + b2 + c2 ) a + b + c . SolutionX a2 (b + c) b2 + bc + c2 = 4a2 (b + c)(ab + bc + ca) (b2 + bc + c2) (ab + bc + ca) ≥ 4a2 (b + c)(ab + bc + ca) (b2 + bc + c2 + ab + bc + ca) 2 = 4a2 (ab + bc + ca) (b + c)(a + b + c)2 , it suffi™es to prove a2 b + c ≥ (a + b + c) a2 + b2 + c2 2(ab + bc + ca) , or a2 b + c + a ≥ (a + b + c)3 2(ab + bc + ca) , or a b + c ≥ (a + b + c)2 2(ab + bc + ca) , whi™h is true ˜y g—u™hyEƒ™hw—rz inequ—lity a b + c = a2 a(b + c) ≥ (a + b + c)2 2(ab + bc + ca) . ‡e just w—nt to give — little note hereF xoti™e th—t a2 (b + c) b2 + bc + c2 + a(b + c) a + b + c = a(b + c)(a2 + b2 + c2 + ab + bc + ca) (b2 + bc + c2)(a + b + c) , —nd 2(a2 + b2 + c2 ) a + b + c + a(b + c) a + b + c = 2(a2 + b2 + c2 + ab + bc + ca) a + b + c . „hereforeD the inequ—lity ™—n ˜e written in the form a(b + c) b2 + bc + c2 + b(c + a) c2 + ca + a2 + c(a + b) a2 + ab + b2 ≥ 2, xote th—t cyc a(b + c) b2 + bc + c2 = cyc 4a(b + c)(ab + bc + ca) 4(b2 + bc + c2)(ab + bc + ca) cyc 4a(ab + bc + ca) (b + c)(a + b + c)2 . PI
  • 21. ƒo th—t ‡e h—ve to proveX cyc 4a(ab + bc + ca) (b + c)(a + b + c)2 2, or cyc a b + c (a + b + c)2 2(ab + bc + ca) , whi™h is o˜viously true due to the g—u™hyEƒ™hw—rz inequ—lityF „his is —nother new SolutionF pirstD ‡e will prove th—t (a2 + ac + c2)(b2 + bc + c2) ≤ ab(a + b) + bc(b + c) + ca(c + a) a + b .(1) indeedD using the g—u™hyEƒ™hw—rz inequ—lityD ‡e h—ve √ ac · √ bc + a2 + ac + c2 · b2 + bc + c2 ≤ (ac + a2 + ac + c2)(bc + b2 + bc + c2) = (a + c)(b + c). it follows th—t (a2 + ac + c2)(b2 + bc + c2) ≤ ab + c2 + c a + b − √ ab ≤ ab + c2 + c a + b − 2ab a + b = ab(a + b) + bc(b + c) + ca(c + a) a + b . xowD from @IAD using the ewEqw inequ—lityD ‡e get 1 a2 + ac + c2 + 1 b2 + bc + c2 ≥ 2 (a2 + ac + c2)(b2 + bc + c2) ≥ 2(a + b) ab(a + b) + bc(b + c) + ca(c + a) . (2) prom (2) ‡e h—ve a(b + c) b2 + bc + c2 = ab 1 a2 + ac + c2 + 1 b2 + bc + c2 ≥ 2ab(a + b) ab(a + b) + bc(b + c) + ca(c + a) = 2. PWD if a, b, c > 0 then the following inequ—lity holdsX a2 (b + c) b2 + bc + c2 + b2 (c + a) c2 + ca + a2 + c2 (a + b) a2 + ab + b2 ≥ 2 a3 + b3 + c3 a + b + c „his inequ—lity is equiv—lent to a2 (b + c)(a + b + c) b2 + bc + c2 ≥ 2 (a3 + b3 + c3) (a + b + c) or a2 + a2 (ab + bc + ca) b2 + bc + c2 ≥ 2 (a3 + b3 + c3) (a + b + c), PP
  • 22. ˜e™—use 2 (a3 + b3 + c3) (a + b + c) ≤ a2 + b2 + c2 + a3 + b3 + c3 (a + b + c) a2 + b2 + c2 , it suffi™es to prove th—t a2 b2 + bc + c2 ≥ a3 + b3 + c3 (a + b + c) (a2 + b2 + c2) (ab + bc + ca) , ˜y g—u™hyEƒ™hw—rz inequ—lityD ‡e h—ve a2 b2 + bc + c2 ≥ a2 + b2 + c2 2 a2 (b2 + bc + c2) = a2 + b2 + c2 2 2 a2b2 + a2bc , it suffi™es to prove th—t a2 + b2 + c2 3 (ab + bc + ca) ≥ a3 + b3 + c3 (a + b + c) 2 a2 b2 + a2 bc . vet A = a4 , B = 1 2 a3 b + ab3 , C = a2 b2 , D = a2 bc, ‡e h—ve a2 + b2 + c2 2 = A + 2C, a2 + b2 + c2 (ab + bc + ca) = 2B + D, a3 + b3 + c3 (a + b + c) = A + 2B, —nd 2 a2 b2 + a2 bc = 2C + D. „hereforeD it suffi™es to prove th—t (A + 2C) (2B + D) ≥ (A + 2B) (2C + D) , or 2 (A − D) (B − C) ≥ 0, whi™h is true ˜e™—use A ≥ D —nd B ≥ C QHD qiven a, b, c ≥ 0 su™h th—t a + b + c = 1 €rove th—t 2 a2b + b2c + c2a + ab + bc + ca ≤ 1 ‚ewrite the inform inequ—lity —s 2 a2b + b2c + c2a + ab + bc + ca ≤ (a + b + c)2 PQ
  • 23. 2 (a2b + b2c + c2a) (a + b + c) ≤ a2 + b2 + c2 + ab + bc + ca essume th—t ˜ is the num˜er ˜etien — —nd ™F „henD ˜y —pplying the ewEqw inequ—lityD ‡e get 2 (a2b + b2c + c2a) (a + b + c) ≤ a2 b + b2 c + c2 a b + b(a + b + c) it is thus suffi™ient to prove the stronger inequ—lity a2 + b2 + c2 + ab + bc + ca ≥ a2 b + b2 c + c2 a b + b(a + b + c) „his inequ—lity is equiv—lent to c(a − b)(b − c) b ≥ 0, whi™h is o˜viously true —™™ording to the —ssumption of b row to prove a4 + 2 a3 c ≥ a2 b2 + 2 a3 b only ˜y ewEqw iquiv—lent to prove (a − b)2 (a + b)2 ≥ 4(a − b)(b − c)(a − c)(a + b + c) ‡vyq ‡e ™—n —ssume th—t a ≥ b ≥ c, a − b = x, b − c = y then ‡e need to prove th—t x2 (2c + 2y + x)2 + y2 (2c + y)2 + (x + y)2 (2c + x + y)2 ≥ xy(x + y)(3c + 2x + y) ˜y (x + y)4 ≥ xy(x + y)(x + 2y) —nd (x + y)3 ≥ 3xy(x + y) ‡e h—ve ™ompleted the Solution QID vet a, b, c ˜e positive num˜ers su™h th—t a2 b2 + b2 c2 + c2 a2 ≥ a2 b2 c2 pind the minimum of e A = a2 b2 c3(a2 + b2) + b2 c2 a3(b2 + c2) + c2 a2 b3(c2 + a2) xo one like this pro˜lemc ƒetting x = 1 a , y = 1 b , z = 1 c PR
  • 24. ‡e h—ve x2 + y2 + z2 ≥ 1 ‡e will prove th—t x3 y2 + z2 + y3 x2 + z2 + z3 x2 + y2 ≥ √ 3 2 …sing g—u™hyEƒ™hw—rzX LHS ≥ (x2 + y2 + z2 )2 x(y2 + z2) + y(x2 + z2) + z(x2 + y2) fy ewEqw ‡e h—veX x(y2 +z2 )+y(x2 +z2 )+z(x2 +y2 ) ≤ 2 3 (x2 +y2 +z2 )(x+y+z) ≤ 2 √ 3 (x2 +y2 +z2 ) x2 + y2 + z2 fe™—use x2 + y2 + z2 ≥ 1 ƒo (x2 + y2 + z2 )2 2√ 3 (x2 + y2 + z2) x2 + y2 + z2 ≥ √ 3 2 ‡e done3 QPF vet xDyDz ˜e non neg—tive re—l num˜ers su™h th—t x2 + y2 + z2 = 1 F find the minimum —nd m—ximum of f = x + y + z − xyz. Solution IF pirst ‡e fix z —nd let m = x+y = x+ √ 1 − x2 − z2 = g(x)(0 ≤ x ≤ √ 1 − z2), then ‡e h—ve g (x) = 1 − x √ 1 − x2 − z2 , ‡e get g (x) > 0 ⇔ 0 ≤ x < 1 − z2 2 —nd g (x) < 0 ⇔ 1 − z2 2 < x ≤ 1 − z2, so ‡e h—ve mmin = min{g(0), g( 1 − z2)} = 1 − z2 —nd mmax = g 1 − z2 2 = 2 − 2z2. e™tu—llyD f —nd written —s f = f(m) = − z 2 m2 + m + 1 − z2 z 2 + z, e—sy to prove th—t the —xis of symmetry m = 1 z > 2 − 2z2 PS
  • 25. so f(m) is in™re—sing in the interv—l of mD thusD ‡e h—ve f(m) ≥ f( 1 − z2) = 1 − z2 + z —nd f(m) ≤ f( 2 − 2z2) = z3 2 + z 2 + 2 − 2z2. ƒin™e ( 1 − z2 + z)2 = 1 + 2z 1 − z2 ≥ 1 ‡e get f(m) ≥ 1 —nd when two of xDyDz —re zero ‡e h—ve f = 1, soWegetfmin = 1. vet h(z) = z3 2 + z 2 + 2 − 2z2, e—sy to prove th—t h (z) > 0 ⇔ 0 ≤ z < 1 √ 3 andh (z) < 0 ⇔ 1 √ 3 < z ≤ 1 then ‡e get f(m) ≤ h 1 √ 3 = 8 √ 3 9 , when x = y = z = 1 √ 3 Wehavef = 8 √ 3 9 D so ‡e getfmax = 8 √ 3 9 . honeF Solution PF ‡hen two of xDyDz —re zero ‡e h—vef = 1D —nd ‡e will prove th—t f ≥ 1 then ‡e ™—n get fmin = 1F e™tu—llyD ‡e h—ve f ≥ 1 ⇔ x + y + z − xyz ≥ 1 ⇔ (x + y + z) x2 + y2 + z2 − xyz ≥ x2 + y2 + z2 3 ⇔ (x + y + z) x2 + y2 + z2 − xyz 2 ≥ x2 + y2 + z2 3 ⇔ x2 y2 z2 + 2 sym x5 y + x3 y3 + x3 y2 z ≥ 0, the l—st inequ—lity is o˜vious trueD so ‡e got f ≥ 1; ‡henx = y = z = 1 √ 3 ‡e h—ve f = 8 √ 3 9 , —nd ‡e will prove th—t f ≤ 8 √ 3 9 then ‡e ™—n get fmax = 8 √ 3 9 e™tu—llyD ‡e h—ve f ≤ 8 √ 3 9 ⇔ x + y + z − xyz ≤ 8 √ 3 9 ⇔ (x + y + z) x2 + y2 + z2 − xyz ≤ 8 √ 3 9 x2 + y2 + z2 3 ⇔ 27 (x + y + z) x2 + y2 + z2 − xyz 2 ≤ 64 x2 + y2 + z2 3 ⇔ 1 4 cyc S (x, y, z) (y − z) 2 ≥ 0, PT
  • 26. where S(x, y, z) = 17y2 (2y−x)2 +17z2 (2z−x)2 +56y2 (z−x)2 ++56z2 (y−x)2 +24x4 +6y4 +6z4 +57x2 (y2 +z2 )+104y2 z2 is o˜vious positiveD so the l—st inequ—lity is o˜vious trueD so ‡e gotfmax = 8 √ 3 9 . QQD por positive re—l num˜ersD show th—t a3 (b + c − a) a2 + bc + b3 (c + a − b) b2 + ca + c3 (a + b − c) c2 + ab ≤ ab + bc + ca 2 ineq a2 + b2 + c2 + ab + bc + ca 2 ≥ a3 (b + c − a) a2 + bc + a2 a2 + b2 + c2 + ab + bc + ca 2 ≥ (ab + bc + ca)( a2 a2 + bc ) a2 + b2 + c2 + (ab + bc + ca)( bc a2 + bc ) ≥ 5 2 (ab + bc + ca) a2 + b2 + c2 ab + bc + ca + bc a2 + bc ≥ 5 2 …se two ineq bc a2 + bc + ab c2 + ab + ac b2 + ac ≥ 4abc (a + b)(b + c)(c + a) + 1(1) it is e—sy to proveF a2 + b2 + c2 ab + bc + ca + 8abc (a + b)(b + c)(c + a) ≥ 2(2) ƒo e—sy to see th—t a2 + b2 + c2 ab + bc + ca + bc a2 + bc ≥ a2 + b2 + c2 ab + bc + ca + 4abc (a + b)(b + c)(c + a) + 1 ≥ a2 + b2 + c2 2(ab + bc + ca) + 2 ≥ 5 2 ‡e h—ve done 3 a3 (b + c − a) a2 + bc + b3 (c + a − b) b2 + ca + c3 (a + b − c) c2 + ab ≤ 3abc(a + b + c) 2(ab + bc + ca) Solution a2 + b2 + c2 ab + bc + ca + bc a2 + bc + 3abc(a + b + c) 2(ab + bc + ca) 2 ≥ 3 end ‡e prove th—t 3abc(a + b + c) 2(ab + bc + ca) 2 ≥ 4abc (a + b)(b + c)(c + a) 3(a + b + c)(a + b)(b + c)(c + a) ≥ 8(ab + bc + ca)2 „his ineq is true ˜e™—use 3(a + b + c)(a + b)(b + c)(c + a) ≥ 8 3 (a + b + c)2 (ab + bc + ca) ≥ 8(ab + bc + ca)2 PU
  • 27. ƒo LHS ≥ a2 + b2 + c2 ab + bc + ca + 4abc (a + b)(b + c)(c + a) + 4abc (a + b)(b + c)(c + a) + 1 ≥ 3 vet a, b, c > 0 ƒhow th—t a3 (b + c − a) a2 + bc + b3 (c + a − b) b2 + ca + c3 (a + b − c) c2 + ab ≤ 9abc 2(a + b + c) pirstD‡e prove this lenm—X a2 a2 + bc + b2 b2 + ca + c2 c2 + ab ≤ (a + b + c)2 2(ab + bc + ca) bc a2 + bc + ac b2 + ac + ab c2 + ab + a2 + b2 + c2 2(ab + bc + ca) ≥ 2 whi™h is true from bc a2 + bc + ac b2 + ac + ab c2 + ab ≥ 1 + 4abc (a + b)(b + c)(c + a) a2 + b2 + c2 2(ab + bc + ca) + 4abc (a + b)(b + c)(c + a) ≥ 1 equ—lity o™™ur if —nd if only a = b = c or a = b, c = 0 or —ny ™y™li™ permutionF ‚eturn to your inequ—lityD‡e h—ve ( a3 (b + c − a) a2 + bc + a2 ) ≤ a2 + b2 + c2 + 9abc 2(a + b + c) or (ab + bc + ca) a2 a2 + bc ≤ a2 + b2 + c2 + 9abc 2(a + b + c) prom a2 a2 + bc + b2 b2 + ca + c2 c2 + ab ≤ (a + b + c)2 2(ab + bc + ca) ‡e only need to prove th—t (a + b + c)2 2 ≤ a2 + b2 + c2 + 9abc 2(a + b + c) or a2 + b2 + c2 + 9abc a + b + c ≥ 2(ab + bc + ca) ‡hi™h is s™hur inequ—lityF yur Solution —re ™ompleted equ—lity o™™ur if —nd if only a = b = c, a = b, c = 0 or —ny ™y™li™ permutionF QQD vet a, b, c > 0 PV
  • 28. su™h th—t a + b + c = 1 „hen a3 + bc a2 + bc + b3 + ca b2 + ca + c3 + ab c2 + ab ≥ 2 prom the ™ondition a − 1 = −(b + c) it follows th—t a3 + bc a2 + bc = − a2 (b + c) a2 + bc + 1 „hus it suffi™es to prove th—t a + b + c ≥ a2 (b + c) a2 + bc por a, b, c positive re—ls prove th—t ab(a + b) c2 + ab ≥ a <=> a4 + b4 + c4 − b2 c2 − c2 a2 − a2 b2 ≥ 0 ab(a + b) c2 + ab + c2 (a + b) c2 + ab = 2 a —nd our inequ—lity ˜e™omes c2 (a + b) (c2 + ab) ≤ a ˜ut c2 (a + b) (c2 + ab) = c2 (a + b)2 (c2 + ab)(a + b) = (ca + cb)2 a(b2 + c2) + b(a2 + c2) ≤ c2 a2 a(b2 + c2) + c2 b2 b(a2 + c2) = a QR vet a, b, c ≥ 0 su™h th—t a + b + c = 1 „hen 6(a2 + b2 + c2 ) ≥ a3 + bc a2 + bc + b3 + ca b2 + ca + c3 + ab c2 + ab Solution 6(a2 + b2 + c2 ) + a2 (b + c) a2 + bc ≥ 3 6(a2 + b2 + c2 ) − 2(a + b + c)2 ≥ (a− a2 (b + c) a2 + bc ) 4 (a − b)(a − c) ≥ a(a − b)(a − c) a2 + bc (a − b)(a − c)(4 − a a2 + bc ) ≥ 0 PW
  • 29. essuming ‡vyq a ≥ b ≥ c then e—sy to see th—t 4 − a a2 + bc ≥ 0 —nd 4 − c c2 + ab ≥ 0 (c − a)(c − b)(4 − c c2 + ab ) ≥ 0and(a − b)(a − c)(4 − a a2 + bc ) ≥ 0 ‡e h—ve two ™—ses g—se I 4 − b b2 + ac ≤ 0 then (b − c)(b − a)(4 − b b2 + ac ) ≥ 0 so this ineq is true g—se P 4 − b b2 + ac ≤ 0 e—sy to see th—t 4 − c c2 + ab ≥ 4 − b b2 + ac ƒo LHS ≥ (c − b)2 (4 − c c2 + ab ) + (a − b)(b − c)( b b2 + ac − c c2 + ab ) ≥ 0 FiFh QSF vet x, y, z ˜e re—l num˜ers s—tisfyX x2 y2 + 2yx2 + 1 = 0 pind the m—ximum —nd minimum v—lues ofX f(x, y) = 2 x2 + 1 x + y(y + 1 x + 2) SolutionX €ut t = 1 x ; k = y + 1 D ‡e h—veX t2 + k2 = 1 f(x, y) = t2 + tk €ut t = cos α; k = sin α then f(x, y) = cos α2 + cos α sinα = sin 2α2 = 1 2 + 1 √ 2 cos (2α − π 4 ) max f(x, y) = 1 2 + 1 √ 2 QH
  • 30. min f(x, y) = 1 2 − 1 √ 2 FiFh F QTF ƒuppose —D˜D™Dd —re positive integers with ab + cd = 1. „henD por We = 1, 2, 3, 4,let (xi)2 + (yi)2 = 1, where xi —nd yi —re re—l num˜ersF ƒhow th—t (ay1 + by2 + cy3 + dy4)2 + (ax4 + bx3 + cx2 + dx1)2 ≤ 2( a b + b a + c d + d c ). ƒolitionX …se g—u™hyEƒ™hw—rtz D ‡e h—ve (ay1 + by2 + cy3 + dy4)2 ≤ (ab + cd)( (ay1 + by2)2 ab + (cy3 + dy4)2 cd ) = (ay1 + by2)2 ab + (cy3 + dy4)2 cd ƒimil—rX (ax4 + bx3 + cx2 + dx1)2 ≤ (ab + cd)( (ax4 + bx3)2 ab + (cx2 + dx1)2 cd ) = (ax4 + bx3)2 ab + (cx2 + dx1)2 cd futX (ay1 + by2)2 ≤ (ay1 + by2)2 + (ax1 − bx2)2 = a2 + b2 + 2ab(y1y2 − x1x2) ƒimil—rF (cx2 + dx1)2 ≤ c2 + d2 + 2cd(x1x2 − y1y2) D then ‡e getX (ay1 + by2)2 ab + (cx2 + dx1)2 cd ≤ a b + b a + c d + d c @IA „he s—me —rgument show th—tX (cy3 + dy4)2 cd + (ax4 + bx3)2 ab ≤ a b + b a + c d + d c @PA gom˜ining @IAY@PA ‡e get F FiFh QUF in —ny ™onvex qu—dril—ter—l with sides a ≤ b ≤ c ≤ d QI
  • 31. —nd —re— F €rove th—t X F ≤ 3 √ 3 4 c2 SolutionX „he inequ—lity is rewritten —sX (−a + b + c + d)(a − b + c + d)(a + b − c + d)(a + b + c − d) ≤ 27c4 . ‡e su˜stitute x = −a + b + c + dD y = a − b + c + dD z = a + b − c + dD t = a + b + c − dF „hen x + y − z + t 4 = c —nd x ≥ y ≥ z ≥ t. „hus ‡e h—veX xyzt ≤ 27( x + y − z + t 4 )4 . „he left side of the inequ—lity is m—ximum when z = y while the right side of the inequ—lity is minimum @‡e h—ve fixed xDy —nd tAF „hen ‡e just prove th—t xy2 t ≤ 27( x + t 4 )4 . fe™—use xy2 t ≤ x3 tD ‡e just h—ve to prove x3 t ≤ ( x + t 4 )4 end then it follows th—t the —˜ove inequ—lity is —lso trueF x 3 + x 3 + x 3 + t ≥ 4 4 x 3 · x 3 · x 3 · t hen™e 27( x + y 4 )4 ≥ x3 t QVF vet efg ˜e — tri—ngleF €rove th—tX 1 a + 1 b + 1 c ≤ 1 a + b − c + 1 b + c − a + 1 c + a − b SolutionX IF 1 a + b − c + 1 b + c − a = 2b (a + b − c)(b + c − a) = 2b b2 − (c − a)2 ≥ 2b b2 = 2 b ƒimil—rlyD ‡e h—ve 1 b + c − a + 1 c + a − b ≥ 2 c 1 c + a − b + 1 a + b − c ≥ 2 a edd three inequ—lities together —nd divide ˜y P to get the desired resultF PF use u—r—m—t— for the num˜er —rr—ys (b + c − a; c + a − b; a + b − c) (a; b; c) —nd the ™onvex fun™tion f (x) = 1 x QP
  • 32. yr m—ke the su˜stitution x = 1 2 (b + c − a)D y = 1 2 (c + a − b)D z = 1 2 (a + b − c) —nd get a = y + z, b = z + x, c = x + y, so th—t the inequ—lity in question ™—n ˜e rewritten —s 1 y + z + 1 z + x + 1 x + y ≤ 1 2x + 1 2y + 1 2z D wh—t dire™tly follows from ewErwX 2 y + z ≤ 1 2y + 1 2z , 2 z + x ≤ 1 2z + 1 2x , 2 x + y ≤ 1 2x + 1 2y QWF vet a, b, c ˜e nonneg—tive re—l num˜ersF €rove th—t a3 + b3 + c3 + 3abc ≥ (a2 b + b2 c + c2 a)2 ab2 + bc2 + ca2 + (ab2 + bc2 + ca)2 a2b + b2c + c2a SolutionX if a = 0 or b = 0 or c = 0 Dit9s trueFif abc > 0 €ut x = a b , y = b c , z = c a F‡e need prove x z + y x + z y + 3 ≥ (xy + yz + zx)2 xyz(x + y + z) + (x + y + z)2 xy + yz + zx x z + y x + z y ≥ x2 y2 + y2 z2 + z2 x2 xyz(x + y + z) + x2 + y2 + z2 xy + yz + zx x2 z + z2 y + y2 x ≥ (x2 + y2 + z2 )(x + y + z) xy + yz + zx x3 y z + y3 z x + z3 x y ≥ x2 y + y2 z + z2 x fy using ew qw9inequ—lityD ‡e h—veX x3 y z + xyz ≥ 2x2 y, y3 z x + xyz ≥ 2y2 z z3 x y + xyz ≥ 2z2 y, x2 y + y2 z + z2 x ≥ 3xyz ‡e h—ve done RHF vet x, y, z ˜e positive re—l num˜ersF €rove th—tX x + 1 y − 1 y + 1 z − 1 ≥ 3. SolutionF ‡e rewrite the inequ—lity —s y x + 1 xyz − 2 x + 1 − 2 xyz xy + 3 ≥ 0. QQ
  • 33. €utting xyz = k3 D then there exist a, b, c > 0 su™h th—t x = ka b , y = kb c , z = kc a . „he inequ—lity ˜e™omes a2 bc + 1 k2 − 2k a b + k2 − 2 k b a + 3 ≥ 0 f(k) = a3 + k2 − 2 k a2 b + 1 k2 − 2k ab2 + 3abc ≥ 0 ‡e h—ve th—t f (k) = 2(k3 + 1) k3 k a2 b − ab2 f (k) = 0 ⇔ k = ab2 a2b . prom nowD —™™ording to the †—ri—tion fo—rdD ‡e ™—n dedu™e our inequ—lity to show th—t f ab2 a2b ≥ 0 or equiv—lentlyD a3 + b3 + c3 + 3abc ≥ (a2 b + b2 c + c2 a)2 ab2 + bc2 + ca2 + (ab2 + bc2 + ca2 )2 a2b + b2c + c2a . FiFh RIF qiven a, b, c ≥ 0F€rove th—tX (a + b + c)2 2(ab + bc + ca) ≥ a2 a2 + bc + b2 b2 + ca + c2 c2 + ab SolutionX ‡e h—ve 2a2 (a + b)(a + c) − a2 a2 + bc = a2 (a − b)(a − c) (a + b)(a + c)(a2 + bc) ≥ 0 @e—sy to ™he™k ˜y †orni™u ƒ™hurA it suffi™es to prove th—t (a + b + c)2 2(ab + bc + ca) ≥ 2a2 (a + b)(a + c) = 2 ab(a + b) (a + b)(b + c)(c + a) essume th—t a + b + c = 1 —nd put q = ab + bc + ca, r = abcD then the inequ—lity ˜e™omes 1 4q ≥ q − 3r q − r ⇔ q − r q − 3r ≥ 4q ⇔ 2r q − 3r ≥ 4q − 1 fy ƒ™hur9s inequ—lity for third degreeD ‡e h—ve r ≥ 4q−1 9 D then 2r q − 3r ≥ 2r q − 4q−1 3 = 6r 1 − q QR
  • 34. it suffi™es to show th—t 6r ≥ (4q − 1)(1 − q) fut this is just ƒ™hur9s inequ—lity for fourth degree a4 + abc a ≥ ab(a2 + b2 ) ‡e h—ve doneF PF ƒuppose a + b + c = 3F ‡e need to proveX f(r) = 4q4 − 9q3 + 24qr2 − 54q2 r − 72r2 − 243r + 216qr ≤ 0 f (r) = 48qr − 54q2 − 144r − 243 + 216q f (r) = 48(q − 3) ≤ 0, sof (r) ≤ f (0) = −54q2 − 144 + 216q ≤ 0 ƒoD with q ≤ 9 4 , f(r) ≤ f(0) = q3 (4q − 9) ≤ 0 ‡ith q ≥ 9 4 D ‡e h—veX f(r) ≤ f(4q−9 3 ) ≤ 0 @trues with q ≥ 9 4 A RPF vet a, b, c ˜e nonneg—tive re—l num˜ers su™h th—t a2 + b2 + c2 = 1F €rove th—t a3 b2 − bc + c2 + b3 + c3 a2 ≥ √ 2 SolutionX a3 b2 − bc + c2 + b3 + c3 a2 ≥ (a2 + b2 + c2 )2 a[b2 − bc + c2 + a(b + c)] ≥ 1 2.a[3−2a2 4 ] = 1 2 a2( 3 2 −a2 2 )2 ≥ √ 2 RQF vet ∆ABC —nd max(A, B, C) ≤ 90F €rove th—t X cosAcosB sin2C + cosBcosC sin2A + cosCcosA sin2B ≥ √ 3 2 SolutionX fut if A = 90◦ the left side does not existF if max{A, B, C} < 90◦ F vet a2 + b2 − c2 = z, a2 + c2 − b2 = y —nd b2 + c2 − a2 = x. ren™eD x, y —nd z —re positive —nd cyc cos A cos B sin 2C = cyc b2 +c2 −a2 2bc · a2 +c2 −b2 2ac 2 · 2S ab · a2+b2−c2 2ab = = cyc ab(b2 + c2 − a2 )(a2 + c2 − b2 ) 8c2S(a2 + b2 − c2) = cyc xy (x + y)z a2b2 2(a2b2 + a2c2 + b2c2) − a4 − b4 − c4 = = cyc xy (x + y)z (x + z)(y + z) 4(xy + xz + yz) . „husD it rem—ins to prove th—t cyc xy (x + y)z (x + z)(y + z) xy + xz + yz ≥ √ 3, QS
  • 35. whi™h is equiv—lent to cyc x2 y2 (x + z)(y + z) x + y ≥ xyz 3(xy + xz + yz). fy g—u™hyEƒ™hw—rtz ‡e o˜t—inX cyc x2 y2 (x + z)(y + z) x + y · cyc x + y (x + z)(y + z) ≥ (xy + xz + yz)2 . ren™eD ‡e need to prove th—t (xy + xz + yz)2 ≥ cyc x + y (x + z)(y + z) · xyz 3(xy + xz + yz). ‡e o˜t—inX (xy + xz + yz)2 ≥ cyc x + y (x + z)(y + z) · xyz 3(xy + xz + yz) ⇔ ⇔ √ xy + xz + yz 3 (x + y)(x + z)(y + z) ≥ cyc xyz(x + y) 3(x + y) ⇔ ⇔ (xy + xz + yz)3 (x + y)(x + z)(y + z) ≥ 3x2 y2 z2 cyc (x + y)3 + +6x2 y2 z2 cyc (x + y)(x + z) (x + y)(x + z) ⇔ ⇔ (xy + xz + yz)3 (x + y)(x + z)(y + z) ≥ 3x2 y2 z2 cyc (2x3 + 3x2 y + 3x2 z)+ +3x2 yz cyc (x + y)(x + z)2 z2(x + y)y2(x + z). fy ewEqw 2 z2(x + y)y2(x + z) ≤ y2 x + y2 z + z2 x + z2 y. ren™eD it rem—ins to prove th—t ⇔ (xy + xz + yz)3 (x + y)(x + z)(y + z) ≥ 3x2 y2 z2 cyc (2x3 + 3x2 y + 3x2 z)+ +3x2 yz cyc(x + y)(x + z)(y2 x + y2 z + z2 x + z2 y), whi™h is equiv—lent to sym(x5 y4 + x5 y3 z − 5x4 y3 z2 + x4 y4 z + 2x3 y3 z3 ) ≥ 0, whi™h is true ˜y ewEqw ˜e™—use sym (x5 y4 + x5 y3 z − 5x4 y3 z2 + x4 y4 z + 2x3 y3 z3 ) ≥ 0 ⇔ ⇔ sym (x5 y4 + x5 y3 z + 1 3 x4 y4 z + 2 3 x4 z4 y + 2x3 y3 z3 ) ≥ sym 5x4 y3 z2 . FiFh RRF vet a, b, c ˜e nonneg—tive re—l num˜ersD no two of whi™h —re zeroF €rove th—t a b + c + b c + a + c a + b + a2 b + b2 c + c2 a ab2 + bc2 + ca2 ≥ 5 2 QT
  • 36. Solution IFFF—sume p = 1 —nd vemm— ab2 + bc2 + ca2 ≤ 4 27 − r ⇔ a b + c + b c + a + c a + b + ab(a + b) ab2 + bc2 + ca2 ≥ 7 2 ‡e h—ve a b + c + b c + a + c a + b + ab(a + b) ab2 + bc2 + ca2 ≥ 1 − 2q + 3r q − r + 27q − 81r 4 − 27r ‡e need prove th—t 1 − 2q + 3r q − r + 27q − 81r 4 − 27r ≥ 7 2 ⇔ 1 + r q − r − 2 + 27q − 12 4 − 27r + 3 ≥ 7 2 ⇔ 1 + r q − r + 27q − 12 4 − 27r ≥ 5 2 ⇔ −135r2 + r(81q + 2) + (54q2 + 8 − 44q) ≥ 0 f(r) = −135r2 + r(81q + 2) + (54q2 + 8 − 44q) f (r) = −270r + 81q + 2 ≥ 0 @˜e™—use q ≥ 9rA ⇒ f(r) ≥ f( 4q − 1 9 ) = 570q2 − 349q + 55 9 ≥ 0 PFFFFFFFF a b + c + b c + a + c a + b + a2 b + b2 c + c2 a ab2 + bc2 + ca2 ≥ 5 2 ⇔ ⇔ cyc a b + c − 1 2 ≥ (a2 c − a2 b) a2c ⇔ ⇔ cyc a − b − (c − a) 2(b + c) ≥ (a − b)(b − c)(c − a) a2c + b2a + c2b ⇔ ⇔ cyc a − b 2 1 b + c − 1 a + c ≥ (a − b)(b − c)(c − a) a2c + b2a + c2b ⇔ ⇔ cyc (a − b)2 (a + c)(b + c) ≥ 2(a − b)(b − c)(c − a) a2c + b2a + c2b . if (a − b)(b − c)(c − a) ≤ 0 then the inequ—lity holdsF vet (a − b)(b − c)(c − a) > 0 —nd a2 c+b2 a+c2 b a2b+b2c+c2a = t. „hen t > 1. fy ewEqw ‡e o˜t—inX cyc (a − b)2 (a + c)(b + c) ≥ 3 3 (a − b)2(b − c)2(c − a)2 (a + b)2(a + c)2(b + c)2 . „husD it rem—ins to prove th—t 27(a2 c + b2 a + c2 b)3 ≥ 8(a + b)2 (a + c)2 (b + c)2 (a − b)(b − c)(c − a). QU
  • 37. fut (a + b)(a + c)(b + c) = cyc (a2 b + a2 c) + 2abc ≤ 4 3 cyc (a2 b + a2 c). id estD it rem—ins to prove th—t 27t3 ≥ 8 · 16 9 (t + 1)2 (t − 1), whi™h o˜viousF RSF por —ll nonneg—tive re—l num˜ers a, b —nd cD no two of whi™h —re zeroD 1 (a + b)2 + 1 (b + c)2 + 1 (c + a)2 ≥ 3 3abc(a + b + c)(a + b + c)2 4(ab + bc + ca)3 Solution ‚epl—™ing a, b, c ˜y 1 a , 1 b , 1 c respe™tivelyD ‡e h—ve to prove th—t a2 b2 (a + b)2 ≥ 3 3(ab + bc + ca)(ab + bc + ca)2 4(a + b + c)3 . xowD using g—u™hy ƒ™hw—rz inequ—lityD ‡e h—ve a2 b2 (a + b)2 ≥ (ab + bc + ca)2 (a + b)2 + (b + c)2 + (c + a)2 = (ab + bc + ca)2 2(a2 + b2 + c2 + ab + bc + ca) . it suffi™es to prove th—t (ab + bc + ca)2 2(a2 + b2 + c2 + ab + bc + ca) ≥ 3 3(ab + bc + ca)(ab + bc + ca)2 4(a + b + c)3 or equiv—lentlyD 2(a + b + c)3 ≥ 3 3(ab + bc + ca)(a2 + b2 + c2 + ab + bc + ca), th—t is 4(a + b + c)6 ≥ 27(ab + bc + ca)(a2 + b2 + c2 + ab + bc + ca)2 fy ewEqwD ‡e see th—t 27(ab + bc + ca)(a2 + b2 + c2 + ab + bc + ca)2 ≤ 1 2 2(ab + bc + ca) + (a2 + b2 + c2 + ab + bc + ca) + (a2 + b2 + c2 + ab + bc + ca) 3 = 4(a+b+c)6 . „hereforeD our Solution is ™ompleted RTF 2 3 ( 1 a2 + bc + 1 b2 + ca + 1 c2 + ab ) ≥ 1 ab + bc + ca + 2 a2 + b2 + c2 SolutionX ‚ewrite our inequ—lity —sX 1 a2 + bc ≥ 3(a + b + c)2 2(a2 + b2 + c2)(ab + bc + ca) . ‡e will ™onsider P ™—sesX g—se IF a2 +b2 +c2 ≤ 2(ab+bc+ca), then —pplying g—u™hy ƒ™hw—rz inequ—lityD ‡e ™—n redu™e our inequ—lity to 6 a2 + b2 + c2 + ab + bc + ca ≥ (a + b + c)2 (a2 + b2 + c2)(ab + bc + ca) , (a2 + b2 + c2 − ab − bc − ca)(2ab + 2bc + 2ca − a2 − b2 − c2 ) ≥ 0, whi™h is trueF g—se PF a2 + b2 + c2 ≥ 2(ab + bc + ca), then (a + b + c)2 ≤ 2(a2 + b2 + c2 ), whi™h yields th—t 3(a + b + c)2 2(a2 + b2 + c2)(ab + bc + ca) ≤ 3 ab + bc + ca , QV
  • 38. —nd ‡e just need to prove th—t 1 a2 + bc + 1 b2 + ca + 1 c2 + ab ≥ 3 ab + bc + ca , whi™h is just your very known @—nd ni™eA inequ—lityF RUF vet a, b, c ˜e nonneg—tive re—l num˜ersD no two of whi™h —re zeroF €rove th—t (a) 1 2a2 + bc + 1 2b2 + ca + 1 2c2 + ab ≥ 1 ab + bc + ca + 2 a2 + b2 + c2 SolutionX Ist Solution fy g—u™hy inequ—lityD cyc (b + c)2 (2a2 + bc) cyc 1 2a2 + bc ≥ 4(a + b + c)2 it rem—ins to show th—t cyc (b + c)2 (2a2 + bc) ≤ 4(a2 + b2 + c2 )(ab + bc + ca) whi™h is e—syF Pnd SolutionF ƒin™e cyc ab + bc + ca 2a2 + bc = cyc bc 2a2 + bc + cyc a(b + c) 2a2 + bc ‡e need to show cyc bc 2a2 + bc ≥ 1 —nd cyc a(b + c) 2a2 + bc ≥ 2(ab + bc + ca) a2 + b2 + c2 „he former is illEknownX if x, y, z ≥ 0 su™h th—t xyz = 1D then 1 2x + 1 + 1 2y + 1 + 1 2z + 1 ≥ 1 „he l—terX ˜y g—u™hy inequ—lityD cyc a(b + c)(2a2 + bc) cyc a(b + c) 2a2 + bc ≥ 4(ab + bc + ca)2 „he result then follows from the following identity cyc a(b + c)(2a2 + bc) = 2(ab + bc + ca)(a2 + b2 + c2 ) Qrd SolutionF LHS − RHS a + b + c = 2(a + b + c)(a − b)2 (b − c)2 (c − a)2 + 3abc cyc(a2 + ab + b2 )(a − b)2 (2a2 + bc)(2b2 + ca)(2c2 + ab)(ab + bc + ca)(a2 + b2 + c2) Qrd SolutionF essume th—t c = min{a, b, c}D then the g—u™hy ƒ™hw—rz inequ—lity yields 1 2a2 + bc + 1 2b2 + ca ≥ 4 2(a2 + b2) + c(a + b) , QW
  • 39. then ‡e just need to prove th—t 4 2(a2 + b2) + c(a + b) + 1 ab + 2c2 ≥ 1 ab + bc + ca + 2 a2 + b2 + c2 , or equiv—lently c(a + b − 2c) (ab + 2c2)(ab + bc + ca) ≥ 2c(a + b − 2c) (a2 + b2 + c2)(2a2 + 2b2 + ac + bc) , th—t is (a2 + b2 + c2 )(2a2 + 2b2 + ac + bc) ≥ 2(ab + 2c2 )(ab + bc + ca), whi™h is true sin™e a2 + b2 + c2 ≥ ab + bc + ca —nd 2a2 + 2b2 + ac + bc ≥ 2(ab + 2c2 ). honeF RVF vet a, b, c ˜e positive re—l num˜er F €rove th—tX (c) 2( 1 a2 + 8bc + 1 b2 + 8ca + 1 c2 + 8ab ) ≥ 1 ab + bc + ca + 1 a2 + b2 + c2 SolutionX ‚epl—™ing a, b, c ˜y 1 a , 1 b , 1 c respe™tivelyD ‡e ™—n rewrite our inequ—lity —s 4(a + b + c) a 8a2 + bc + b 8b2 + ca + c 8c2 + ab ≥ 2 + 2abc(a + b + c) a2b2 + b2c2 + c2a2 . xowD —ssume th—t c = mina, b, cD then ‡e h—ve the following estim—tionsX a(4a + 4b + c) 8a2 + bc + b(4a + 4b + c) 8b2 + ca − 2 = (a − b)2 (32ab − 12ac − 12bc + c2 ) (8a2 + bc)(8b2 + ca) ≥ 0, —nd 2abc(a + b + c) a2b2 + b2c2 + c2a2 ≤ 2c(a + b + c) ab + 2c2 . ‡ith these estim—tionsD ‡e ™—n redu™e our inequ—lity to 3ac 8a2 + bc + 3bc 8b2 + ca + 4c(a + b + c) 8c2 + ab ≥ 2c(a + b + c) ab + 2c2 , or 3a 8a2 + bc + 3b 8b2 + ca ≥ 2(a + b + c)(4c2 − ab) (ab + 2c2)(ab + 8c2) . e™™ording to g—u™hy ƒ™hw—rz inequ—lityD ‡e h—ve 3a 8a2 + bc + 3b 8b2 + ca ≥ 12 8(a + b) + c a b + b a . it suffi™es to show th—t 6 8(a + b) + c a b + b a ≥ (a + b + c)(4c2 − ab) (ab + 2c2)(ab + 8c2) . if 4c2 ≤ abD then it is trivi—lF ytherwiseD ‡e h—ve a + b ≤ c + ab c , —nd a b + b a ≤ ab c2 + c2 ab . ‡e need to prove 6 8 c + ab c + c ab c2 + c2 ab ≥ 2c + ab c (4c2 − ab) (ab + 2c2)(ab + 8c2) , RH
  • 40. whi™h isD —fter exp—ndingD equiv—lent to (9ab − 4c2 )(ab − c2 )2 c(ab + 8c2)(c4 + 8abc2 + 9a2b2) ≥ 0, whi™h is true —s c = mina, b, c. yur Solution is ™ompletedF RWF vet a, b, c > 0F€rove th—tX 5 3 ( 1 4a2 + bc + 1 4b2 + ca + 1 4c2 + ab ) ≥ 2 ab + bc + ca + 1 a2 + b2 + c2 SolutionX essume th—t c = min(a, b, c)D then ‡e h—ve the following estim—tionsX 1 4a2 + bc + 1 4b2 + ca − 4 8ab + ac + bc = (a − b)2 (32ab − 12ac − 12bc + c2 ) (4a2 + bc)(4b2 + ca)(8ab + ac + bc) ≥ 0, —nd 1 a2 + b2 + c2 ≤ 1 2ab + c2 . …sing theseD ‡e ™—n redu™e our inequ—lity to 20 8ab + ac + bc + 5 ab + 4c2 ≥ 6 ab + ac + bc + 3 2ab + c2 . henote x = a + b ≥ 2 √ ab then this inequ—lity ™—n ˜e rewritten —s f(x) = 20 cx + 8ab − 6 cx + ab + 5 ab + 4c2 − 3 2ab + c2 ≥ 0. ‡e h—ve f (x) = 6c (cx + ab)2 − 20 (cx + 8ab)2 ≥ 20c (cx + ab)(cx + 8ab) − 20c (cx + 8ab)2 = 140abc (cx + ab)(cx + 8ab)2 ≥ 0. „his shows th—t f(x) is in™re—singD —nd ‡e just need to prove th—t f(2 √ ab) ≥ 0, whi™h is equiv—lent to 7c(13t2 + 6tc + 8c2 )(t − c)2 t(t + 2c)(4t + c)(2t2 + c2)(t2 + 4c2) ≥ 0, ‡here t = √ ab „his is o˜viously nonneg—tiveD so our Solution is ™ompletedF SHF vet a, b —nd c re—l num˜ers su™h th—t a + b + c + d = e = 0F €rove th—tX 30(a4 + b4 + c4 + d4 + e4 ) ≥ 7(a2 + b2 + c2 )2 SolutionX xoti™e th—t there exitst three num˜ers —mong a, b, c, d, e h—vinh the s—me singF vet these num˜er ˜e a, b, c, d, e F‡ithout loss of gener—lityD‡e m—y —ssume th—t a, b, c ≥ 0@it notD‡e ™—n t—ke −1, −b, −cAF xow Dusing the g—u™hyEƒ™h—wrz inequ—lityD‡e h—veX [(9(a4 +b4 +c4 )+2(d4 +e4 ))+7d4 +7e4 )](84+63+63) ≥ [2 21(9(a4 + b4 + c4) + 2(d4 + e4))+21d2 +21e2 ]2 . RI
  • 41. end thusDit suffi™es to prove th—tX 2 9(a4 + b4 + c4) + 2(d4 + e4)) ≥ √ 21(a2 + b2 + c2 ). yr 36(a4 + b4 + c4 ) + 8(d4 + e4 ) ≥ 21(a2 + b2 + c2 )2 . ƒin™e d4 + e4 ≥ (d2 + e2 )2 2 ≥ (d + e)4 8 = (a + b + c)4 8 , it is enough to show th—t 36(a4 + b+ c4 ) + (a + b + c + d)4 ≥ 21(a2 + b2 + c2 )2 ‡hi™h is true —nd it is e—sy to proveF SIF vet a, b, c > 0F€rove th—tX a(b + c) a2 + bc + b(c + a) b2 + ca + c(a + b) c2 + ab ≤ 1 2 27 + (a + b + c) 1 a + 1 b + 1 c Solution „he inequ—lity is equiv—lent to a2 (b + c)2 (a2 + bc)2 + 2 ab(b + c)(c + a) (a2 + bc)(b2 + ca) ≤ 15 2 + 1 4 b + c a xoti™e th—t (a2 + bc)(b2 + ca) − ab(b + c)(c + a) = c(a + b)(a − b)2 then 2 ab(b + c)(c + a) (a2 + bc)(b2 + ca) ≤ 6 @IA yther h—ndD a2 (b + c)2 (a2 + bc)2 ≤ a2 (b + c)2 4a2bc = 1 4 b c + c b + 2 @PA prom @IA —nd @PA ‡e h—ve done3 fesidesD ˜y the s—m w—ysD ‡e h—ve — ni™e Solution for —n old pro˜lemX a(b + c) a2 + bc ≤ √ a 1 √ a SPF por —ny positive re—l num˜ers a, b —nd cD a(b + c) a2 + bc + b(c + a) b2 + ca + c(a + b) c2 + ab ≤ √ a + √ b + √ c 1 √ a + 1 √ b + 1 √ c SolutionX ‡e h—ve the inequ—lity is equiv—lent to a(b + c) a2 + bc 2 ≤ √ a 1 √ a RP
  • 42. <=> a(b + c) a2 + bc + 2 ab(a + c)(b + c) (a2 + bc)(b2 + ca) ≤ √ a 1 √ a ‡e ™—n e—sily prove th—t ab(a + c)(b + c) (a2 + bc)(b2 + ca) ≤ 3 ƒoD it suffi™es to prove th—t <=> a(b + c) a2 + bc + 6 ≤ √ a 1 √ a „o prove this ineqD ‡e only need to prove th—t a + b √ ab − c(a + b) c2 + ab − 1 ≥ 0 fut this is trivi—lD ˜e™—use a + b √ ab − c(a + b) c2 + ab − 1 = (a + b) 1 √ ab − c c2 + ab − 1 ≥ 2 √ ab 1 √ ab − c c2 + ab − 1 = c − √ ab 2 c2 + ab ≥ 0 ‡e —re doneF SQF vet a, b, c > 0F €rove th—tX (a + b + c)3 3abc + ab2 + bc2 + ca2 a3 + b3 + c3 ≥ 10 Solution a3 + b3 + c3 3abc + ab2 + bc2 + ca2 a3 + b3 + c3 + (a + b)(b + c)(c + a) abc ≥ 10 …sing ewEqw9s inequ—lity D‡e h—veX a3 + b3 + c3 3abc + ab2 + bc2 + ca2 a3 + b3 + c3 ≥ 2 ab2 + bc2 + ca2 3abc ≥ 2 (a + b)(b + c)(c + a) abc ≥ 8 SRF in —ny tri—ngle efg show th—t ama + bmb + cmc ≤ √ bcma + √ camb + √ abmc SolutionX ‡e h—ve to prove the inequ—lity ama + bmb + cmc ≤ √ bcma + √ camb + √ abmc D where maD mbD mc —re the medi—ns of — tri—ngle efgF ƒin™e 2bc b+c ≤ √ bcD 2ca c+a ≤ √ ca —nd 2ab a+b ≤ √ ab RQ
  • 43. ˜y the rwEqw inequ—lityD it will ˜e enough to show the stronger inequ—lity ama + bmb + cmc ≤ 2bc b + c ma + 2ca c + a mb + 2ab a + b mc D sin™e then ‡e will h—ve ama + bmb + cmc ≤ 2bc b + c ma + 2ca c + a mb + 2ab a + b mc ≤ √ bcma + √ camb + √ abmc —nd the initi—l inequ—lity will ˜e provenF ƒo in the followingD ‡e will ™on™entr—te on proving this stronger inequ—lityF fe™—use the inequ—lity ‡e h—ve to prove is symmetri™D ‡e ™—n ‡vyq —ssume th—t a ≥ b ≥ cF „henD ™le—rlyD bc ≤ ca ≤ abF yn the other h—ndD using the formul—s m2 a = 1 4 2b2 + 2c2 − a2 —nd m2 b = 1 4 2c2 + 2a2 − b2 D ‡e ™—n get —s — result of — str—ightforw—rd ™omput—tionF ma b + c 2 − mb c + a 2 = 3ac + 3bc + a2 + b2 + 4c2 (a + b − c) (b − a) 4 (b + c) 2 (c + a) 2 xowD the fr—™tion on the right h—nd side is ≤ 0D sin™e 3ac + 3bc + a2 + b2 + 4c2 ≥ 0 @this is trivi—lAD a + b − c > 0 @in f—™tD a + b > c ˜e™—use of the tri—ngle inequ—lityA —nd b − a ≤ 0 @sin™e a ≥ bAF ren™eD ma b + c 2 − mb c + a 2 ≤ 0 wh—t yields ma b+c 2 ≤ mb c+a 2 —nd thus ma b+c ≤ mb c+a F ƒimil—rlyD using b ≥ cD ‡e ™—n find mb c+a ≤ mc a+b F „husD ‡e h—ve ma b + c ≤ mb c + a ≤ mc a + b ƒin™e ‡e h—ve —lso bc ≤ ca ≤ abD the sequen™es ma b + c ; mb c + a ; mc a + b —nd (bc; ca; ab) —re equ—lly sortedF „husD the ‚e—rr—ngement inequ—lity yields ma b + c · bc + mb c + a · ca + mc a + b · ab ≥ ma b + c · ca + mb c + a · ab + mc a + b · bc —nd ma b + c · bc + mb c + a · ca + mc a + b · ab ≥ ma b + c · ab + mb c + a · bc + mc a + b · ca ƒumming up these two inequ—litiesD ‡e get 2 ma b + c · bc + 2 mb c + a · ca + 2 mc a + b · ab RR
  • 44. ≥ ma b + c · (ca + ab) + mb c + a · (ab + bc) + mc a + b · (bc + ca) „his simplifies to 2bc b + c ma + 2ca c + a mb + 2ab a + b mc ≥ ma b + c · a (b + c) + mb c + a · b (c + a) + mc a + b · c (a + b) iF eF to 2bc b + c ma + 2ca c + a mb + 2ab a + b mc ≥ ama + bmb + cmc „husD ‡e h—ve ama + bmb + cmc ≤ 2bc b + c ma + 2ca c + a mb + 2ab a + b mc —nd the Solution is ™ompleteF xote th—t in e—™h of the inequ—lities ama + bmb + cmc ≤ √ bcma + √ camb + √ abmc —nd ama + bmb + cmc ≤ 2bc b + c ma + 2ca c + a mb + 2ab a + b mc equ—lity holds only if the tri—ngle efg is equil—ter—lF SSF por aD bD c positive re—ls prove th—t a2 + 3 b2 + 3 c2 + 3 ≥ 4 3 3 √ abc (ab + bc + ca) SolutionX hivide abc for ˜oth term —nd t—ke x = bc a ; y = ac b ; z = ab c —nd ‡e must prove th—tX (xy + 3 xy ) ≥ (4 3 )3 (x + y + z) xote th—tX LHS ≥ 3(x2 +y2 +z2 )+x2 y2 z2 + 4 x2y2z2 ≥ (x+y+z)2 +4 ≥ 4(x+y+z) ≥ ( 4 3 )3 (x+y+z). STF veta, b, c > 0 F€rove th—tX a + b c √ a2 + b2 + b + c a √ b2 + c2 + c + a b √ c2 + a2 ≥ 3 √ 6 √ a2 + b2 + c2 . Solution IFFFeltern—tivelyD using ghe˜yshev —nd g—u™hyD cycl a + b c √ a2 + b2 ≥ 2(a + b + c) 3 · 9 cycl c √ a2 + b2 = 6(a + b + c) cycl c √ a2 + b2 —nd cycl c a2 + b2 ≤ a + b + c 3 cycl a2 + b2 ≤ a + b + c 3 6(a2 + b2 + c2) gom˜ining ‡e get the desired resultF SUF vet a, b, c > 0 su™h th—t a2 + b2 + c2 + abc = 4 €rove th—t a2 b2 + b2 c2 + c2 a2 ≤ a2 + b2 + c2 RS
  • 45. SolutionX vet a = 2 yz (x+y)(x+z) , b = 2 xz (x+y)(y+z) —nd c = 2 xy (x+z)(y+z) , where x, y —nd z —re positive num˜ers @ e—sy to ™he™k th—t it exists AF „husD it rem—ins to prove th—t cyc xy (x + z)(y + z) ≥ cyc 4x2 yz (x + y)(x + z)(y + z)2 , whi™h equiv—lent to cyc(x4 y2 +x4 z2 −2x4 yz+2x3 y3 −2x2 y2 z2 ) ≥ 0, whi™h true ˜y ewEqwF SVF vet a, b, c > 0 su™h th—t a + b + c = 1F€rove th—t b2 a + b2 + c2 b + c2 + a2 c + a2 ≥ 3 4 Solution ‡e h—ve b2 a + b2 + c2 b + c2 + a2 c + a2 ≥ a2 + b2 + c2 2 (a4 + b4 + c4) + (ab2 + bc2 + ca2) ren™e it suffi™es to prove th—t a2 + b2 + c2 2 (a4 + b4 + c4) + (ab2 + bc2 + ca2) ≥ 3 4 ⇔ 4 a2 2 ≥ 3 ab2 a + 3 a4 ⇔ 4 a4 + 8 a2 b2 ≥ 3 a4 + 3 a2 b2 + abc2 + a3 c ⇔ a4 + 5 a2 b2 ≥ 3abc a + 3 a3 c ƒin™e ‡e —lw—ys h—ve 3 a3 c + b3 a + c3 b ≤ a2 + b2 + c2 2 = a4 + b4 + c4 + 2 a2 b2 + b2 c2 + c2 a2 „herefor it suffi™es to prove th—t 3 a2 b2 + b2 c2 + c2 a2 ≥ 3abc (a + b + c) whi™h o˜viously trueF SWF vet a; b; c > 0F €rove th—t ab+c + ba+c + ca+b ≥ 1 Solution if a ≥ 1 or b ≥ 1 or c ≥ 1 then the inequ—lity is true if 0 ≤ a, b, c ≤ 1 then suppose c = mina, b, c C if a + b < 1 ‡e h—ve b + c < 1 Dc + a < 1 RT
  • 46. epply fernoull‡e 9 inequ—lity ( 1 a )b+c ) = (1 + 1 − a a )b+c < 1 + (b + c)(1 − a) a < a + b + c a „herefore ab+c > a a+b+c ƒimil—r for bc+a —nd ca+b dedu™e ab+c + ba+c + ca+b > 1 C if a + b > 1 then ab+c + ba+c + ca+b > ab+c + ba+c ≥ aa+b + ba+b epply fernoull‡e 9 inequ—lity ‡e h—ve X haa+b = (1 + (a − 1))a+b > 1 + (a + b)(a − 1) ƒimil—r forba+b when™e ab+c + ba+c + ca+b > 2 + (a + b)(a + b − 2) = (a + b − 1)2 + 1 ≥ 1 THF vet a, b, c ˜e the sidelengths of tri—ngle with perimeter 2 (⇒ a + b + c = 2). €rove th—t a3 b + b3 c + c3 a − a3 c − b3 a − c3 b < 3 SolutionX „his ineq is equiv—lent toX |a4 c + c4 b + b4 a − a4 b − bc − c4 a| ≤ 3abc <=> |(a − b)(b − c)(c − a)(a2 + b2 + c2 + ab + bc + ca)| ≤ 3abc fy ‚—v‡e ƒu˜stitution D denoteX a = x + y, b = y + z, c = z + xD so x + y + z = 1D this ineq ˜e™omesX |(x − y)(y − z)(z − x)(3(x2 + y2 + z2 ) + 5(xy + yz + zx)| ≤ 3(x + y)(y + z)(z + x) i—sy to see th—t |(x − y)(y − z)(z − x)| ≤ (x + y)(y + z)(z + x) ƒo ‡e need to prove (3(x2 + y2 + z2 ) + 5(xy + yz + zx) ≤ 3 = 3(x + y + z)2 <=> xy + yz + zx ≥ 0 whi™h is o˜vious true FiFh F TIF qiven xDyDzbHF€rove th—t x(y + z)2 2x + y + z + y(x + z)2 x + 2y + z + z(x + y)2 x + y + 2z = (3xyz(x + y + z)) SolutionX cyc x(y + z)2 2x + y + z − 3xyz(x + y + z) = = cyc x(y + z)2 2x + y + z − yz + xy + xz + yz − 3xyz(x + y + z) = = cyc z2 (x − y) − y2 (z − x) 2x + y + z + cyc z2 (x − y)2 2 xy + xz + yz + 3xyz(x + y + z) = RU
  • 47. = cyc (x − y) z2 2x + y + z − z2 2y + x + z + + cyc z2 (x − y)2 2 xy + xz + yz + 3xyz(x + y + z) = = cyc (x − y)2   z2 2 xy + xz + yz + 3xyz(x + y + z) − z2 (2x + y + z)(2y + x + z)   . „husD it rem—ins to prove th—t (2x + y + z)(2y + x + z) ≥ 2 xy + xz + yz + 3xyz(x + y + z) . fut (2x + y + z)(2y + x + z) ≥ 2 xy + xz + yz + 3xyz(x + y + z) ⇔ ⇔ 2x2 + 2y2 + z2 + 3xy + xz + yz ≥ 2 3xyz(x + y + z), whi™h is true ˜e™—use x2 + y2 + z2 ≥ xy + xz + yz ≥ 3xyz(x + y + z). it seems th—t the following inequ—lity is true tooF vet x, y —nd z —re positive num˜ersF €rove th—tX x(y + z)2 3x + 2y + 2z + y(x + z)2 2x + 3y + 2z + z(x + y)2 2x + 2y + 3z ≥ 4 7 3xyz(x + y + z) TPF vet a, b ∈ R su™h th—t 9a2 + 8ab + 7b2 ≤ 6 €rove th—t X7a + 5b + 12ab ≤ 9 SolutionX IFFF fy ewEqw inequ—lityD ‡e see th—t 7a + 5b + 12ab ≤ 7 a2 + 1 4 + 5 b2 + 1 4 + 12ab = (9a2 + 8ab + 7b2 ) − 2(a − b)2 + 3 ≤ (9a2 + 8ab + 7b2 ) + 3 ≤ 6 + 3 = 9. iqu—lity holds if —nd only if a = b = 1 2 . TQF vet x, y —nd z —re positive num˜ers su™h th—t x + y + z = 1 x + 1 y + 1 z . €rove th—t xyz + yz + zx + xy ≥ 4. SolutionX xyz(x + y + z) yz + zx + xy + x + y + z − 4xyz(x + y + z)2 (yz + zx + xy)2 ≥ 3xyz yz + zx + xy + x + y + z − 4xyz(x + y + z)2 (yz + zx + xy)2 = x3 (y − z)2 + y3 (z − x)2 + z3 (x − y)2 (yz + zx + xy)2 ≥ 0 =⇒ 1 + 1 x + 1 y + 1 z ≥ 4 xyz ⇐⇒ xyz + yz + zx + xy ≥ 4 RV
  • 48. TRF vet a, b, ca0 s—tisfy a + b + c = 1 €rove th—t (a2 + b2 )(b2 + c2 )(c2 + a2 )a 1 32 SolutionX let f(a, b, c) = (a2 + b2 )(b2 + c2 )(c2 + a2 ) let c = max(a, b, c); ‡e h—ve f(a, b, c) ≤ f(a + b, 0, c)@whi™h is equiv—lent ab(−4abc2 + a3 b + ab3 − 4a2 c2 − 4b2 c2 − 2c4 ) ≤ 0@trueA ‡e will prove th—t f(a + b, 0, c) = f(1 − c, 0, c) ≤ 1 2 whi™h is equiv—lent to 1 32 ∗ (16c4 − 32c3 + 20c2 − 4c − 1))(−1 + 2c)2 ≤ 0 remem˜er th—t 16c4 − 32c3 + 20c2 − 4c − 1 = 4(2c2 − 2c + 1− √ 5 4 )(2c2 − 2c + 1+ √ 5 4 ) ≥ 0 for every c ∈ [0, 1] TSF vet a, b, c ˜e the sides of tri—ngleF €rove th—tX a 2a − b + c + b 2b − c + a + c 2c − a + b ≥ 3 2 SolutionX the inequ—lity is equiv—lent to 1 1 + a a+c−b ≤ 3 2 fy g—u™hy ‡e h—ve X a a + c − b + 1 ≥ 2 a a + c − b ƒo ‡e need to prove a + c − b a ≤ 3 fe™—use a, b, c ˜e the sides of — tri—ngle so ‡e h—veX a + c − b a = sin A + sin C − sin B sin A = 2 sin C 2 cos B 2 cos A 2 it9s following th—t a + c − b a = cos B 2 √ sin C + cos C 2 √ sin A + cos A 2 √ sin B cos A 2 cos B 2 cos C 2 ≤ cos2 B 2 + cos2 C 2 + cos2 A 2 (sin C + sin A + sin B) cos A 2 cos B 2 cos C 2 = 2 (cos B + cos C + cos A) + 6 ≤ 2. 3 2 + 6 = 3 TTF vet a, b, c, d > 0F€rove the following inequ—lityF‡hen does the equ—lity holdc 3 1 + 5 1 + a + 7 1 + a + b + 9 1 + a + b + c + 36 1 + a + b + c + d 4 1 + 1 a + 1 b + 1 c + 1 d RW
  • 49. SolutionX ‡e ™—n h—ve (1 + a + b + c + d)( 4 25 + 16 25a + 36 25b + 64 25c + 4 d ) ≥ ( 2 5 + 4 5 + 6 5 + 8 5 + 2)2 = 9 so ( 4 25 + 16 25a + 36 25b + 64 25c + 4 d ) ≥ 36 1 1 + a + b + c + d —nd (1 + a + b + c)( 9 100 + 9 25a + 81 100b + 36 25c ) ≥ ( 3 10 + 3 5 + 9 10 + 6 5 ) so ‡e h—ve ( 9 100 + 9 25a + 81 100b + 36 25c ) ≥ 9 1 1 + a + b + c —nd (1 + a + b)( 7 36 + 7 9a + 7 4b ) ≥ 7 ‡e get ( 7 36 + 7 9a + 7 4b ) ≥ 7 1 + a + b —nd (1 + a)( 5 9 + 20 9a ) ≥ 5 then 5 9 + 20 9a ≥ 5 1 + a —nd —dd these inequ—lity up ‡e ™—n solve the pro˜lemF TUF vet —D˜D™ ˜e positive re—l num˜er su™h th—t 9 + 3abc = 4(ab + bc + ca) €rove th—t a + b + c ≥ 3 SolutionX „—ke a = x + 1; b = y + 1; c = z + 1Dthen ‡e must prove th—tX x + y + z ≥ 0 when 5(x + y + z) + xy + xz + yz = 3xyz ‡e ™onsider three ™—seX g—se IXxyz ≥ 0 ⇒ (x+y+z)2 3 + 5(x + y + z) ≥ 5(x + y + z) + xy + xz + yz = 3xyz ≥ 0 ⇒ x + y + z ≥ 0 g—se PX x ≥ 0; y ≥ 0; z ≤ 0 essume th—t x + y + z ≤ 0 ⇒ −x ≥ y + zF 5(x + y + z) = yz(3x − 1) − x(y + z) ≥ −4yz + (y + z)2 = (y − z)2 ≥ 0 g—se QX x ≤ 0; y ≤ 0; z ≤ 0Fy˜serve th—t −x, −y, −z ∈ [0, 1] thenX 0 = 5(x+y+z)+xy+yz+xz−3xyz ≤ 5(x+y+z)+2(xy+xz+yz) ≤ 5(x+y+z)+ 2(x + y + z)2 3 ⇒ x+y+z ≥ 0 ƒo ‡e h—ve doneF TVF if a, b, c, d —re nonEneg—tive re—l num˜ers su™h th—t a + b + c + d = 4D then a2 b2 + 3 + b2 c2 + 3 + c2 d2 + 3 + d2 a2 + 3 ≥ 1. SH
  • 50. ƒyv…„iyxX fy g—u™hyEƒ™hw—rz [a2 (b2 + 3) + b2 (c2 + 3) + c2 (d2 + 3)]( a2 b2 + 3 + b2 c2 + 3 + c2 d2 + 3 + d2 a2 + 3 ) ≥ (a2 + b2 + c2 )2 . prom g—u™hy ‡e see th—t it is suffi™ient to prove th—t (a2 + b2 + c2 + d2 )2 ≥ 3(a2 + b2 + c2 + d2 ) + a2 b2 + b2 c2 + c2 d2 + d2 a2 whi™h ™—n ˜e rewritten —s (a2 + b2 + c2 + d2 )(a2 + b2 + c2 + d2 − 3) ≥ a2 b2 + b2 c2 + c2 d2 + d2 a2 xow you must homogeneize to h—ve r—r—z‡e form (a2 + b2 + c2 + d2 )(a2 + b2 + c2 + d2 − 3( a + b + c + d 4 )2 ) ≥ a2 b2 + b2 c2 + c2 d2 + d2 a2 whi™h follows from a2 + b2 + c2 + d2 ≥ 4 —nd (x + y + z + t)2 ≥ 4(xy + yz + zt + tx) with x = a2 —nd simil—rF TWF if a ≥ 2D b ≥ 2D c ≥ 2 —re re—lsD then prove th—t 8 a3 + b b3 + c c3 + a ≥ 125 (a + b) (b + c) (c + a) ƒyv…„iyxX vets write LHS —s 8 ∗ (a3 b3 c3 + abc + a4 b3 + b4 c3 + c4 a3 + a4 c + b4 a + c4 b) prom the wuirhe—ds inequ—lity ‡e h—ve th—t a4 b3 + b4 c3 + c4 a3 ≥ a3 b2 c2 = a2 b2 c2 (a + b + c) —nd a4 c + b4 a + c4 b ≥ a3 bc = abc(a2 + b2 + c2 ) F(∗∗)Nowletsseethat a2 b2 c2 = (ab)(bc)(ca) ≥ (a + b)(b + c)(c + a)Thisiseasytoprove. Fromthe@BBAWegetthat LHS ≥ 8(a3 b3 c3 + abc + a2 b2 c2 (a + b + c) + abc(a2 + b2 + c2 )) = 8a2 b2 c2 (abc + 1 abc + a + b + c + a2 + b2 + c2 abc ) so ‡e h—ve to prove th—t X abc + 1 abc + a + b + c + a2 + b2 + c2 abc ≥ 125 8 SI
  • 51. or 8a2 b2 c2 + 8 + 8abc(a + b + c) + 8(a2 + b2 + c2 ) ≥ 125abc ‡vyq let a ≥ b ≥ c xow let abc = P ‡e will m—ke th—t following ™h—nge a → a —nd b → b where ≥ 1 —nd a F „he RHS doesnt ™h—ngeF in the LHS the first p—rt —lso doesn9t ™h—ngeF a + b ≥ a + b equiv—lent to ( − 1)(a − b ) whi™h is trueF elso ‡e get th—t a2 + b2 ≥ a2 2 + b2 2 ƒo —s ‡e get the num˜ers ™loser to e—™h other the LHS de™re—ses while the RHS rem—ins the s—me so it is enough to prove the inequ—lity for the ™—se a = b = c whi™h is equiv—lent to X 8a6 + 8 + 24a4 + 24a2 ≥ 125a3 ‡hi™h is pretty e—syF UHD vet a, b, c ≥ 0F €rove th—tX 1 √ 2a2 + ab + bc + 1 √ 2b2 + bc + ca + 1 √ 2c2 + ca + ab ≥ 9 2(a + b + c) . SolutionX ‡e h—veX cyc 1 √ 2a2 + ab + bc = cyc 2a + b + c 2 · 2a+b+c 2 √ 2a2 + ab + bc ≥ ≥ cyc 2a + b + c 2a+b+c 2 2 + 2a2 + ab + bc . fut cyc 2a + b + c 2a+b+c 2 2 + 2a2 + ab + bc ≥ 9 2(a + b + c) ⇔ ⇔ cyc (100a6 + 600a5 b + 588a5 c + 1123a4 b2 − 357a4 c2 − 1842a3 b3 + +1090a4 bc − 1414a3 b2 c + 1330a3 c2 b − 1218a2 b2 c2 ) ≥ 0, whi™h is e—syF UIF vet —D˜D™ b H F€rove th—t X a a + 2b + b b + 2c + c c + 2a ≤ 3(a2 + b2 + c2 ) (a + b + c)2 SP
  • 52. SolutionX ⇐⇒ 1 − a a + 2b ≥ 3 − 3(a2 + b2 + c2 ) (a + b + c)2 = 6(ab + bc + ca) (a + b + c)2 ⇐⇒ b a + 2b + c b + 2c + a c + 2a ≥ 3(ab + bc + ca) (a + b + c)2 fy g—u™hyEƒ™hw—rz ‡e get b a + 2b b(a + 2b) ≥ (a + b + c)2 it suffi™e to show th—t (a + b + c)4 ≥ 3(ab + bc + ca)(ab + bc + ca + 2a2 + 2b2 + 2c2 ) ‡ithout loss of generosityD—ssume th—t ab + bc + ca = 3Dthen it ˜e™omes (a + b + c)2 − 9 2 ≥ 0 whi™h is o˜viousF UPF vet a, b, c, d ˜e positive re—l num˜ersF €rove th—t the following inequ—lity holds a4 + b4 (a + b)(a2 + ab + b2) + b4 + c4 (b + c)(b2 + bc + c2) + c4 + d4 (c + d)(c2 + cd + d2) + d4 + a4 (d + a)(d2 + da + a2) ≥ a2 + b2 + c2 + d2 a + b + c + d SolutionX a4 + b4 (a + b)(a2 + ab + b2) ≥ 1 2 (a2 + b2 )2 (a + b)(a2 + ab + b2) „hus D it rem—ins to prove th—t cyc (a2 + b2 )2 (a + b)(a2 + ab + b2) ≥ 2(a2 + b2 + c2 + d2 ) a + b + c + d e™ording to g—u™hyEƒhw—rz inequ—lity ‡e h—ve X LHS ≥ 4(a2 + b2 + c2 + d2 )2 A where A = cyc(a + b)(a2 + ab + b2 ) = 2(a3 + b3 + c3 + d3 ) + 2 cyc ab(a + b) it suffi™es to show th—t 2(a2 + b2 + c2 + d2 )(a + b + c + d) ≥ 2(a3 + b3 + c3 + d3 ) + 2 cyc ab(a + b) ⇔ cyc 2a3 + 2a2 b + 2a2 c + 2a2 d − 2a3 − 2ab(a + b) = cyc 2a2 c ≥ 0 UQF if a, b, c —re nonneg—tive re—l num˜ersD then a a2 + 4b2 + 4c2 ≥ (a + b + c)2 . SolutionF in the nontrivi—l ™—se when two of —D˜D™ —re nonzeroD ‡e t—ke squ—re ˜oth sides —nd write SQ
  • 53. the inequ—lity —s a2 (a2 + 4b2 + 4c2 ) + 2 ab (a2 + 4b2 + 4c2)(4a2 + b2 + 4c2) ≥≥ ( a) 4 . epplying the g—u™hyEƒ™hw—rz inequ—lity in ™om˜in—tion with the trivi—l inequ—lity (u + 4v)(v + 4u) ≥ 2u + 2v + 2uv u + v ∀u, v ≥ 0, u + v > 0, ‡e get ab (a2 + 4b2 + 4c2)(4a2 + b2 + 4c2) ≥ ab (a2 + 4b2)(b2 + 4a2) + 4c2 ≥ ab 2a2 + 2b2 + 2a2 b2 a2 + b2 + 4c2 . „hereforeD it suffi™es to prove th—t a4 + 8 a2 b2 + 4 ab(a2 + b2 ) + 8abc a + 4 a3 b3 a2+b2 ≥ ≥ ( a) 4 . „his inequ—lity redu™es toF URF vet —D˜D™ ˜e nonneg—tive re—l num˜ersD no two of whi™h —re zeroF €rove th—t b2 + c2 a2 + bc + c2 + a2 b2 + ca + a2 + b2 c2 + ab ≥ 2a b + c + 2b c + a + 2c a + b . SolutionF fy the g—u™hyEƒ™hw—rz inequ—lityD ‡e h—ve b2 + c2 a2 + bc ≥ (b2 + c2 ) 2 (b2 + c2)(a2 + bc) = 4(a2 + b2 + c2 )2 ab(a2 + b2) + 2 a2b2 . „hereforeD it suffi™es to prove th—t 2(a2 + b2 + c2 )2 ≥ a ab(a2 + b2 ) + 2 a2 b2 b + c .4 ƒin™e ab(a2 + b2 ) + 2 a2 b2 = = (b + c)[a3 + 2a2 (b + c) + bc(b + c) + a(b2 − bc + c2 )] − 4a2 bc, this inequ—lity ™—n ˜e written —s 2 a2 2 + 4abc a2 b + c ≥ ≥ a[a3 + 2a2 (b + c) + bc(b + c) + a(b2 − bc + c2 )], or equiv—lentlyD a4 + 2 a2 b2 + 4abc a2 b + c ≥ abc a + 2 a3 (b + c). xowD ˜y ghe˜yshev9s inequ—lityD ‡e h—ve a2 b + c ≥ 3(a2 + b2 + c2 ) 2(a + b + c) , SR
  • 54. —nd thusD it suffi™es to show th—t a4 + 2 a2 b2 + 6abc a2 a ≥ abc a + 2 a3 (b + c). efter some simple ™omput—tionsD ‡e ™—n write this inequ—lity —s a3 (a − b)(a − c) + b3 (b − c)(b − a) + c3 (c − a)(c − b) ≥ 0, whi™h is ƒ™hur9s inequ—lityF „he Solution is ™ompletedF iqu—lity holds if —nd only if a = b = c, ora = b —nd c = 0, or —ny ™y™li™ permut—tionFF a2 b2 + 2 a3 b3 a2 + b2 ≥ 2abc a, or a2 b2 (a + b)2 a2 + b2 ≥ 2abc a. fy the g—u™hyEƒ™hw—rz inequ—lityD ‡e h—ve a2 b2 (a + b)2 a2 + b2 ≥ [ab(a + b) + bc(b + c) + ca(c + a)]2 (a2 + b2) + (b2 + c2) + (c2 + a2) , —nd thusD it is enough to to ™he™k th—t [ab(a + b) + bc(b + c) + ca(c + a)]2 ≥ 4abc(a + b + c)(a2 + b2 + c2 ). ‡ithout loss of gener—lityD —ssume th—t b is ˜etien a—nd cF prom the ewEqw inequ—lityD ‡e h—ve 4abc(a + b + c)(a2 + b2 + c2 ) ≤ [ac(a + b + c) + b(a2 + b2 + c2 )]2 . yn the other h—ndD one h—s ac(a + b + c) + b(a2 + b2 + c2 ) − [ab(a + b) + bc(b + c) + ca(c + a)] = = −b(a − b)(b − c) ≤ 0. gom˜ining these two inequ—litiesD the ™on™lusion followsF iqu—lity o™™urs if —nd only if —a˜a™D or a = b = 0, orb = c = 0, orc = a = 0. USF vet a, b, c ˜e positive re—l num˜erF €rove th—tX 2(b + c) a ≥ 27(a + b)(b + c)(c + a) 4(a + b + c)(ab + bc + ca) SolutionF fy ewEqw inequ—lity ‡e h—ve 1 a 2(a2 + bc) = √ b + c √ 2a. (ab + ac)(a2 + bc) ≥ 2(b + c) √ a.(a + b)(a + c) it suffi™es to show th—t (b + c) 2(b + c) a ≥ 9(a + b)(b + c)(c + a) 2(ab + bc + ca) fy ghe˜yselv inequ—lity ‡e h—ve (b + c) 2(b + c) a ≥ 2 3 (a + b + c). 2(b + c) a SS
  • 55. ren™eD it suffi™es to show th—t 2(b + c) a ≥ 27(a + b)(b + c)(c + a) 4(a + b + c)(ab + bc + ca) fy g—u™hyEƒ™hw—rz inequ—lityD ‡e get 2(b + c) a 2 a(b + c)2 ≥ 16(a + b + c)3 end ˜y ewEqw inequ—lityD 27(a + b)(b + c)(c + a) ≤ 8(a + b + c)3 pin—llyD ‡e need to show th—t 16(a + b + c)3 a(b + c)2 ≥ 27.8(a + b + c)3 (a + b)(b + c)(c + a) 16(a + b + c)2(ab + bc + ca)2 or 32(a + b + c)2 (ab + bc + ca)2 ≥ 27(a + b)(b + c)(c + a) ((a + b)(b + c)(c + a) + 4abc) or 5x2 + 32y2 ≥ 44xy where ‡e setting x = (a + b)(b + c)(c + a)D y = abc —nd using the equ—lity (a + b + c)(ab + bc + ca) = x + y „he l—st inequ—lity is true ˜e™—use it equiv—lent (x − 8y)(5x − 4y) ≥ 0D o˜viouslyF UTF if —D˜D™ —re positive re—l num˜ersD then 1 a 2(a2 + bc) ≥ 9 2(ab + bc + ca) . pirst SolutionF fy rolder9s inequ—lityD ‡e h—ve 1 a √ a2 + bc 2 a2 + bc a ≥ 1 a 3 . it follows th—t 1 a √ a2 + bc 2 ≥ (ab + bc + ca)3 a2 b2 c2 a2 b2 + a2 bc , —nd hen™eD it suffi™es to prove th—t 2(ab + bc + ca)5 ≥ 81a2 b2 c2 a2 b2 + a2 bc . ƒetting x = bc, y = ca —nd z = abD this inequ—lity ˜e™omes 2(x + y + z)5 ≥ 81xyz(x2 + y2 + z2 + xy + yz + zx). …sing the illEknown inequ—lity xyz ≤ (x + y + z)(xy + yz + zx) 9 , ST
  • 56. ‡e see th—t it is enough to ™he™k th—t 2(x + y + z)4 ≥ 9(xy + yz + zx)(x2 + y2 + z2 + xy + yz + zx), whi™h is equiv—lent to the o˜vious inequ—lity (x2 + y2 + z2 − xy − yz − zx)(2x2 + 2y2 + 2z2 + xy + yz + zx) ≥ 0. „he Solution is ™ompletedF iqu—lity holds if —nd only if a = b = c. ƒe™ond SolutionF fy the ewEqw inequ—lityD ‡e h—ve 1 a 2(a2 + bc) = √ b + c √ 2a (ab + ac)(a2 + bc) ≥ 2(b + c) √ a(a + b)(a + c) . „hereforeD it suffi™es to prove th—t b + c 2a · 1 (a + b)(a + c) ≥ 9 4(ab + bc + ca) . ‡ithout loss of gener—lityD —ssume th—t a ≥ b ≥ cF ƒin™e b + c 2a ≤ c + a 2b ≤ a + b 2c —nd 1 (a + b)(a + c) ≤ 1 (b + c)(b + a) ≤ 1 (c + a)(c + b) , ˜y ghe˜yshev9s inequ—lityD ‡e get b + c 2a · 1 (a + b)(a + c) ≥ 1 3 b + c 2a 1 (a + b)(a + c) = 2(a + b + c) 3(a + b)(b + c)(c + a) b + c 2a . ƒoD it suffi™es to show th—t b + c 2a ≥ 27(a + b)(b + c)(c + a) 8(a + b + c)(ab + bc + ca) . ƒetting t = 6 (a + b)(b + c)(c + a) 8abc t ≥ 1. fy the ewEqw inequ—lityD ‡e h—ve b + c 2a ≥ 3t. elsoD it is e—sy to verify th—t 27(a + b)(b + c)(c + a) 8(a + b + c)(ab + bc + ca) = 27t6 8t6 + 1 . ƒoD it is enough to ™he™k th—t 3t ≥ 27t6 8t6 + 1 , SU
  • 57. or 8t6 − 9t5 + 1 ≥ 0. ƒin™e t ≥ 1D this inequ—lity is true —nd the Solution is ™ompletedF UTF qive a1, a2, ..., an ≥ 0—re num˜ers h—ve sum is IF €rove th—t if n > 3 so a1a2 + a2a3 + ... + ana1 ≤ 1 4 SolutionX vet n = 2kD where k ∈ N —nd a1 + a3 + ... + a2k−1 = xF ren™eD a1a2 + a2a3 + ... + ana1 ≤ (a1 + a3 + ... + a2k−1) (a2 + a4 + ... + a2k) = = x(1 − x) ≤ 1 4 vet n = 2k − 1 —nd a1 = mini{ai}F ren™eD a1a2 + a2a3 + ... + ana1 ≤ ≤ a1a2 + a2a3 + ... + ana2 ≤ (a1 + a3 + ... + a2k−1) (a2 + a4 + ... + a2k−2) = = x(1 − x) ≤ 1 4 UUF vet a, b, c ˜e nonEneg—tive re—l num˜ersF €rove th—t a3 + 2abc a3 + (b + c)3 + b3 + 2abc b3 + (c + a)3 + c3 + 2abc c3 + (a + b)3 ≥ 1 Solution ‡e h—ve 1− a3 a3 + (b + c)3 = a3 a3 + b3 + c3 − a3 a3 + (b + c)3 = 3a3 bc(b + c) (a3 + b3 + c3)(a3 + (b + c)3) ren™eD it suffi™es to show th—t 2 a3 + b3 + c3 a3 + (b + c)3 ≥ 3a2 (b + c) a3 + (b + c)3 ⇔ 2a3 − 3a2 (b + c) + 2b3 + 2c3 a3 + (b + c)3 ≥ 0 ⇔ (a − b)(a2 − 2ab − 2b2 ) + (a − c)(a2 − 2ac − 2c2 ) a3 + (b + c)3 ≥ 0 ⇔ (a − b)3 − (c − a)3 + 3c2 (c − a) − 3b2 (a − b) a3 + (b + c)3 ≥ 0 it suffi™es to show th—t c2 (c − a) − b2 (a − b) a3 + (b + c)3 ≥ 0 —nd (a − b)3 − (c − a)3 a3 + (b + c)3 ≥ 0 SV
  • 58. „he first inequ—lity is equiv—lent to ⇔ (a − b) a2 b3 + (c + a)3 − b2 a3 + (b + c)3 ≥ 0 pin—llyD to finish the SolutionD ‡e will show th—t if a ≥ bD then a2 b3 + (c + a)3 ≥ b2 a3 + (b + c)3 ⇔ a5 − b5 ≥ b2 (c + a)3 − a2 (b + c)3 ⇔ a5 − b5 ≥ a2 b2 (a − b) + c3 (b2 − a2 ) + 3c2 ab(b − a) whi™h is o˜viously true sin™e a ≥ b —nd c ≥ 0F end the se™ond inequ—lity is equiv—lent to (a − b)3 1 a3 + (b + c)3 − 1 b3 + (c + a)3 ≥ 0 ⇔ (a − b)4 (3c(a + b) + 3c2 ) (a3 + (b + c)3)(b3 + (c + a)3) ≥ 0 whi™h is o˜viously trueF iqu—lity holds for a = b = c or abc = 0 UVF vet a, b, c —re positive re—l num˜ersD prove th—t 3 a b + b c + c a + 2 ab + bc + ca a2 + b2 + c2 ≥ 5 SolutionX fy using the ill known 2 a b + b c + c a + 1 ≥ 21 a2 + b2 + c2 (a + b + c) 2 ƒetting x = ab+bc+ca a2+b2+c2 ≤ 1F it suffi™es to show th—t 3 (10 − x2) 2x2 + 1 + 2x ≥ 5 ⇔ 3 10 − x2 2x2 + 1 ≥ 4x2 − 20x + 25 ⇔ −8x4 + 40x3 − 57x2 + 20x + 5 2x2 + 1 ≥ 0 ⇔ (x − 1) −8x3 + 32x2 − 25x − 5 2x2 + 1 ≥ 0 whi™h is ™le—rly trueF a b + b c + c a 3 a b + b c + c a 3 ab + bc + ca a2 + b2 + c2 ≥ 1 end ‡e —lso note th—t the folloid is not true a b + b c + c a 3 ab + bc + ca a2 + b2 + c2 ≥ 1 SW
  • 59. UWF vet a, b, c ˜e the sideElengths of — tri—ngle su™h th—t a2 + b2 + c2 = 3. €rove th—t bc 1 + a2 + ca 1 + b2 + ab 1 + c2 ≥ 3 2 . SolutionF ‡rite the inequ—lity —s 2bc 4a2 + b2 + c2 ≥ 1. ƒin™e 1 = b2 c2 a2b2+b2c2+c2a2 , this inequ—lity is equiv—lent to 2bc 4a2 + b2 + c2 − b2 c2 a2b2 + b2c2 + c2a2 ≥ 0, or abc a2b2 + b2c2 + c2a2 (2a2 − bc)(b − c)2 a(4a2 + b2 + c2) ≥ 0. ‡ithout loss of gener—lityD —ssume th—t a ≥ b ≥ c. ƒin™e (2a2 − bc)(b − c)2 a(4a2 + b2 + c2) ≥ 0, it suffi™es to prove th—t (2b2 − ca)(c − a)2 b(4b2 + c2 + a2) + (2c2 − ab)(a − b)2 c(4c2 + a2 + b2) ≥ 0. ƒin™e a, b, c —re the sideElengths of — tri—ngle —nd a ≥ b ≥ c, ‡e h—ve 2b2 − ca ≥ c(b + c) − ca = c(b + c − a) ≥ 0, —nd a − c − b c (a − b) = (b − c)(b + c − a) c ≥ 0. „hereforeD (2b2 − ca)(c − a)2 b(4b2 + c2 + a2) ≥ b(2b2 − ca)(a − b)2 c2(4b2 + c2 + a2) . it suffi™es to show th—t b(2b2 − ca) 4b2 + c2 + a2 + c(2c2 − ab) 4c2 + a2 + b2 ≥ 0, or b(2b2 − ca) 4b2 + c2 + a2 ≥ c(ab − 2c2 ) 4c2 + a2 + b2 . ƒin™e ab − 2c2 − (2b2 − ca) = a(b + c) − 2(b2 + c2 ) ≤ a(b + c) − (b + c)2 ≤ 0, it is enough to ™he™k th—t b 4b2 + c2 + a2 ≥ c 4c2 + a2 + b2 , whi™h is true ˜e™—use b(4c2 + a2 + b2 ) − c(4b2 + c2 + a2 ) = (b − c)[(b − c)2 + (a2 − bc)] ≥ 0. TH
  • 60. „he Solution is ™ompletedF VHF vet a, b, c, d > 0 su™h th—t a2 + b2 + c2 + d2 = 4Dthen 1 a + 1 b + 1 c + 1 d ≤ 2 + 2 abcd Solution write the inequ—lity —s abc + bcd + cda + dab ≤ 2abcd + 2. ‡ithout loss of gener—lityD —ssume th—t a ≥ b ≥ c ≥ d. vet t = a2 + b2 2 , => 1 ≤ t ≤ √ 2. ƒin™e 2 1 c + 1 d ≥ 1 a + 1 b + 1 c + 1 d ≥ 16 a + b + c + d ≥ 16 4(a2 + b2 + c2 + d2) = 4, ‡e h—vec + d ≥ 2cd. „hereforeD abc + bcd + cda + dab − 2abcd = ab(c + d − 2cd) + cd(a + b) ≤ a2 + b2 2 (c + d − 2cd) + cd 2(a2 + b2) = t2 (c + d − 2cd) + 2tcd. it suffi™es to prove th—t t2 (c + d − 2cd) + 2tcd ≤ 2, or 2tcd(1 − t) + t2 (c + d) ≤ 2. …sing the ewEqw inequ—lityD ‡e get c + d ≤ (c + d)2 + 4 4 = (4 − 2t2 + 2cd) + 4 4 = 4 − t2 + cd 2 . ƒoD it is enough to ™he™k th—t 4tcd(1 − t) + t2 (4 − t2 + cd) ≤ 4, or tcd(4 − 3t) ≤ (2 − t2 )2 . ƒin™e 2 − t2 = c2 +d2 2 ≥ cd, ‡e h—ve (2 − t2 )2 − tcd(4 − 3t) ≥ cd(2 − t2 ) − tcd(4 − 3t) = 2cd(t − 1)2 ≥ 0. TI
  • 61. „he Solution is ™ompletedF VIF vet a, b, c ˜e positive re—l num˜er F€roveX cyc 3 (a2 + ab + b2)2 ≤ 3 3 cyc (2a2 + bc) 2 SolutionX cyc 3 (a2 + ab + b2)2 ≤ 3 3 cyc (2a2 + bc) 2 ↔ cyc 3 3(a2 + ab + b2)2 ≤ 3 9 cyc (2a2 + bc) 2 fy holder9s inequ—lityX cyc 3 3(a2 + ab + b2)2 ≤ 3 9( cyc (a2 + ab + b2))2 ƒo ‡e must proveX 3 9( cyc (a2 + ab + b2))2 ≤ 3 ( cyc (2a2 + bc)) 2 <=> ( cyc (a2 + ab + b2 ))2 ≤ 3 cyc (2a2 + bc)2 …sing g—u™hyEƒ™h—wrz9s inequ—lityD 3 cyc (2a2 + bc)2 ≥ ( cyc (2a2 + bc))2 = cyc (a2 + ab + b2 )2 FiFh F VPF vet a, b, c > 0Fprove th—tX cyc 1 a2 + bc ≥ cyc 1 a2 + 2bc + ab + bc + ca 2(a2b2 + b2c2 + c2a2) SolutionX IFFF (a2 b2 + b2 c2 + c2 a2 ) cyc 1 a2 + bc = cyc bc + a2 (b2 + c2 − bc) a2 + bc <=> 2 cyc a2 (b2 − bc + c2 ) a2 + bc ≥ ab + bc + ca <=> 2 cyc a2 1 + b2 − bc + c2 a2 + bc ≥ 2(a2 + b2 + c2 ) + ab + bc + ca <=> cyc a2 a2 + bc ≥ 1 + ab + bc + ca 2(a2 + b2 + c2) TP
  • 62. fy g—u™hyEƒ™hw—rzD cyc a2 a2 + bc ≥ (a + b + c)2 cyc a2 + cyc bc = 1 + ab + bc + ca cyc a2 + cyc bc ≥ 1 + ab + bc + ca 2(a2 + b2 + c2) FiFh PFF „— ™âX (a2 b2 + b2 c2 + c2 a2 ) cyc 1 a2 + bc = cyc b2 + c2 − bc b2 + c2 − bc a2 + bc <=> 2(a2 + b2 + c2 ) − cyc bc b2 + c2 − bc a2 + bc ≤ 3 2 (a2 + b2 + c2 ) r—y l 2 cyc bc b2 + c2 − bc a2 + bc ≥ a2 + b2 + c2 fy ewEqw9s inequ—lityX 2(b2 + c2 − bc) ≥ b2 + c2 end ‡e will proveX cyc bc(b2 + c2 ) a2 + bc ≥ a2 + b2 + c2 fy g—u™hyEƒ™hw—rz9s inequ—lityX LHS ≥ ( ab √ a2 + b2)2 bc(a2 + bc) ( bc b2 + c2)2 ≥ ( a2 )(abc a + a2 b2 ) …sing g—u™hyEƒ™hw—rzD a2 + b2 a2 + c2 ≥ a2 + bc <=> abc(a3 + b3 + c3 + 3abc − a2 b ≥ 0) it is trueF VQF if —D˜D™ —re positive re—l num˜ersD then a2 (b + c) b2 + c2 + b2 (c + a) c2 + a2 + c2 (a + b) a2 + b2 ≥ a + b + c. pirst SolutionF ‡e h—ve a2 (b + c) b2 + c2 − a = ab(a − b) − ca(c − a) b2 + c2 = ab(a − b) 1 b2 + c2 − 1 c2 + a2 = ab(a + b)(a − b)2 (a2 + c2)(b2 + c2) ≥ 0. TQ
  • 63. „husD it follows th—t a2 (b + c) b2 + c2 − a ≥ 0, or a2 (b + c) b2 + c2 + b2 (c + a) c2 + a2 + c2 (a + b) a2 + b2 ≥ a + b + c, whi™h is just the desired inequ—lityF iqu—lity holds if —nd only if a = b = c. ƒe™ond SolutionF r—ving in view of the identity a2 (b + c) b2 + c2 = (b + c)(a2 + b2 + c2 ) b2 + c2 − b − c, ‡e ™—n write the desired inequ—lity —s b + c b2 + c2 + c + a c2 + a2 + a + b a2 + b2 ≥ 3(a + b + c) a2 + b2 + c2 . ‡ithout loss of gener—lityD —ssume th—t a ≥ b ≥ cF ƒin™e a2 + c2 ≥ b2 + c2 —nd b + c b2 + c2 − a + c a2 + c2 = (a − b)(ab + bc + ca − c2 ) (a2 + c2)(b2 + c2) ≥ 0, ˜y ghe˜yshev9s inequ—lityD ‡e h—ve [(b2 + c2 ) + (a2 + c2 )] b + c b2 + c2 + a + c a2 + c2 ≥ 2[(b + c) + (a + c)], or b + c b2 + c2 + a + c a2 + c2 ≥ 2(a + b + 2c) a2 + b2 + 2c2 . „hereforeD it suffi™es to prove th—t 2(a + b + 2c) a2 + b2 + 2c2 + a + b a2 + b2 ≥ 3(a + b + c) a2 + b2 + c2 , whi™h is equiv—lent to the o˜vious inequ—lity c(a2 + b2 − 2c2 )(a2 + b2 − ac − bc) (a2 + b2)(a2 + b2 + c2)(a2 + b2 + 2c2) ≥ 0. Solution Q xote th—t from g—u™hyEƒ™hw—rtz inequ—lity ‡e h—ve cyc a2 (b + c) (b2 + c2) ≥ cyc cyc a2 (b + c) 2 cyc a2(b + c)(b2 + c2) „herefore it suffi™es to show th—t cyc a2 (b + c) 2 ≥ (a + b + c) cyc a2 (b + c)(b2 + c2 ) efter exp—nsion —nd using the ™onvention p = a + b + c; q = ab + bc + ca; r = abc this is equiv—lent to withX ⇔ r(2p3 + 9r − 7pq) ≥ 0 TR
  • 64. futD sin™e @from trivi—l inequ—lityA ‡e h—vep2 − 3q ≥ 0D hen™e it suffi™es to show th—t p3 + 9r ≥ 4pq, whi™h follows from ƒ™hur9s inequ—lityF iqu—lity o™™urs if —nd only if —a˜a™ or when a = b; c = 0 —nd its ™y™li™ permut—tionsF VRF if —D˜D™ —re positive num˜ers su™h th—t a + b + c = 3 then a 3a + b2 + b 3b + c2 + c 3c + a2 ≤ 3 4 . SolutionX is equiv—lent to a 3a + b2 ≤ 3 4 3a 3a + b2 − 1 ≤ − 3 4 or b2 b2 + 3a ≥ 3 4 fy g—u™hy ƒ™hw—rz inequ—lityD ‡e h—ve LHS ≥ (a2 + b2 + c2 )2 a4 + (a + b + c) ab2 it suffi™es to prove 4(a2 + b2 + c2 )2 ≥ 3 a4 + 3 a2 b2 + 3 ab3 + 3 a2 bc ⇔ (a2 + b2 + c2 )2 − 3 ab3 + 3( a2 b2 − a2 bc) ≥ 0 fy †—sg9s inequ—lityD ‡e h—ve (a2 + b2 + c2 )2 − 3 ab3 ≥ 0 fy em Eqw inequ—lityD a2 b2 − a2 bc ≥ 0 VSF if a ≥ b ≥ c ≥ d ≥ 0 —nd a + b + c + d = 2, then ab(b + c) + bc(c + d) + cd(d + a) + da(a + b) ≤ 1. SolutionX pirstDlet us prove — lemm—X vemm—X por —ny a + b + c + d = 2 —nd a ≥ b ≥ c ≥ d ≥ 0 a2 b + b2 c + c2 d + d2 a ≥ ab2 + bc2 + cd2 + da2 Solution of lemm—X vet F(a) = (b − d)a2 + (d2 − b2 )a + b2 c + c2 d − bc2 − cd2 F (a) = (b − d)(2a − b − d) ≥ 0 <=> F(a) ≥ F(b) = (c − d)b2 + (d2 − c2 )b + cd(c − d) TS
  • 65. F (b) = (c − d)(2b − c − d) ≥ 0 <=> F(b) ≥ F(c) = 0 xowDlet us turn ˜—™k the Solution of the pro˜lemF prom lemm— ‡e h—veX a2 b+b2 c+c2 d+d2 a+ab2 +bc2 +cd2 +da2 +2(abc+bcd+cda+dab) ≥ 2(ab2 +bc2 +cd2 +da2 +abc+bcd+cda+dab it follows th—tY (a + b + c + d)(a + c)(b + d) ≥ 2(ab(b + c) + bc(c + d) + cd(d + a) + da(a + b)) futD ˜y ewEqw inequ—lityX (a + c)(b + d) = (a + c)(b + d) ≤ (a + b + c + d)2 4 = 1 VTF if x, y, z, p, q ˜e nonneg—tive re—l num˜ers su™h th—t (p + q)(yz + zx + xy) > 0 €rove th—tX 2(p + q) (y + z)(py + qz) + 2(p + q) (z + x)(pz + qx) + 2(p + q) (x + y)(px + qy) ≥ 9 yz + xy + zx SolutionX cyc 2(p + q) (y + z)(py + qz) − 9 yz + zx + xy ≡ F(x, y, z) (y + z)(z + x)(x + y)(py + qz)(pz + qx)(px + qy)(yz + zx + xy) . F(x, y, z) = F(x, x + s, x + t) = 16x4 p3 + q3 s2 − st + t2 +x3 16(p + q) p2 + q2 (s + t)(s − t)2 + (p − 2q)2 (15p + 7q) +5q2 (p + q)s2 t + (q − 2p)2 (15q + 7p) + 5p2 (q + p) st2 +x2 2(s − t)2 [p2 (p + 47q)s2 + 51pq(p + q)st + q2 (q + 47p)t2 ] 3 + 2[(5p + q)(5p − 12q)2 + 6q3 ]s4 75 + [(77p − 145q)2 (7918p + 6699q) + 3003p3 + 14297pq2 ]s3 t 5633859 + 34(p + q)(p − q)2 s2 t2 3 + [(77q − 145p)2 (7918q + 6699p) + 3003q3 + 14297qp2 ]st3 5633859 + 2[(5q + p)(5q − 12p)2 + 6p3 ]t4 75 +x [(2p + 5q)s − (2q + 5p)t] 2 4pq(p + q)s3 (2p + 5q)2 + (4p4 + 40p3 q + 65p2 q2 + 113pq3 + 30q4 )s2 t (2p + 5q)3 + (4q4 + 40q3 p + 65q2 p2 + 113qp3 + 30p4 )st2 (2q + 5p)3 + 4qp(q + p)t3 (2q + 5p)2 + s3 t2 (2p + 5q)2(2q + 5p)3 (2p − 3q)2 1505p6 + 9948p5 q + 19439p4 q2 TT
  • 66. +16869p3 q3 + 6709p2 q4 + 852pq5 + 117q6 +4960p7 q + 6800p6 q2 + p5 q3 + 4p4 q4 + 5p3 q5 + 8q6 p2 + 4pq7 + 7q8 + s2 t3 (2p + 5q)3(2q + 5p)2 (2q−3p)2 1505q6 + 9948q5 p + 19439q4 p2 + 16869q3 p3 + 6709q2 p4 + 852qp5 + 117p6 +4960q7 p + 6800q6 p2 + q5 p3 + 4q4 p4 + 5q3 p5 + 8p6 q2 + 4qp7 + 7p8 +st [(13p + 47q)s − (13q + 47p)t]2 2pq(p + q)s2 (13p + 47q)2 + 49pq(p + q)st 6(743p2 + 6914pq + 743q2) + 2qp(q + p)t2 (13q + 47p)2 +(p − q)2 st q(324773p3 + 3233274p2 q + 836101pq2 + 419052q3 )s2 6(13p + 47q)(743p2 + 6914pq + 743q2) + 2(p + q)(p − q)2 (832132509p4 + 9284734492p3 q + 9070265998p2 q2 + 9284734492pq3 + 832132509q4 )st 3(13p + 47q)2(13q + 47p)2(743p2 + 6914pq + 743q2) + p(324773q3 + 3233274q2 p + 836101qp2 + 419052p3 )t2 6(13q + 47p)(743p2 + 6914pq + 743q2) ≥ 0, whi™h is ™le—rly true for x = min{x, y, z}. VUF vet —D˜D™ ˜e positive num˜ers su™h th—t ab + bc + ca = 3. €rove th—t 1 a + b + 1 b + c + 1 c + a ≥ 1 + 3 2(a + b + c) . SolutionX IFFFFFvet f(a, b, c) = 1 a+b + 1 b+c + 1 c+a − 1 − 3 2(a+b+c) —nd a = min{a, b, c}. „hen f(a, b, c) − f a, (a + b)(a + c) − a, (a + b)(a + c) − a = = √ a + b − √ a + c 2 1 (a + b)(a + c) − 1 2(b + c) (a + b)(a + c) − a + 3 2(a + b + c) 2 · (a + b)(a + c) − a ≥ ≥ √ a + b − √ a + c 2 1 4bc − 1 2(b + c) · √ bc + 2 3(b + c)2 ≥ 0 sin™eD (a + b)(a + c) ≥ a + √ bc —nd 2 · (a + b)(a + c) − a ≤ a + b + c ≤ 3(b+c) 2 . „husD rem—in to prove th—t f(a, b, b) ≥ 0, whi™h equiv—lent to (a − b)2 (2a3 + 9a2 b + 12ab2 + b3 ) ≥ 0. PFFFFFFF „he inequ—lity is equiv—lent toX 1 a + b + 1 b + c + 1 c + a ≥ 3 3(ab + bc + ca) + 3 2(a + b + c) ↔ 1 a + b + 1 b + c + 1 c + a − 9 2(a + b + c) ≥ 3 3(ab + bc + ca) − 3 a + b + c TU
  • 67. ↔ (a − b)2 2(a + c)(b + c)(a + b + c) + (b − c)2 2(a + b)(a + c)(a + b + c) + (c − a)2 2(b + c)(a + b)(a + b + c) ≥ 3[(a − b)2 + (b − c)2 + (c − a)2 ] 2(a + b + c + 3(ab + bc + ca))(a + b + c)( 3(ab + bc + ca)) ↔ (a − b)2 .M + (b − c)2 .N + (c − a)2 .P ≥ 0 with X M = (a + b)[(a + b + c) + 3(ab + bc + ca)] 3(ab + bc + ca) − 3(a + b)(b + c)(c + a) N = (b + c)[(a + b + c) + 3(ab + bc + ca)] 3(ab + bc + ca) − 3(a + b)(b + c)(c + a) P = (c + a)[(a + b + c) + 3(ab + bc + ca)] 3(ab + bc + ca) − 3(a + b)(b + c)(c + a) ƒuppose th—tXa ≥ b ≥ cF ƒo ‡e h—veX M = (a + b)([(a + b + c) + 3(ab + bc + ca)] 3(ab + bc + ca) − 3(b + c)(c + a)) ≥ 0 fe™—use (a + b + c) 3(ab + bc + ca) ≥ 3c2 P = (a + c)([(a + b + c) + 3(ab + bc + ca)] 3(ab + bc + ca) − 3(a + b)(b + c)) ≥ 0 fe™—use (a + b + c) 3(ab + bc + ca) ≥ 3b2 ƒo ‡e must proveX N + P ≥ 0 it –s equiv—lent toX X = [(a + b + c) + 3(ab + bc + ca)] 3(ab + bc + ca)(a + b + 2c) − 6(a + b)(b + c)(c + a) ≥ 0 €ut x = a + b + c; y = 3(ab + bc + ca) X ≥ [(a + b + c) + 3(ab + bc + ca)] 3(ab + bc + ca)(a + b + c) − 6(a + b + c)(ab + bc + ca) ↔ x2 y ≥ xy2 ↔ x ≥ y @it –s true for —ll positive num˜ers —D˜D™AF VVF vet a, b, c˜e positive re—l num˜er F €rove th—tX 1 a + b + 1 b + c + 1 c + a ≥ a + b + c 2(ab + bc + ca) + 3 a + b + c SolutionX vet put p = a + b + c, q = ab + bc + ca, r = abcD „his inequ—lity is equiv—lent toX p2 + q pq − r ≥ p 2q + 3 p ⇐⇒ p2 + 3 3p − r ≥ p 6 + 3 p fy exp—nding expression ‡e h—veX (p2 + 3)6p − p2 (3p − r) − 18(3p − r) ≥ 0 TV
  • 68. ⇐⇒ 3p3 + p2 r − 36p + 18r ≥ 0 prom the illEknown inequ—lityD the third degree ƒ™hur9s inequ—lity st—tesX p3 − 4pq + 9r ≥ 0 ⇐⇒ p3 − 12p + 9r ≥ 0 ‡e h—veX ⇐⇒ 3p3 + p2 r − 36p + 18r ≥ 0 ⇐⇒ 3(p3 − 12p + 9r) + r(p2 − 9) ≥ 0 yn the other h—ndD ‡e h—veX r(p2 − 9) ≥ 0 ⇐⇒ (a − b)2 + (b − c)2 + (c − a)2 ≥ 0 VWF if x, y, z —re nonneg—tive re—l num˜ers su™h th—t x + y + z = 3D then 4( √ x + √ y + √ z) + 15 ≤ 9( x + y 2 + y + z 2 + z + x 2 ) SolutionX „he inequ—lity9s true when x = 3, y = z = 0F if no two of x, y, z —re HD set x = a2 et™F it ˜e™omes 8(a + b + c) + 10 3a2 + 3b2 + 3c2 ≤ 9 2a2 + 2b2 + 2b2 + 2c2 + 2c2 + 2a2 ⇐⇒ 10 3a2 + 3b2 + 3c2 − (a + b + c) ≤ 9 cyc 2a2 + 2b2 − (a + b) ⇐⇒ 10 cyc (a − b)2 a + b + c + √ 3a2 + 3b2 + 3c2 ≤ 9 cyc (a − b)2 a + b + √ 2a2 + 2b2 ⇐⇒ cyc 9 a + b + √ 2a2 + 2b2 − 10 a + b + c + √ 3a2 + 3b2 + 3c2 (a − b)2 ≥ 0 xow e—™h term is nonneg—tiveD for in f—™tD 9(a + b + 3a2 + 3b2) ≥ 10(a + b + 2a2 + 2b2) ˜e™—use 9 √ 3 √ 2 − 10 2a2 + 2b2 > 2a2 + 2b2 ≥ a + b WHF if a, b, c —re nonneg—tive re—l num˜ers su™h th—t a + b + c = 3D then 1 4a2 + b2 + c2 + 1 4b2 + c2 + a2 + 1 4c2 + a2 + b2 ≤ 1 2 SolutionX 1 4a2 + b2 + c2 + 1 4b2 + c2 + a2 + 1 4c2 + a2 + b2 ≤ 1 2 ⇔ ⇔ sym (a6 − 4a5 b + 13a4 b2 − 2a4 bc − 6a3 b3 − 12a3 b2 c + 10a2 b2 c2 ) ≥ 0 ⇔ ⇔ cyc (a − b)2 (2c4 + 2(a2 − 4ab + b2 )c2 + a4 − 2a3 b + 4a2 b2 − 2ab3 + b4 ) ≥ 0, whi™h true ˜e™—use (a2 − 4ab + b2 )2 − 2(a4 − 2a3 b + 4a2 b2 − 2ab3 + b4 ) = TW
  • 69. = −(a − b)2 (a2 + 6ab + b2 ) ≤ 0. WIF vet —D˜D™ ˜e nonneg—tive re—l num˜ers su™h th—ta + b + c = 3. €rove th—t a2 b 4 − bc + b2 c 4 − ca + c2 a 4 − ab ≤ 1. SolutionF ƒin™e 4a2 b 4 − bc = a2 b + a2 b2 c 4 − bc the inequ—lity ™—n ˜e written —s abc ab 4 − bc ≤ 4 − a2 b. …sing the illEknown inequ—lity a2 b + b2 c + c2 a + abc ≤ 4D ‡e get 4 − (a2 b + b2 c + c2 a) ≥ abc, —nd hen™eD it suffi™es to prove th—t abc ab 4 − bc ≤ abc, or equiv—lentlyD ab 4 − bc + bc 4 − ca + ca 4 − ab ≤ 1. ƒin™e ab + bc + ca ≤ (a + b + c)2 3 = 3, ‡e get ab 4 − bc ≤ ab 4 3 (ab + bc + ca) − bc = 3ab 4ab + bc + 4ca . „hereforeD it is enough to ™he™k th—t x 4x + 4y + z + y 4y + 4z + x + z 4z + 4x + y ≤ 1 3 , where x = ab, y = ca —nd z = bcF „his is — illEknown inequ—lityF WPF if a, b, c —re nonneg—tive re—l num˜ersD then a3 + b3 + c3 + 12abc ≤ a2 a2 + 24bc + b2 b2 + 24ca + c2 c2 + 24ab SolutionX a2 + 24bc 7 a2 + b2 + c2 + 8(bc + ca + ab) 2 − 7a3 + 8a2 (b + c) + 7a b2 + c2 + 92abc + 48bc(b + c) 2 = 24bc (b − c)2 109a2 + 77ab + 77ac + 49b2 + 89bc + 49c2 +(b + c − 2a)2 (25bc + 7ab + 7ca) ≥ 0 =⇒ a2 a2 + 24bc ≥ a2 [7a3 + 8a2 (b + c) + 7a(b2 + c2 ) + 92abc + 48bc(b + c)] 7(a2 + b2 + c2) + 8(bc + ca + ab) UH
  • 70. = a3 + b3 + c3 + 12abc. WQF where a, b, c, d —re nonneg—tive re—l num˜ersF€rove the inequ—lityX a 9a2 + 7b2 + b 9b2 + 7c2 + c 9c2 + 7d2 + d 9d2 + 7a2 ≥ (a + b + c + d)2 SolutionX fy g—u™hyƒ™hw—rzD ‡e h—ve 4 a 9a2 + 7b2 ≥ a(9a + 7b) it suffi™es to prove th—t 9(a2 +b2 +c2 +d2 )+7(a+c)(b+d) ≥ 4(a+b+c+d)2 = 4(a+c)2 +4(b+d)2 +8(a+c)(b+d) ⇔ 9(a2 + b2 + c2 + d2 ) ≥ 4(a + c)2 + 4(b + d)2 + (a + c)(b + d) whi™h is true ˜e™—use a2 + b2 + c2 + d2 ≥ (a + c)(b + d) 2(a2 + b2 + c2 + d2 ) = 2(a2 + c2 ) + 2(b2 + d2 ) ≥ (a + c)2 + (b + d)2 WRF vet a, b, c ˜e positive F €rove th—tF 3a2 7a2 + 5(b + c)2 ≤ 1 ≤ a2 a2 + 2(b + c)2 SolutionX „he right h—nd is trivi—l ˜y the rolder inequ—lity sin™e   a a2 + 2 (b + c) 2   2 a a2 + 2 (b + c) 2 ≥ a 3 end ( a) 3 ≥ a a2 + 2 (b + c) 2 ⇔ ab (a + b) ≥ 6abcF por the left h—nd ˜y the g—u™hy ƒ™hw—rz inequ—lity ‡e h—ve   a 7a2 + 5 (b + c) 2   2 ≤ a a 7a2 + 5 (b + c) 2 essume a + b + c = 3D denote ab + bc + ca = 9−q2 3 , r = abc then ‡e will prove a 12a2 − 30a + 45 ≤ 1 9 ⇔ f (r) = 48r2 + 222 + 52q2 r + 20q4 + 75q2 − 270 ≥ 0 ‡e h—ve r ≥ max 0, (3 + q) 2 (3 − 2q) 27 „hereforD if q ≥ 3 2 UI
  • 71. then get r ≥ 0 —nd f (r) ≥ f (0) = 20 q − 3 2 q + 3 2 q2 + 6 ≥ 0 if q ≤ 3 2 then get r ≥ (3+q)2 (3−2q) 27 —nd f (r) ≥ f (3 + q) 2 (3 − 2q) 27 = q2 (2q − 3) 96q3 − 396q2 + 2322q − 5103 729 ≥ 0 ‡e h—ve doneF iqu—lity holds if —n only if a = b = c or a = b, c = 0 or —ny ™y™li™ permut—E tionsF WSF if a, b, c, d, e —re positive re—l num˜ers su™h th—t a + b + c + d + e = 5D then 1 a + 1 b + 1 c + 1 d + 1 e + 20 a2 + b2 + c2 + d2 + e2 ≥ 9 SolutionD sym f(a, b) me—ns f(a, b)+f(a, c)+f(a, d)+f(a, e)+f(b, c)+f(b, d) +f(b, e)+ f(c, d) + f(c, e) + f(d, e)F ‡e will firstly rewrite the inequ—lity —s 1 a + 1 b + 1 c + 1 d + 1 e − 25 a + b + c + d + e ≥ 4 − 4(a + b + c + d + e)2 5(a2 + b2 + c2 + d2 + e2) . …sing the identities (a + b + c + d + e) 1 a + 1 b + 1 c + 1 d + 1 e − 25 = sym (a − b)2 ab —nd 5(a2 + b2 + c2 + d2 + e2 ) − (a + b + c + d + e)2 = sym(a − b)2 ‡e ™—n rewrite —g—in the inequ—lity —s 1 a + b + c + d + e sym (a − b)2 ab ≥ 4 5 × sym(a − b)2 a2 + b2 + c2 + d2 + e2 or sym Sab(a − b)2 ≥ 0 where Sxy = 1 xy − 4 a2 + b2 + c2 + d2 + e2 for —ll x, y ∈ {a, b, c, d, e}F essume th—t a ≥ b ≥ c ≥ d ≥ e > 0F ‡e will show th—t Sbc + Sbd ≥ 0 —nd Sab + Sac + Sad + Sae ≥ 0F indeedD ‡e h—ve Sbc + Sbd = 1 bc + 1 bd − 8 a2 + b2 + c2 + d2 + e2 > 1 bc + 1 bd − 8 b2 + b2 + c2 + d2 ≥ 1 bc + 1 bd − 8 2bc + 2bd ≥ 0 —nd Sab+Sac+Sad+Sae = 1 ab + 1 ac + 1 ad + 1 ae − 16 a2 + b2 + c2 + d2 + e2 ≥ 16 a(b + c + d + e) − 16 a2 + 1 4 (b + c + d + e)2 ≥ 0. ren™eD with noti™e th—t Sbd ≥ Sbc —nd Sae ≥ Sad ≥ Sac ≥ Sab UP
  • 72. ‡e h—ve Sbd ≥ 0 —nd Sae ≥ 0, Sae + Sad ≥ 0, Sae + Sad + Sac ≥ 0F „husD Sbd(b − d)2 + Sbc(b − c)2 ≥ (Sbd + Sbc)(b − c)2 ≥ 0(1) —nd Sae(a−e)2 +Sad(a−d)2 +Sac(a−c)2 +Sab(a−b)2 ≥ (Sae+Sad)(a−d)2 +Sac(a−c)2 +Sab(a−b)2 ≥ (Sae + Sad + Sac)(a − c)2 + Sab(a − b)2 ≥ (Sae + Sad + Sac + Sab)(a − b)2 ≥ 0(2) yn the other h—ndD Sbe ≥ Sbd ≥ 0 —nd Sde ≥ Sce ≥ Scd ≥ Sbd ≥ 0(3)F „hereforeD from @IAD @PA —nd @QA ‡e get sym Sab(a − b)2 ≥ 0. iqu—lity o™™urs when a = b = c = d = e or a = 2b = 2c = 2d = 2eF WTF vet a, b, c ˜e nonneg—tive re—l num˜ersF €rove th—t 1 + 3abc a2b + b2c + c2a ≥ 2(ab + bc + ca) a2 + b2 + c2 SolutionX ‡e ™—n prove it —s followX ‚ewriting the inequ—lity —s 3abc a2b + b2c + c2a ≥ 2(ab + bc + ca) − a2 − b2 − c2 a2 + b2 + c2 if 2(ab + bc + ca) ≤ a2 + b2 + c2 D it is trivi—lF if 2(ab + bc + ca) ≥ a2 + b2 + c2 D —pplying ƒ™hur9s inequ—lityX 3abc ≥ (a + b + c)[2(ab + bc + ca) − a2 − b2 − c2 ] 3 it suffi™es to show th—t (a + b + c)[2(ab + bc + ca) − a2 − b2 − c2 ] 3(a2b + b2c + c2a) ≥ 2(ab + bc + ca) − a2 − b2 − c2 a2 + b2 + c2 (a + b + c)(a2 + b2 + c2 ) ≥ 3(a2 b + b2 c + c2 a) b(a − b)2 + c(b − c)2 + a(c − a)2 ≥ 0 @„rueA WUF vet a, b, c ˜e positive re—l num˜ers su™h th—t abc = 1F €rove th—t 1 a + 1 b + 1 c + 6 a + b + c ≥ 5. SolutionX IFFFF‡vyq —ssume a ≥ b ≥ cF vet f(a, b, c) = 1 a + 1 b + 1 c + 6 a + b + c f(a, b, c) ≥ f(a, √ bc, √ bc) <=> ( √ b − √ c)2 bc(a + b + c)(a + 2 √ bc) ((a + b + c)(a + 2 √ bc) − 6bc) ≥ 0 ⇔ UQ
  • 73. (a + b + c)(a + 2 √ bc) ≥ 6bc. es a ≥ b + c 2 ≥ √ bc so (a + b + c)(a + 2 √ bc) ≥ 9bc ≥ 6bc hen™e the —˜ove inequ—lity is trueF f( 1 x2 , x, x) ≥ 5 ⇔ (x − 1)2 (2x4 + 4x3 − 4x2 − x + 2) ≥ 0. es 2x4 + 4x3 − 4x2 − x + 2 > 0 if x > 0D so the —˜ove inequ—lity is trueF „herefore f(a, b, c) ≥ f(a, √ bc, √ bc) = f( 1 bc , √ bc, √ bc) ≥ 5. ab FiFh PFFFFFFFFF essume th—t a ≥ b, cF ‡rite x = √ a, y = b c F „hen x ≥ 1 —nd the inequ—lity (ab + bc + ca)(a + b + c) + 6 ≥ 5(a + b + c) ˜e™omes x3 (y + y−1 ) − 5x2 + (y2 + 9 + y−2 ) − 5x−1 (y + y−1 ) + x−3 (y + y−1 ) ≥ 0 „his ™—n ˜e seper—ted —s 2x3 − 5x2 + 11 − 10x−1 + 2x−3 ≥ 0 —nd x3 (y + y−1 − 2) + (y2 + y−2 − 2) − 5x−1 (y + y−1 − 2) + x−3 (y + y−1 − 2) ≥ 0 „he first one is e—syF e˜out the se™ond oneD xote th—t x3 + x−3 ≥ 2 ≥ 2x−1 —nd (y2 + y−2 − 2) ≥ 3(y1 + y−1 − 2) ≥ 3x−1 (y1 + y−1 − 2) sin™e y2 − 3y + 4 − 3y−1 + y−2 = (y − 1)2 (y − 1 + y−1 ) QFFFFFFFFF vem— of †—ile girto—je (a + b) (b + c) (c + a) + 7 ≥ 5 (a + b + c) @g—n e—sy prove ˜y w†A fut (a + b) (b + c) (c + a) = a2 b + a2 c + b2 c + b2 a + c2 a + c2 b + 2abc = a2 b + a2 c + b2 c + b2 a + c2 a + c2 b + 3abc − abc UR
  • 74. = (a + b + c) (bc + ca + ab) − abc = (a + b + c) (bc + ca + ab) − 1 where ‡e h—ve used th—t —˜™ a I in the l—st step of our ™—l™ul—tionF „husD ‡e h—ve ((a + b + c) (bc + ca + ab) − 1) + 7 ≥ 5 (a + b + c) Y in other wordsD (a + b + c) (bc + ca + ab) + 6 ≥ 5 (a + b + c) …pon division ˜y — C ˜ C ™D this ˜e™omes (bc + ca + ab) + 6 a + b + c ≥ 5 pin—llyD sin™e abc = 1D ‡e h—ve bc = 1 a D ca = 1 b —nd ab = 1 c D —nd thus ‡e get 1 a + 1 b + 1 c + 6 a + b + c ≥ 5 WVF vet a, b, c > 0 —nd with —ll k ≥ −3/2 F €rove the inequ—lityX a3 + (k + 1)abc b2 + kbc + c2 ≥ a + b + c SolutionX yur inequ—lity is equiv—lent to a(a2 + bc − b2 − c2 ) b2 + kbc + c2 + b(b2 + ca − c2 − a2 ) c2 + kca + a2 + c(c2 + ab − a2 − b2 ) a2 + kab + b2 ≥ 0, (a2 −b2 ) a b2 + kbc + c2 − b a2 + kac + c2 +c a(b − c) b2 + kbc + c2 + b(a − c) a2 + kac + c2 + c2 + ab − a2 − b2 a2 + kab + b2 ≥ 0. prom nowD ‡e see th—t a b2 + kbc + c2 − b a2 + kac + c2 = (a − b)(a2 + b2 + c2 + ab + kac + kbc) (b2 + kbc + c2)(a2 + kac + c2) , c2 + ab − a2 − b2 a2 + kab + b2 = (c − a)(c − b) − a(b − c) − b(a − c) − (a − b)2 a2 + kab + b2 , a(b − c) b2 + kbc + c2 − a(b − c) a2 + kab + b2 = a(a − c)(b − c)(a + c + kb) (a2 + kab + b2)(b2 + kbc + c2) , b(a − c) a2 + kac + c2 − b(a − c) a2 + kab + b2 = a(a − c)(b − c)(b + c + ka) (a2 + kab + b2)(a2 + kac + c2) . „hereforeD the inequ—lity ™—n ˜e rewritten —s A(a − b)2 + c(a − c)(b − c) a2 + kab + b2 B ≥ 0, where A = (a + b)(a2 + b2 + c2 + ab + kac + kbc) (a2 + kac + c2)(b2 + kbc + c2) − c a2 + kab + b2 , —nd B = a(a + c + kb) b2 + kbc + c2 + b(b + c + ka) a2 + kac + c2 + 1. US